Professional Responsibility MC Quizzes 2022

¡Supera tus tareas y exámenes ahora con Quizwiz!

Attorney Saint works as a public defender and feels completely overwhelmed because of her heavy caseload. She has started to double-book two or more trials for the same day, counting on one or more of them to resolve in a plea-bargain agreement before trial. Most clients meet her for only a few minutes before their plea-bargaining session, and she emphatically insists with her clients that they accept the prosecutor's second or third offer for a deal. When her cases go to trial, she must waive voir dire entirely and often does no factual investigation or case research—yet she still wins acquittals in many cases because she is very gifted at destroying the credibility of hostile witnesses during cross-examination. It has become increasingly clear to her that she cannot meet her basic ethical obligations in representing even her existing clients—she does not have time to do competent legal research, in-depth factual investigation, or keep her clients updated about developments in their cases. Must Attorney Saint withdraw from representing some of her current clients? (A) Yes, if she cannot fulfill her ethical duties, she must not continue representation of her current clients. (B) Yes, as long as her clients and the court both consent to her withdrawal. (C) No, she is not required to withdraw, but she may seek to withdraw if it would not materially prejudice a client and the court allows. (D) No, because as long as she takes no new clients, her current cases will resolve soon and her caseload will become more workable.

(A) Yes, if she cannot fulfill her ethical duties, she must not continue representation of her current clients. The correct answer is A. ABA Formal Ethics Op. 06-441 clarifies that a lawyer must withdraw from representation of some clients if necessary to provide competent and diligent representation to the remaining ones. Answer Ais therefore the correct answer. Even though each individual defendant might have been better off with her representation, she cannot represent all of them and has to make some choices. Answer B is incorrect, even though it correctly acknowledges that withdrawal is appropriate, because client consent is not required for a lawyer to withdraw from the representation. Answer C incorrectly presents withdrawal in this situation as permissive—instead, withdrawal is mandatory if the lawyer does not have time to provide competent and diligent representation, which includes a reasonable amount of legal research, factual investigation, and timely communication (updates) to the clients. Answer D is wrong because it contradicts the position the ABA Ethics Committee has taken on this issue—if the lawyer cannot provide adequate representation for the current clients, she cannot continue representing all of them, much less take on new ones.

While conducting research on a litigation matter, Attorney Bentley finds a very new case from the highest court in a neighboring jurisdiction that is directly adverse to his client's legal position in the case. The issue presents a case of first impression in Attorney Bentley's own jurisdiction, where the case is taking place. The opposing party did not mention the case in its briefs, and Attorney Bentley realizes that the opposing party's lawyer has been recycling his firm's briefs for this type of case for several years without updating his research. Does Attorney Bentley have an ethical duty to disclose the unfavorable authority to the court? (A) No, because it would be a breach of the attorney's duty of loyalty to his own client to disclose a case that undermines their position (B) No, because the case is not controlling authority in that jurisdiction (C) Yes, because a lawyer must disclose to the tribunal legal authority known to the lawyer to be directly adverse to the position of the client and not disclosed by opposing counsel (D) Yes, because it is common for litigators to recycle their briefs for years at a time, and lawyers should help each other with updating relevant legal research

(B) No, because the case is not controlling authority in that jurisdiction The correct answer is B. Model Rule 3.3(a)(2) contains a requirement that seems counterintuitive to many non-lawyers—a lawyer must disclose any unfavorable controlling authority (statutes or caselaw) that the lawyer finds, if the other side has not already disclosed it to the court. This is an exception to the usual premise of the adversarial system—that each party is responsible for presenting the best evidence and arguments for their own side, not the opposing side. This rule helps keep lower courts consistent with the decisions of the appellate courts in their jurisdiction and also helps protect judges from embarrassing mistakes that would result in reversal by the higher court later on. The rule applies only to controlling law or precedent in the same jurisdiction, though not to "persuasive authority" from sister jurisdictions. Answer B is therefore the right answer because the new case is from a neighboring jurisdiction, and therefore the attorney has no duty to disclose it. If it were from the same jurisdiction, the duty would apply. Answer A is incorrect because Rule 3.3(a)(2) carves out an important exception to the normal duty of loyalty to one's own client—if the case had been controlling authority from the lawyer's own jurisdiction, there would have been a duty to disclose it to the tribunal, even though this would run counter to the client's best interests. Answer C reflects the wording used in the rule, but the rule does not apply to the facts in this question because the case was from another jurisdiction. Answer D is incorrect because the rule does not apply to precedent from neighboring jurisdictions, although Answer D explains one of the most common ways for this situation to arise: the lawyer for the opposing party reuses sections from a previous brief and misses a relevant case that has since been decided.

A commercial real estate developer hired an attorney to secure environmental permits to build a shopping center. The relevant federal and state agencies granted the necessary permits. The attorney's representation of the developer ended once the environmental permits were securely in hand. Nevertheless, construction of the shopping center did not begin immediately, because clearing the land exhausted the developer's initial supply of investment funds. Two years later, the developer found another investor and began preparations for the construction of the shopping center. In the meantime, the residents of the neighborhoods around the proposed shopping center had turned against the project, out of concerns for the increase in traffic and litter that it could bring to the area, as well as the flooding of adjacent yards that would result from the rainwater runoff from a new parking lot. The "Not In My Back Yard" Association (NIMBY) formed and learned that the rezoning of the property by municipal authorities to permit a shopping center was still pending, with an upcoming public hearing on the schedule. NIMBY hired the same attorney to represent the neighbors in opposing the rezoning on the basis of environmental considerations. Under the Model Rules of Professional Responsibility, would it be proper for this attorney to represent the neighbors in this matter? (A) No, because the neighbors are acting in their own self-interest rather than thinking about the greater good that would result from constructing the shopping center, and even among the group of neighbors, there are probably conflicts of interest depending on who lives closest to the proposed shopping center (B) No, because the matters are 'substantially related,' since it is likely that confidential information from the prior representation would materially advance the neighbors' position in the subsequent lawsuit, such as detailed reports about the potential environmental impact of constructing the shopping center (C) Yes, because the attorney's prior representation of the developer involved securing environmental permits from state and federal authorities, and the new representation would involve a rezoning hearing before a municipal authority (D) Yes, because the attorney's representation of the developer terminated more than a year and a day prior to the commencement of the representation of NIMBY so there is no potential for betraying a current client's confidential information by representing the adverse interests of the neighbors

(B) No, because the matters are 'substantially related,' since it is likely that confidential information from the prior representation would materially advance the neighbors' position in the subsequent lawsuit, such as detailed reports about the potential environmental impact of constructing the shopping center The correct answer is B. Comment 3 to Model Rule 1.9 defines the parameters of "substantially related" matters for purposes of analyzing conflicts of interest between successive clients—matters that "involve the same transaction or legal dispute or if there otherwise is a substantial risk that confidential factual information as would normally have been obtained in the prior representation would materially advance the client's position in the subsequent matter." Comment 3 even gives a few brief examples, including one similar to the scenario in this question—a lawyer who obtains environmental permits for one client (such as a commercial developer) cannot represent neighbors challenging the construction project on environmental grounds, due to the overlap in issues in the representation. Thus, Answer B is the correct answer, both because of the risk of exploiting confidential information from the developer in portraying the construction as a threat to the community, and because the lawyer's advocacy in each case would probably be self-contradictory (minimizing the environmental harms to obtain the permits originally, and then emphasizing the harms in order to halt the construction for the NIMBY group). Answer A incorrectly suggests that the self-interest of the parties negates their right to a disinterested lawyer, which is not the case. Most parties hire attorneys to represent their interests, and lawyers must consider the client's own self-interest in screening for conflicts of interest. Answer Cis wrong because it portrays the situation as presenting no conflict of interest merely because different levels of government are involved. The matters here in the two representations are substantially related because they center around the same underlying legal and factual issues. Answer D incorrectly suggests that the mere passage of time eliminates conflicts of interest. This is not the case (there is no statute of limitations on conflicts), and even if time were a factor, the period elapsed here would not be enough to diminish the conflict since it is the same developer and the same construction project at issue.

While working for Conglomerate Corporation as in-house counsel, an attorney discovered that the Chief Financial Officer (CFO) falsified the corporation's quarterly earnings report to boost the firm's share price. Many of the top executives at Conglomerate, including the CFO, receive compensation partly in stock options. The attorney knew that these misrepresented earnings appeared in the filings to the Securities and Exchange Commission and will eventually result in severe regulatory fines or civil liability for the corporation. What should the attorney do in this situation? (A) The attorney should drop the matter unless the SEC makes an inquiry about it. (B) The attorney should start with the CFO, and then take the matter up the chain of command in the organization if necessary, eventually bringing the matter to the Board of Directors if no one in management will address the problem. (C) The attorney should immediately report the matter to the appropriate government authorities without warning the CFO or his friends within the corporation, lest they have an opportunity to destroy evidence. (D) The attorney should confront the CFO, but if the CFO remains recalcitrant, the attorney must drop the matter.

(B) The attorney should start with the CFO, and then take the matter up the chain of command in the organization if necessary, eventually bringing the matter to the Board of Directors if no one in management will address the problem. The correct answer is B. Model Rule 1.13(a) lays down a bright-line rule: "A lawyer employed or retained by an organization represents the organization acting through its duly authorized constituents." In other words, the managers or executives at the corporation are not the client—the entity is the client, and the lawyer's duty is to the entity or organization, not to individuals within it. As a result, when lawyers discover wrongdoing by an employee (including a top manager) that would result in substantial injury to the organization, the lawyer must intervene to protect the entity. Rule 1.13(b) directs attorneys to work up the chain of command, first trying to persuade the wrongdoer to change course or fix the problem, then proceeding up the chain until someone responds. Answer B is therefore the best answer—the lawyer in this situation should proceed to the person who has authority over the CFO (perhaps the CEO or president) and eventually go to the Board of Directors as a last resort. Answer A is incorrect—the lawyer's duty of diligence requires an intervention to protect the corporation, which is the client. Answer C wrongly suggests that the lawyer should skip intermediate steps and act as a whistleblower, breaching client confidentiality and reporting the matter to a government agency. Going to the authorities (and thereby breaching confidentiality) would be permissible under the Model Rules if the Board of Directors also refuses to act—that is, if the lawyer has exhausted all other options—but the lawyer should first try to resolve the problem in-house. Answer D is partly right and partly wrong—confronting the wrongdoer should indeed be the lawyer's first step, but if that proves unfruitful, the lawyer must go to the next higher authority in the organization.

Client Williams hired Attorney McKnight to represent him in a criminal matter. Williams faces charges for abducting a young girl from her home three months ago. McKnight learns from his client that Williams indeed abducted the girl, that the girl is probably still alive and hidden in a secluded location, and that the child was left alone, locked in a rural cottage, with some food and water two weeks ago when police arrested Williams. Williams refuses to disclose the location of the girl to authorities. There is a chance that someone may happen upon the cottage where the girl is trapped and help her. Does Attorney McKnight have a duty to disclose the location of the girl to authorities or the parents in order to save the girl's life? (A) Yes, because the disclosure is reasonably necessary to prevent reasonably certain death or substantial bodily harm (B) Yes, but only if the lawyer discloses the location as an anonymous tip and is reasonably certain that the discovery of the girl will not be prejudicial to the client's case (C) No, the Rules of Professional Conduct do not require the lawyer to reveal the client's misconduct or the girl's location (D) No, because the girl could survive in the cottage until she finds a way to escape or someone happens to find her

(C) No, the Rules of Professional Conduct do not require the lawyer to reveal the client's misconduct or the girl's location The correct answer is C. Comments 7 and 17 for Model Rule 1.6 explain that lawyers do not have to disclose confidential information, even when the disclosure might be permissible to save someone's life, under one of the narrow exceptions to the confidentiality rule. In this question, it is ambiguous whether a disclosure by the lawyer would even fall under the exception making it permissible—it is not entirely clear whether the victim is "reasonably certain" to face death or serious bodily injury confined in a rural cottage, if she indeed has food, water, and shelter from the elements—though nobody would argue that being left alone is "safe" for a young child. Nevertheless, for the question posed, the answer is negative—even if the disclosure were indeed necessary to save the child's life (which would make it permissible for the lawyer), the lawyer is not obligatedunder the Model Rules to do so. Thus, Answer C is the best answer to this question—the lawyer is not required to make the disclosure, even if the disclosure would be permissible, even if this would be the morally correct thing for a person to do. Answer A invokes the "defense of life" exception found in Rule 1.6(b)(1) but misses the point that the exception does not make the disclosure obligatory. Answer B is incorrect both because the disclosure is not required and because there is no exception for anonymous tips—lawyers cannot circumvent the confidentiality rule by leaking the information anonymously. Answer D is incorrect for the same reason as Answer A—it misses the point that the disclosure is not mandatory regardless of whether it would save the child's life.

Attorney Bales has her own law practices and represents criminal defendants in their trials and plea negotiations; she is a bright and energetic lawyer. Most of her clients come from court appointments to represent indigent defendants. Attorney Bales is passionate about defending the rights of indigent defendants, so she takes every case referred to her by the local courts. The volume of cases she handles, due to the number of court appointments she accepts, means that she cannot feasibly keep abreast of changes in the law, investigate and prepare cases, act promptly on behalf of clients, or even communicate effectively with each client. She strongly believes, however, that indigent defendants are better off receiving some legal help than no help at all, so she is unwilling to turn away any cases. Besides, 95% of the criminal cases in her jurisdiction result in plea agreements, which sometimes take only a few hours to negotiate and finalize. Is it permissible for Attorney Bales to continue accepting court appointments, given these facts? (A) Yes, there is a special exception to the workload-limit rules for public defenders, considering the pressing need for representation of indigent defendants. (B) Yes, if most of the defendants would indeed be worse off if they had to represent themselves, so the attorney's minimal representation is better for them than nothing. (C) No, lawyers have an ethical duty to accept judicial appointments for indigent defense. (D) No, lawyers must control their workload so that each matter receives competent, diligent representation

(D) No, lawyers must control their workload so that each matter receives competent, diligent representation Answer D is the best answer. ABA Model Rule 1.3, Comment 2, specifically requires that lawyers manage their workloads so that each client receives fully competent and diligent representation, which is not occurring in this Question. In fact, the ABA has addressed how workload and competence concerns relate to criminal defense practice. ABA Formal Op. 06-441 (2006) states "If workload prevents a lawyer from providing competent and diligent representation to existing clients, she must not accept new clients. If the clients are being assigned through a court appointment system, the lawyer should request that the court not make any new appointments." Answer A is incorrect—in fact, the exact opposite is true, as further noted by the ABA in Op. 06-441: "The Rules provide no exception for lawyers who represent indigent persons charged with crimes." Answer Bis tricky but is incorrect—even though it may be true that defendants are even worse off with no representation than they are with minimal or incomplete representation (a point of continuing lively debate in the academic literature), lawyers cannot shirk their ethical obligations under the rules because of some larger societal-wide problem, such as the shortage of lawyers available to represent what the ABA calls "virtually unlimited number of indigent criminal defendants." Id. Answer C is also incorrect—even though Model Rule 6.2 creates a general duty to accept judicial appointments, it includes provisions allowing lawyers to decline representation for good cause. Both the official Comment to Rule 6.2 and ABA Formal Op. 06-441 specify that lawyers with an overload of cases have a duty to decline, rather than accept, additional judicial appointments.

A client and her estranged husband have lived separately for several years. The client faces charges for involvement in an armed robbery, and she retains Attorney Graves to represent her. The client's estranged husband learns about the criminal charges and looks up her attorney's information in the local court records. The husband then contacts Attorney Graves and asks to make a payment for the client's representation because he feels guilty for having left her several years before. Attorney Graves accepts payment from the client's estranged husband, without asking the client, and merely tells the client that she does not need to worry about the fees for his representation. Could Attorney Graves be subject to discipline under the conflict of interest rules? (A) Yes, attorneys are required to obtain informed consent from the client before accepting payment from a third party. (B) Yes, but only because the person paying is an estranged spouse whose interests are inherently adverse to those of the client. (C) No, attorneys may accept payments toward client cases from third persons, as long as the third person is a relative of the client. (D) No, attorneys may accept any payments toward client cases, as long as they do not disclose information about the case to the third person.

(A) Yes, attorneys are required to obtain informed consent from the client before accepting payment from a third party. The correct answer is A. Many students would view questions about third-party payors as coming under the ethical rules that cover attorney fees, but in fact these fall under the conflict of interest rules. Rule 1.7 (see Comment 13) and Rule 1.8(f) both address this situation. By far, the most common third-party payor situation is that of insurance companies, who hire lawyers to represent their policyholders—Rules 1.7 and 1.8(f) both contemplate scenarios like the one described here, where an individual wants to pay someone's legal fees, perhaps even anonymously. The estranged spouse or partner problem illustrates the difficulty—the client may feel manipulated by this gesture, and the party paying the lawyer may exercise undue influence over the lawyer's judgment about matters like settlement versus trial. Answer A correctly reflects the rules—an attorney must obtain informed consent from the client before accepting payment from a third party, and the lawyer in this case did not do so. Thus, the lawyer could be subject to discipline under the conflict of interest rules. Answer B incorrectly suggests that the only problem here is that the payor is an estranged spouse, but the rules require informed consent in any third-party payor situation, including an insurer, parents, or employers. Answer C is wrong for two reasons: first, it overlooks the requirement that clients consent to anyone else paying their legal fees (which did not occur in this question), and second, there is no requirement that the third-party payor be a relative—in fact, the most common third-party payor is an insurance company. Finally, Answer D is incorrect for ignoring the requirement of client consent—though it is true that the client would also have to authorize any disclosure of confidential information to the third-party payor, besides giving consent to the payment arrangement itself.

A lawyer agreed to represent Seller, who wished to sell her business to Buyer. The sale has already bogged down in protracted negotiations over purchase price, outstanding liabilities, and certain trade secrets. The lawyer also represents Buyer in unrelated litigation over child support payments after a divorce. Buyer and Seller are not litigating against each other—the lawyer represents each in wholly unrelated matters. Must the lawyer obtain informed consent from each client to undertake representation of Seller in the negotiations over the sale of the business? (A) Yes, directly adverse conflicts can also arise in transactional matters, so a lawyer cannot represent the seller of a business in negotiations with a buyer that the lawyer represents in another matter without the informed consent of each client. (B) Yes, but only if the confidential information the lawyer will learn from representing Seller in the transaction could be relevant to Buyer's child custody dispute, or vice versa. (C) No, because Buyer and Seller cannot be adverse parties if there is no litigation pending between them, nor do they plan to commence litigation against each other in the future. (D) No, directly adverse conflicts cannot arise in purely transactional matters, as both parties have impliedly consented to the representation by agreeing to negotiate over the transaction; a lawyer can undertake transactional representation without the informed consent of each client.

(A) Yes, directly adverse conflicts can also arise in transactional matters, so a lawyer cannot represent the seller of a business in negotiations with a buyer that the lawyer represents in another matter without the informed consent of each client. The correct answer is A. Conflicts of interest arise in transactional matters as well as in litigation. For example, a lawyer representing a seller and a purchaser in the sale of a business or a parcel of real estate would face a dilemma—the client who is the seller would normally want his lawyer to negotiate for the highest possible sale price, while the client who is the buyer would want a lawyer to negotiate for the lowest possible price. A lawyer trying to represent both parties thus has contradictory expectations from each client. Moreover, the lawyer would have a conflict even if she represents only one of the parties in the transaction itself but represents the other party in some unrelated matter—in such a case, questions of loyalty and favoritism would arise. This question presents a very common transactional conflict of interest. Answer A is the correct answer—the lawyer has a conflict of interest here and cannot represent Seller without first obtaining informed consent, in writing, from both clients involved. If both clients are willing to consent to the representation despite the conflict, which is conceivable if the negotiations in the transaction are not overly acrimonious, then the lawyer could proceed. Answer B incorrectly focuses on the problem of the lawyer having confidential (that is, prejudicial) information about both clients. Even though confidentiality concerns are an important part of assessing any potential conflict, there are equally important concerns that the lawyer might favor one client over the other in the negotiations, especially if one client is paying more or generates a lot more business for the lawyer. Moreover, one or both clients may perceive that the lawyer is biased, or being unfair, even if the lawyer is in fact remaining objective and neutral through the representation. Answer C wrongly suggests that conflicts of interest pertain only to litigation, where one client sues another. Transactional matters can also create conflicts of interest, and often lawyers find these conflicts more difficult to identify and address. Answer D makes the same mistake as Answer C but phrases it differently—transactional matters do, in fact, present conflicts of interest. The parties to a transaction have adverse interests, when, for example, one party wants the price to be higher and the other wants it to be lower, or when co-purchasers or partnerships want to divide the asset unevenly.

Attorney Gibbons represents a client before an Administrative Law Judge (ALJ) in a regulatory enforcement matter. The ALJ orders Attorney Gibbons to disclose whether the client had received legal counsel about the regulatory requirements in question before the violation occurred. The client forbids Attorney Gibbons to answer the question. Attorney Gibbons can think of three arguments he might use to persuade the ALJ that the disclosure order is inappropriate: that the ALJ lacks statutory authority to make such orders, that the information is not relevant to the proceeding, and that attorney-client privilege applies. Each of these arguments is weak and unlikely to prevail, but they are not completely frivolous. Should Attorney Gibbons object and try to assert these defenses against disclosure, even though they are unlikely to succeed? (A) Yes, lawyers should assert any nonfrivolous claims before capitulating to a court order that mandates a disclosure over the client's objection (B) Yes, because the information relates only to what the lawyer told the client, not to what the client told the lawyer, so the duty of confidentiality does not apply (C) No, because an Administrative Law Judge is not a court or tribunal for purposes of the exceptions to the confidentiality rules that might permit disclosures in response to a court order (D) Yes, because a lawyer shall not reveal information relating to the representation of a client unless the client gives informed consent

(A) Yes, lawyers should assert any nonfrivolous claims before capitulating to a court order that mandates a disclosure over the client's objection The correct answer is A. Comment 15 to Rule 1.6 adds an important qualification to the specific exception set forth in Model Rule 1.6(b)(6), which authorizes lawyers to disclosure confidential client information when ordered to do so by a tribunal. The qualification is that the lawyer should first try raising any available objections, as long as they are not completely frivolous, before providing the information to the tribunal. Even though the lawyer will ultimately have to obey the order from a court or ALJ, it is worth lodging the objections in case there is any chance that the tribunal will change its mind. In this question, Answer A is the best answer—even weak objections are worth raising, as long as they are not frivolous. Administrative Law Judges almost always have authority to issue subpoenas or compel disclosures, but in rare cases, the appellate courts have found that the ALJ overstepped the statutory boundaries, so the assertion is appropriate, though weak. The claims of irrelevance or privilege could be a factor if the case goes up on appeal, though they are unlikely to change the mind of the ALJ who issued the initial order. Answer B misstates the confidentiality rule—communications by the client to the lawyer are the core of what the confidentiality rule covers (if anything, it is communications by the lawyer that are somewhat less clearly covered). Answer C is incorrect because Administrative Law Judges, government agency hearing officers, and similar adjudicators are covered under the exception in Rule 1.6(b)(6), even though they are not Article III judges. Answer D states the basic rule of attorney confidentiality but ignores the exceptions provided in subsection (b) of the rule.

A lawyer represents Mr. Sharp in several contract dispute cases regarding services for which Sharp was paid but he did not provide. The local district attorney's office indicted Sharp on offenses related to a fraudulent investment scheme he ran. Sharp retained the same lawyer he used for the contract disputes to represent him in his criminal case as well. Sharp confided to the lawyer that he solicited and accepted money from a Ms. Mayfield, a 75-year-old widow, for a financial investment company that did not exist. Sharp explained that his construction business ran into financial troubles and he used this scheme to obtain money to pay his construction company's expenses but that he did not plan to do this again. What may the lawyer do in this situation? (A) He may disclose the information, because it involves the commission of fraud that resulted in substantial injury to the financial interests of another. (B) He may disclose the information, because a lawyer may make any disclosures that relate to anticipated fraud or crime by his client. (C) He may not disclose the information, because the client retained him to represent him on the matter and the details provided are confidential. (D) He may not disclose the information, because disclosure of the financial scheme is not reasonably certain to prevent death or substantial bodily injury.

(C) He may not disclose the information, because the client retained him to represent him on the matter and the details provided are confidential. The correct answer is C. Rule 1.6 contains a basic requirement that lawyers should not disclose confidential information related to the representation of a client. The rule also contains two commonsense qualifications (client consent and implied authorization) and some specific exceptions, but the basic confidentiality requirement applies to most information, most of the time. The facts in this problem may seem complicated at first because there are separate matters involved—the same lawyer represents the client for civil matters and a criminal case—but the rule application here is straightforward. The lawyer cannot disclose the information, as indicated in Answer C (the correct answer). Answer A is incorrect because the "fraud" exception does not apply to situations like this one, where the lawyer is defending the client against charges of fraud previously committed. On the other hand, the client's intentions of committing the same fraud in the future, at least as presented here, are too vague or uncertain to trigger any exception to the confidentiality rule. The problem does not indicate the client intends to use the lawyer's services to perpetrate the fraud, so a lawyer would not be free to make a disclosure that a client would be willing to commit crimes or fraud at some indefinite point in the future. Answer D is wrong because it gives an incorrect reason for the lawyer's duty to keep the information confidential—the lawyer has a basic duty to protect client confidentiality in almost all situations, and basing the answer on a specific exception not being applicable is too narrow a description of the rule.

Maxximas Corporation, a large corporate client, is involved in litigation that has received media attention on major news networks and online news portals. The Maxximas's directors have asked their litigation attorneys to give a private presentation to their outside public relations firm, Spin Doctors, Inc., explaining the company's litigation strategy and risk assessment, because the public relations firm handles press inquiries about the litigation. The litigation attorneys have had no prior contact with Spin Doctors, so they opened their presentation with a reminder that the meeting was confidential and that some of the information shared would be privileged. Later, the opposing party in the litigation learns that this meeting occurred and seeks discovery of the PowerPoint slides the lawyers used in their presentation to Spin Doctors. Are the PowerPoint slides protected by attorney-client privilege? (A) Yes, because the communication was private, between lawyers and an agent of the client at the client's direction, and related to litigation (B) Yes, because the lawyers explained at the beginning of the private meeting that the contents of their presentation would be privileged and confidential (C) No, because the public relations firm is not the client (D) No, because attorney-client privilege would apply only to what was said at the meeting, not to the PowerPoint slides, which could potentially be forwarded to individuals who were not at the private meeting

(C) No, because the public relations firm is not the client The correct answer is C. Attorney-client privilege applies only to communications between the client and the lawyer, although it can apply to agents of the client who are the "functional equivalent" of the client, or outside firms hired by the law firm to assist the lawyers with a case (that is, agents of the lawyers). An outside public relations firm would have to be the functional equivalent of a company employee in order for communications to or from the lawyer to be privileged, which usually is not the case—outside public relations firms operate relatively independently of their clients or customers. A few appellate cases have addressed facts similar to those described in this question, and the courts have held that attorney-client privilege did not apply to such situations (note that some courts apply privilege to PR firms hired by the lawyers, but not ones hired by the client). Thus, Answer C is the correct answer here—the fact that the public relations firm is not the client, and is not arguably the functional equivalent of an employee, is fatal to the claim of attorney-client privilege. Answer A wrongly suggests that the Upjohn factors (named after the leading case in the area) would apply to outside PR firms hired by the client, but that is not the case—the factors listed in Answer A might create privilege for a company employee, but not for an outside company providing services. Answer B is incorrect because merely asserting that communications are privileged does not necessarily mean that the privilege will apply. The communication must meet all the requirements for creating privilege, and must not have been waived by the attorney or client at any point, in order for the protection to apply. Finally, Answer D is based on a false dichotomy between oral and written communications—both can be privileged if the privilege applies at all, but neither the oral statements nor the slides presented at the meeting in this question would be privileged.

An attorney agreed to represent a client in a lawsuit. During their discussions of the case, the litigation attorney mentioned to the client that one of the points in the case involved a novel question of law, for which the lawyer would need to seek advice from another lawyer with more expertise in that area. The lawsuit proceeded, the attorney obtained the advice that he needed, and the case eventually reached a verdict. At the end of the representation, the attorney sent the client a bill that included the attorney's agreed-upon fee, as well as a reasonable fee for three hours of work performed by the expert outside counsel for research and a brief memorandum. The attorney reduced his own fee by the same amount, so that the client's total bill was the same. The client had been unaware that he would have to pay the other lawyer as well but reluctantly agreed and paid the bill. Is the attorney subject to discipline for this additional fee? (A) Yes, because the attorney should not have reduced his own fee below the agreed-upon rate if some of his own work was consequently uncompensated (B) No, because the client's total bill was exactly what he expected, so there was no harm done in the attorney dividing the fee with another lawyer (C) Yes, a division of a fee between lawyers who are not in the same firm may be made only if the client agrees to the arrangement, including the share each lawyer will receive, and the agreement is confirmed in writing (D) No, because the client agreed to pay the bill in the end instead of disputing it, which would have triggered an inquiry from the state disciplinary authorities

(C) Yes, a division of a fee between lawyers who are not in the same firm may be made only if the client agrees to the arrangement, including the share each lawyer will receive, and the agreement is confirmed in writing The correct answer is C. This question tests the specific provisions about fee-sharing set forth in Model Rule 1.5(e)(2). It is fairly common for other lawyers to get involved with a case at the request of the lawyer whom the client originally hired—usually because the other lawyers have particular expertise or experience that helps with an aspect of the case. Sometimes, these lawyers are from other law firms, and they end up sharing the fees in the case. The Model Rules require that clients consent to this arrangement ahead of time—including the sharing of the fees between whatever lawyers get involved in the case. The main concern is to avoid the client having to pay unexpected additional fees or bills for other lawyers (making the representation far more expensive than the client expected). In this question, that concern was not a problem because the original lawyer reduced his own fee so that the total bill to the client remained the same. Nevertheless, the Model Rules also require that the client know which lawyers are providing representation (the client should have a right to refuse involvement of other lawyers), and that the client know and agree to the proportion of the fee going to the other lawyers. These requirements protect against conflicts of interest on the part of the other lawyers who come into a case at later stages and protect the client from having nearly all the legal work done by someone other than the lawyer the client hired. Answer C correctly reflects the requirements of Rule 1.5(e). The attorney here did not explain to the client that some of the fees paid by the client would go to another lawyer. A lawyer must obtain a client's consent to this arrangement up front. Answer A is incorrect because lawyers may indeed reduce their own fees or provide some of their legal work without compensation (in pro bono cases, all the legal work is uncompensated). Answer B is incorrect because the Model Rules require the client's consent to the fee sharing (including the proportion each lawyer will receive) regardless of the total bill being the same. Answer D is incorrect because a client does not have to dispute the charges in order for a lawyer to be subject to discipline over fees. Even if a client accepts the bill and pays it, the lawyer could still be subject to discipline for failing to comply with the rules about fees.

An attorney represents Conglomerate Corporation in a civil suit. During the representation, the attorney began a sexual relationship with a receptionist at Conglomerate. The receptionist's only duties are to answer the phone, route calls, take messages, and prepare outgoing mailings; she has no authority in decision-making at the company. In fact, the receptionist's only workplace communication with the attorney is when the attorney calls Conglomerate and the receptionist routes his call to the person with whom the attorney wishes to speak. Has the attorney violated the ethical rules regarding conflicts of interest by carrying on a relationship with the receptionist? (A) Yes, an attorney that represents an organization may not have a sexual relationship with anyone they speak with at the company on a regular basis, including administrative personnel. (B) Yes, attorneys shall not have sexual relationships with their clients, and when an organization is the client, the attorney shall not have sexual relationships with employees of the organization. (C) No, attorneys are not restricted from having relationships with employees of an organization the attorney represents because the employees are not the client. (D) No, attorneys are only restricted from having relationships with members of an organization who are directly or are regularly involved with the attorney concerning the organization's legal matters.

(D) No, attorneys are only restricted from having relationships with members of an organization who are directly or are regularly involved with the attorney concerning the organization's legal matters. The correct answer is D. The Model Rules of Professional Conduct prohibit sexual relationships between lawyers and clients in two places—in the official Comment to Model Rule 1.7 (the general conflict of interest rule), and in a newer addition to the MRPC, Rule 1.8(j). It is important to note that the prohibition does not apply to sexual relationships that predated the legal representation, but that is not applicable to this question—the lawyer here commenced the relationship once the representation of the corporate client was underway. Comment 19 to Model Rule 1.8(j) adds another caveat, applicable only to corporate clients, that is relevant to this question: "When the client is an organization, paragraph (j) of this Rule prohibits a lawyer ... from having a sexual relationship with a constituent of the organization who supervises, directs, or regularly consults with that lawyer concerning the organization's legal matters." If a company executive or someone in the corporation's in-house legal department hired the lawyer and directs the lawyer's representation on behalf of the entity, the prohibition on sexual relations with clients applies. Note that for purposes of other ethical rules (including some provisions of the conflict of interest rules), the "client" is not a particular individual, but rather the corporate entity itself, acting through its legal representatives (including officers and directors). In this question, Answer D is the correct answer—the lawyer has not violated the ethical rules for lawyers. The receptionist is not the person who "supervises, directs, or regularly consults with the lawyer concerning the organization's legal matters"—the receptionist has limited duties and no decision-making responsibility at the company. Answer A incorrectly extends the prohibition of Rule 1.8(j) to all employees who speak regularly with the lawyer, but Comment 19 explicitly narrows the rule more than this. Answer B similarly states the rule in overly broad terms and is therefore incorrect—regular employees of the corporation do not count as the "client" for purposes of Rule 1.8(j) or the sex-with-client prohibition in Rule 1.7. Answer C errs in the other direction, suggesting that the rule does not apply to corporate clients at all—in fact, the individual or individuals at the corporation who directly supervise the lawyer's work, or consult with the lawyer on legal matters on behalf of the company, would trigger application of the rule.

Attorney Rainier wrote a brief for the court arguing against the opposing party's motion for summary judgment. In his brief, Attorney Rainier never mentioned specific cases that were controlling authority in that jurisdiction and that were adverse to Rainier's position, because the opposing party's brief already discussed all adverse controlling authority. Instead, Attorney Rainier focused on a few outlier cases that supported his side and dismissively referred to all the contrary authority, discussed at length in the opposing party's brief, as "easily distinguishable from the present case on factual grounds." No objective reader would have thought that the brief presented a fair, even-handed exposition of the law relevant to the case, and no objective reader would have found Attorney Rainier's brief very convincing. Has Attorney Rainier violated the duty of candor to the court by writing such a one-sided brief? (A) No, because a lawyer in an adversary proceeding is not required to present an impartial exposition of the law (B) No, because no objective reader would have found the brief convincing, so there is no chance that the brief will mislead the tribunal about the controlling law in that jurisdiction (C) Yes, because the lawyer must not allow the tribunal to be misled by false statements of law or fact or evidence that the lawyer knows to be false (D) Yes, because there are special duties of lawyers as officers of the court to avoid conduct that undermines the integrity of the adjudicative process

(A) No, because a lawyer in an adversary proceeding is not required to present an impartial exposition of the law The correct answer is A. Comment 2 to Model Rule 3.3 qualifies the duty of candor to the court by noting that "a lawyer in an adversary proceeding is not required to present an impartial exposition of the law." Even though it is impermissible for the lawyer to make arguments that have no basis in the law (or to say things about the law that the lawyer knows to be false), the lawyer is an advocate in the litigation context, and it is appropriate for an advocate to argue strongly for one side. Of course, an even-handed tone is often more convincing to the reader, but it does not violate the rules of ethics for a lawyer to present a one-sided, biased-sounding argument. Answer A is the correct answer, echoing the verbiage used in Comment 2. A lawyer cannot misrepresent what the law says, but a brief also does not have to provide an impartial discussion. Answer B is incorrect because writing an unconvincing brief is not an ethical violation, even if it constitutes mediocre legal work. Answer Crestates the general prohibition against misleading a tribunal (also echoing language in Comment 2), but it does not fit the facts in this question. The lawyer did not lie about the cases, or fail to disclose controlling precedent that the other side had missed—he merely offered a weak argument to rebut the opposing party's motion. Answer D is wrong for a similar reason—even though the lawyer does indeed have a duty to avoid undermining the integrity of the proceedings, presenting a one-sided argument as an advocate does not run afoul of this duty.

Attorney Roberts represents Client in a civil matter. Client's case went to trial and Client lost. Client wished to appeal the matter. Attorney Roberts did not file an appeal because there was no agreement that he would handle the appeal. The period in which Client could file an appeal expired and no appeal was filed. Was Attorney Roberts's conduct proper? (A) No, because a lawyer must discuss the possibility of an appeal prior to relinquishing responsibility for a client's case, unless there was a prior agreement about whether the lawyer would handle the appeal process (B) No, because a lawyer shall complete an entire case for a client, including the appeal process, unless the parties have agreed in writing that the attorney's employment terminates after trial (C) Yes, because an attorney is not required to continue working on a case for a client after trial unless the attorney and the client specifically agreed that the attorney would continue to be employed as the attorney for the appeal process (D) Yes, because the same lawyer cannot represent a client at trial and on appeal, and the client should have been aware of this rule

(A) No, because a lawyer must discuss the possibility of an appeal prior to relinquishing responsibility for a client's case, unless there was a prior agreement about whether the lawyer would handle the appeal process The correct answer is A. A common source of misunderstandings between clients and lawyers is whether the lawyer will handle the appeal of a case after a trial—that is, whether the losing side's lawyer will appeal the case and/or the winning side's lawyer will defend the favorable verdict on appeal if the losing side appeals. Many clients simply assume their trial lawyer will carry on with the representation through the appeal stage, but many trial lawyers do not include appellate work in their practice. The Model Rules thus place the burden on the lawyer to communicate with the client about the appeal before ceasing work on the case. Answer Acorrectly states the guideline set forth in Comment 4 to Model Rule 1.3—unless the lawyer and client agreed on this point previously, a lawyer must have a discussion with the client about who is handling the appeal before relinquishing responsibility for the matter. Answer B is incorrect because trial lawyers certainly do not have to handle appeals—many lawyers limit the scope of their representation to a particular part of the case. Answer Cis wrong because it omits the lawyer's duty to have a discussion with the client about the appeal to avoid situations where the client assumes the lawyer is working on the appeal and the lawyer thinks the representation has ended. Answer D states wrongly that trial lawyers cannot handle appeals for their clients—there is no such prohibition, and some lawyers do indeed handle the entire case for a client.

During a criminal trial over an armed robbery, the state's star witness died unexpectedly, and the prosecutor's case quickly unraveled. The jury eventually acquitted the defendant, even though the defendant had initially confessed to the armed robbery and later recanted. A few weeks later, the same prosecutor obtained some uncorroborated evidence that the defendant also committed check fraud. Even though he did not have enough evidence to constitute probable cause, the prosecutor brought charges for check fraud in hopes that the defendant might accept a plea agreement with some jail time rather than undergo the ordeal of another trial. If so, the prosecutor thought, a dangerous armed robber would be off the streets, regardless of the reason. Were the prosecutor's actions proper, according to the ethical rules? (A) No, because the Model Rules prohibit prosecuting a charge when the prosecutor knows there is no probable cause. (B) No, because the prosecutor is trying to use a lesser charge to incarcerate a dangerous criminal, which is tantamount to double jeopardy. (C) Yes, because the high burden of proof in criminal cases protects defendants against unfounded charges. (D) Yes, because the prosecutor has good intentions, as he wants to protect the public from a dangerous criminal, and it was not the prosecutor's fault that he lost the previous case.

(A) No, because the Model Rules prohibit prosecuting a charge when the prosecutor knows there is no probable cause. Answer A is the best answer. ABA Model Rule 3.8(a) says "The prosecutor in a criminal case shall . . . refrain from prosecuting a charge that the prosecutor knows is not supported by probable cause." This ethical rule applies in addition to the rules of criminal procedure that require probable cause—in other words, not only should the defendant be able to seek a dismissal of the charges, but the prosecutor could be subject to discipline by the state bar disciplinary authority, though this seldom occurs. Answer B is wrong because it misstates the nature of double jeopardy, and even if double jeopardy applied, it is not clear that this would constitute an ethical violation for the prosecutor under Rule 3.8. Answer C admittedly states the correct idea about burdens of proof protecting the defendant, but that does not mean the prosecutor could not be subject to discipline. Even if a defendant could obtain a dismissal of the charges, individuals should not have to incur the expense and inconvenience of having to fend off charges that the prosecutor knows he cannot prove. Answer D is incorrect because the prosecutor's good intentions, in the sense of protecting the public, do not absolve the prosecutor of ethical duties to avoid bringing unfounded charges against a criminal, even if the prosecutor is certain of the individual's guilt but lacks admissible proof.

Two former college classmates had formed a business partnership, but later the partnership was dissolved, and litigation ensued. One of the partners hired a lawyer, who also went to college with him, to represent him in the litigation. The former classmates' ten-year college reunion was two months away, and the attorney and the opposing party (the other partner) were both on the Alumni Association's reunion committee. The attorney called the opposing party to discuss arrangements for the upcoming reunion banquet and did not mention the pending litigation at all. Opposing counsel overheard his client talking to the attorney and reported it to the judge, accusing the attorney of engaging in an ex parte communication with the opposing party, in violation of violating the ethical rules. Earlier in the proceedings, the judge had sternly admonished both lawyers against contacting the judge or the opposing party about the case without the other lawyer present. Is the attorney subject to discipline for talking to the opposing party, who was represented by counsel, without opposing counsel present? (A) The lawyer is not subject to sanctions or discipline because the communication was about a matter outside the representation. (B) The lawyer is subject to discipline for violating the Model Rules' no-contact rule, but not to sanctions for violating the judge's order, as the conversation was not about the litigation. (C) The lawyer is subject to discipline for violating the ethical rules AND is subject to sanctions for violating the judge's order. (D) The lawyer is subject to sanctions for violating the court's order but not subject to discipline for violating the Model Rules, as the conversation was about a matter outside the representation.

(A) The lawyer is not subject to sanctions or discipline because the communication was about a matter outside the representation. The correct answer is A. Model Rule 4.2 prohibits lawyers from talking (or writing) about the legal matter at hand with anyone whom "the lawyer knows to be represented by another lawyer in the matter, unless the lawyer has the consent of the other lawyer or is authorized to do so by law or a court order." In this question, the attorney would have violated the rule if he had discussed the pending partnership dissolution proceedings with the opposing party (the other partner) at all without the opposing lawyer present, but that was not the subject of the communication. Answer A is therefore the correct answer to this question. Answer B, in contrast, is wrong because the lawyer did not violate the no-contact rules (which pertain only to communication about the subject of the representation), although Answer B may be partly correct that the lawyer probably did not violate the judge's instructions, either. Answer C is incorrect for the same reason—the subject of the communication was not related to the representation (that is, the partnership dissolution), so it was permissible. Answer D is partly right and partly wrong—the lawyer does not appear to have violated the judge's orders or the Model Rules.

Harvey was a licensed attorney in State A, but his law practice was financially insolvent. He decided to move to another part of the country and start over, so he bought a ticket to State B. In State B the attorney ran advertisements in the local news offering to represent clients on small matters for a deeply discounted rate. Before long, Harvey was representing clients in his new location. All the matters he handled were simple and within his level of competency, and his clients were consistently satisfied with his representation. Eventually, the unauthorized practice of law authorities of State B brought an enforcement action against Harvey for practicing law without a license. Is Harvey subject to discipline for practicing without a license if his license in State A is still valid? (A) Yes, because he has established an ongoing law practice in State B and represents clients there, even though he does not have a license to practice in that jurisdiction. (B) Yes, because it is not possible that a lawyer from State A could competently handle a client matter that pertained to State B law. (C) No, because he still has a valid license in another state, so he is not practicing without a license. (D) No, because he is providing competent representation to his clients, and simple legal matters may not require a comprehensive knowledge of the specific laws of that state.

(A) Yes, because he has established an ongoing law practice in State B and represents clients there, even though he does not have a license to practice in that jurisdiction. Answer A is correct. ABA Model Rule 5.5(b) prohibits practicing law without a license in that state: "A lawyer who is not admitted to practice in this jurisdiction shall not . . . establish an office or other systematic and continuous presence in this jurisdiction for the practice of law." Harvey has been advertising for clients and representing clients in a state where he does not have a license to practice, and this violates Rule 5.5. Note that Rule 5.5 contains exceptions for incidental representation by out-of-state lawyers, such as pro hac vice appearances before tribunals, but none of these exceptions apply here. Answer B is incorrect for two reasons. First, most experienced lawyers could, in fact, handle simply legal matters competently even in other jurisdictions, because of similarities between the laws of each state, and because state variations in the legal rules on simple matters would be easy for a lawyer to discover. Answer C is wrong because the ethical rules require that a lawyer have a license to practice in any state where the lawyer has an ongoing and systematic presence for legal representation of clients—it is not enough to have a license in another state or jurisdiction. Finally, Answer D is incorrect because the lawyer's competence or effectiveness in providing representation is not the issue—state-by-state licensing requirements apply regardless of the lawyer's effectively, knowledge, or skill.

Following an acrimonious divorce, a client hired an attorney to represent her in litigation over the custody of her children. The client was concerned about her former best friend, in whom she had confided about her struggles with substance abuse and mental illness. The friendship had ended because of an intense argument before the client married and had children. In fact, the client had been free from substance abuse since she married and was now managing her mental health issues very well. The attorney located the former friend, explained that the client was fighting for custody of her children, and that he expected the ex-husband's lawyer would call her to testify about the client's former troubles at the hearing. The attorney pleaded with her to show consideration for the years of good friendship with the client and to refuse to betray her former friend's confidence. The former friend felt deeply moved by this entreaty and agreed to stay out of the litigation. Were the attorney's actions proper? (A) No, because the lawyer has a duty to think about the best interests of the children in this case, rather that his client's convenience or feelings (B) No, because a lawyer may not request a person other than a client to refrain from voluntarily giving relevant information to another party, except in circumstances that do not apply here (C) Yes, as long as the lawyer reasonably believes that the friend's interests will not be adversely affected by refraining from giving such information (D) Yes, because a lawyer may request that someone refrain from voluntarily giving relevant information to another party

(B) No, because a lawyer may not request a person other than a client to refrain from voluntarily giving relevant information to another party, except in circumstances that do not apply here The correct answer is B. Model Rule 3.4(f) prohibits lawyers from asking anyone, other than the client, to refrain from testifying at trial or voluntarily giving information to the other party, with narrow exceptions for family members, employees or other agents of the client. A lawyer's attempt to remove a valuable witness through persuasion or inducement is analogous to tampering with or destroying physical evidence. Answer B is correct—the lawyer violated the rules by pleading with the client's former friend not to say anything about her dark past. Of course, the friend might decide on her own not to disclose anything, but the lawyer cannot intervene in this manner. Answer A is incorrect, first, because the lawyer's duty is to the client, not the interests of the children, and second, because the rule is not based on the lawyer's impression of the best interests of the parties—it is impermissible for the lawyer to ask a friend of the client to refrain from testifying, regardless of the circumstances. Answer C is wrong for a similar reason, though it echoes verbiage that qualifies the exception for clients' family members or employees (the lawyer cannot ask them to refrain from testifying if doing so would run counter to that individual's own interests). The lawyer may not ask the friend not to disclose information even if it would benefit the friend to do what the lawyer asks. Answer D is incorrect because the rule says the exact opposite—Rule 3.4 prohibits lawyers form requesting that individuals withhold information from the court or the other party.

Attorney Cole worked at Big Firm, and he exclusively handled litigation for Conglomerate Corporation, one of Big Firm's most important clients. Previously, he had worked for a year at the state Office of the Attorney General. Conglomerate Corporation had no litigation with the state government, so Big Firm made no effort to screen Attorney Cole from any cases, though it would conduct customary conflict checks. In one case, Attorney Cole defended Conglomerate Corporation in a personal injury lawsuit over an accident with one of its delivery truck drivers. The plaintiff's case depended on testimony from a certain eyewitness of the accident, who had already submitted an affidavit saying that Conglomerate's truck had caused the accident. Attorney Cole contacted a lawyer representing the witness and told him that he had documentary evidence that the witness' previous three employers had fired her for fraudulent recordkeeping, lying to supervisors to cover up mistakes, or lying to customers. This was just bluffing—no such documents existed. Attorney Cole believes this should constitute harmless "posturing," which is normally appropriate in negotiations. Under the ethical rules, was it impermissible for Attorney Cole to make this false statement to the other party? (A) Yes, because Attorney Cole's prior work history at the Office of the Attorney General created an impression of credibility or trustworthiness that would induce other parties to rely on the statements that would otherwise have seemed like mere posturing or puffing. (B) Yes, because Attorney Cole represented that he had documentary evidence when no such documents existed. (C) No, because whether a specific statement should count as one of fact can depend on the circumstances. (D) No, a party in a negotiation also might exaggerate or emphasize the strengths, and minimize or deemphasize the weaknesses, of its factual or legal position.

(B) Yes, because Attorney Cole represented that he had documentary evidence when no such documents existed. Answer B is the best answer. ABA Model Rule 4.1 provides, "In the course of representing a client a lawyer shall not knowingly make a false statement of material fact or law to a third person. . . ." Even though Comment 2 to Rule 4.1 allows for some "posturing," the misstatements here fall outside that exception. Similarly, ABA Formal Op. 06-439 specifically mentions the type of misrepresentation in this Question as one example of a violation that would not constitute "posturing." Answer A is incorrect because the lawyer's personal credibility is not a relevant consideration in determining whether misrepresentations are unethical—Attorney Cole's misrepresentation would violate the rules even if he had no prior work history or a tarnished reputation. Answer C is true but inapplicable to this Question—the statement comes directly from Comment 2 to Rule 4.1, but the remainder of the Comment qualifies and narrows this position. Answer D incorrectly overstates the prohibition—lawyers may, in fact, exaggerate or emphasize the strengths of their position during negotiations, but may not make a statement that is objectively false and misleading, as occurred here.

Attorney Adkins had an exceptionally successful law practice, and she was able to charge her clients higher fees than other lawyers in the area. In fact, she had become one of the wealthiest lawyers in her state. One day, a judge called Attorney Adkins and requested that she accept a case and represent an indigent defendant whose case was before his court. The judge explained that he wanted to appoint Attorney Adkins to represent this defendant because his case was unusually complicated, and Attorney Adkins had a stellar reputation in the legal community for her skills and abilities. Attorney Adkins thanked the judge for the compliment but explained that she would have to decline the appointment because the court paid fees for indigent defense that were far lower than what she would normally charge her clients. The judge was indignant and reported Attorney Adkins to the state disciplinary authority. Could Attorney Adkins be subject to discipline for declining the case, given these facts? (A) Yes, because the Model Rules require lawyers to accept appointments from a tribunal regardless of the financial burden imposed or how repugnant the client or cause would be to the lawyer. (B) Yes, because the Model Rules require lawyers to accept appointments from a tribunal unless the lawyer has good cause for declining the representation, which Attorney Adkins does not have. (C) No, mandating that a lawyer accept appointments from a tribunal, even when the lawyer does not want to do so, constitutes an unconstitutional taking of the lawyer's legal services. (D) No, the fact that Attorney Adkins would have to work for a much lower rate than she usually charges her clients is an appropriate reason to decline an appointment from a tribunal.

(B) Yes, because the Model Rules require lawyers to accept appointments from a tribunal unless the lawyer has good cause for declining the representation, which Attorney Adkins does not have. Answer B is the best answer. Model Rule 6.2 states, "A lawyer shall not seek to avoid appointment by a tribunal to represent a person except for good cause," and then it provides three examples of "good cause," none of which are present in this Question. Comment 1 to Rule 6.2 explains, "All lawyers have a responsibility to assist in providing pro bono publico service. . . . A lawyer may also be subject to appointment by a court to serve unpopular clients or persons unable to afford legal services." In other words, it is an ethical violation for a lawyer to refuse to accept a court appoint, unless the lawyer can show good cause for doing so, and the burden is on the lawyer to demonstrate good cause. Answer A is overbroad and therefore not the best answer. Model Rule 6.2 provides three exceptions or examples of "good case" for declining a court appointment: a) the representation would violate other ethical rules (e.g., the lawyer would have a conflict of interest), b) it would impose an "unreasonable financial burden" on the lawyer, or c) "the client or the cause is so repugnant to the lawyer as to be likely to impair the client-lawyer relationship or the lawyer's ability to represent the client." Answer A incorrectly ignores these exceptions. On the other hand, Answer C is incorrect because the "taking" defense applies, if at all, only when courts compel a lawyer to serve without any compensation. See Spees v. Ky. Legal Aid, 274 S.W.3d 447 (Ky. 2009) ("requiring counsel to represent an indigent parent pro bono in a termination case amounts to an unconstitutional taking"); State ex rel. Mo. Pub. Defender Comm'n v. Pratte, 298 S.W.3d 870 (Mo. 2009) (en banc) (uncompensated court appointments can constitute a deprivation of property without due process); Lavalee v. Justices in Hampden Superior Court, 812 N.E.2d 895 (Mass. 2004) (government must help compensate lawyers for indigent defense); DeLisio v. Alaska Supreme Court, 740 P.2d 437 (Alaska 1987) (it is a taking to require a private lawyer to represent indigent criminal defendant without pay); but see Scheehle v. Justices of Supreme Court, 508 F.3d 887 (9th Cir. 2007) (rule requiring lawyers to be subject to appointment as arbitrators for up to two days per year with minimal compensation was not a regulatory taking). Answer D is wrong because Attorney Adkins is not facing an "unreasonable financial burden" by accepting one appointment that pays less than her other clients in her busy, successful law practice.

An insurance company hired Attorney Applegate to represent one of its policyholders. The insurance company requires periodic updates and detailed billing statements about the matter from the attorney as part of its agreement to provide representation for its insured. In addition, the insurance company requires Attorney Applegate to submit detailed billing statements to a third-party auditor, designated and paid by the insurance company. The client, who is the policyholder, consented beforehand to the disclosures to the insurance company but is unaware of the arrangement with the third-party auditor. The third-party auditor receives no information except the name and policy number of the client and the time spent by Attorney Applegate on various tasks necessary for the representation. Could Attorney Applegate be subject to discipline for disclosing confidential information? (A) Yes, because both the disclosures to the insurance company and the third-party auditor violate the lawyer's duty of confidentiality toward the client (B) Yes, even though the policyholder impliedly authorized the lawyer to provide updates and billing statements to the insurance company, submitting the bills to the third-party auditor constitutes an unauthorized disclosure of confidential information (C) No, because the lawyer has implied authorization from the fact that the insurance company hired him to represent the policyholder to make the disclosures. (D) No, because billing information is not privileged and its disclosure could not prejudice the client

(B) Yes, even though the policyholder impliedly authorized the lawyer to provide updates and billing statements to the insurance company, submitting the bills to the third-party auditor constitutes an unauthorized disclosure of confidential information The correct answer is B. ABA Formal Op. 01-421 addressed a specific confidentiality problem that has become widespread in the American legal system—insurance companies hire lawyers to represent their policyholders (extraordinarily common) and then use a third-party billing company to process the lawyer's bills and fee payments. Under the ethical rule of confidentiality, even the information on a lawyer's timesheet would be confidential, because it reveals what activities the lawyer undertook on the client's behalf, the nature of any research and investigation conducted (at least in general terms), and so on. The lawyer would therefore need the client's consent to disclose this information to anyone else—including the insurer's billing company. Answer B correctly represents the lawyer's duty. Even if a client has authorized the lawyer to submit timesheets for billing purposes to the insurance company (as is customary), the client must also consent specifically to the disclosure to the billing company as well. The Multistate Professional Responsibility Exam has included questions in recent years on this particular ethics opinion by the ABA. Answer A is incorrect because the client consented to the disclosures to the insurance company, according to the facts in this question, and client consent always makes it permissible for the lawyer to proceed with the disclosure. The client did not consent to the disclosure to the billing company but did consent to the insurance company receiving the lawyer's bills. Answer C is incorrect because the disclosure to the billing company is not implied by the client's agreement to have the lawyer handle the underlying legal matter—many clients would not be aware that the billing company existed or that the disclosures would be occurring. Finally, Answer D wrongly suggests that billing information is not covered by the duty of confidentiality—the ABA has expressly included lawyer billing information under the confidentiality protection.

A subcontractor on a highway construction project negligently damaged the general contractor's equipment and simultaneously inflicted property damage on a state building storing the equipment. The relevant state office, along with the general contractor, hired Attorney Andrews to represent them in a lawsuit against the subcontractor. The state client and the general contractor, who is a private party, each provide written informed consent to potential conflicts of interest in the form of a waiver. Is it proper for Attorney Andrews to represent both the government and a private party at the same time? (A) Yes, the fact that the state represents the public interest cancels out any potential conflict of interest on the part of the private party and makes the Rules of Professional Conduct inapplicable. (B) Yes, the conflict of interest rules do not strictly prohibit a lawyer from jointly representing a private party and a government agency. (C) No, unless the subcontractor also provides written informed consent to the attorney's joint representation of the two parties that were once contractual partners with the subcontractor. (D) No, because the general contractor's interests are purely financial, while the state's interests involve a balancing of various competing interests of the general public.

(B) Yes, the conflict of interest rules do not strictly prohibit a lawyer from jointly representing a private party and a government agency. The correct answer is B. Comment 9 to Model Rule 1.11 includes a passing remark about joint representation involving a government entity as one of the clients. This question gives an example of the circumstances in which this could occur. The conflicts of interest rules do not prohibit joint representation of a private party and a government agency, such as a corporation and a municipality, or a private individual and a subdivision of the state. Answer B is correct because there is no absolute rule prohibiting the joint representation described here. Of course, it is possible that lawyers could have other conflicts of interest that would preclude representing a particular government entity in a matter, such as the lawyer's prior representation of the opposing party in the case. Answer A wrongly suggests that the Rules of Professional Conduct are inapplicable when a lawyer represents a government entity. The MRPC state in several places that the Rules apply to government lawyers as well. Note that government lawyers are usually subject to additional statutory rules besides the ethical rules that apply to all attorneys in a state. Answer C is incorrect because it is not clear that there is any conflict of interest here that would require the subcontractor's consent. Answer D brings up the essential difference between the interests of the government and private sector clients, but the Model Rules take the position that this difference in interests does not present an inherent or automatic conflict of interest between the two when they are joint clients of the same attorney.

Attorney Talisman is a partner at a mid-size law firm. The firm's computer network uses software that inserts the firm's logo and letterhead into every email sent from the firm's email accounts, as well as a legal disclaimer at the end of every email that reads, "NOTICE: This email may contain PRIVILEGED and CONFIDENTIAL information and is intended only for the use of the specific individual(s) to which it is addressed. If you are not an intended recipient, you must not review, copy or show the message and any attachments to anyone. Please reply to this email and highlight the mistaken transmission to the sender, and then immediately delete the message." Attorney Talisman believes that every email sent by anyone at the firm to anyone outside the firm would be protected against disclosure under the doctrine of attorney-client privilege because each email automatically includes this disclaimer under the sender's signature line. Is Attorney Talisman correct? (A) Yes, because the disclaimer informs anyone who reads the email that it is intended to be a private communication between the lawyers at the firm and individuals seeking legal assistance (B) Yes, because the disclaimer asserts the privilege explicitly, but any emails from a law firm would automatically trigger the attorney-client privilege, even without such a disclaimer (C) No, because blanket privilege inscriptions on law firm correspondence do not guarantee that privilege will apply to the contents of the email, because emails sent to non-clients (or copying non-clients as additional recipients) would not be privileged (D) No, because the disclaimer appears automatically in every email, so the sender might not have had a subjective intent for the communication to be confidential

(C) No, because blanket privilege inscriptions on law firm correspondence do not guarantee that privilege will apply to the contents of the email, because emails sent to non-clients (or copying non-clients as additional recipients) would not be privileged The correct answer is C. Attorney-client privilege applies only to confidential communications between a lawyer and client for the purposes of obtaining legal advice or representation, and both clients and lawyers often waive privilege inadvertently by disclosing the information to other individuals. Blanket privilege disclaimers may help remind recipients not to forward the email to others, or remind the attorney sending the message to reflect on the contents of the email before sending it, but such auto-inserted assertions of privilege are unlikely to make a difference in determining whether any individual email truly comes under the protection of privilege. Answer C correctly reflects the privilege rules—boilerplate disclaimers are not in themselves effective in triggering the attorney-client privilege. Answer A is wrong because it focuses on only one component of attorney-client privilege—that the intended confidentiality of the communication is intended to anyone who reads it—and ignores other required elements for creating privilege, such as the client-lawyer relationship, the purpose of the actual contents (obtaining or providing legal advice), and so on. Answer B incorrectly suggests that all communications by a lawyer are privileged, but this is not the case. Law firms routinely send emails to other lawyers, outside vendors of office supplies or photocopying services, marketers, lenders, prospective employees, and so on. Only communications to or from a client would be privileged, not these other emails. In fact, lawyers often inadvertently waive privilege by sending emails, as when they copy (cc:) non-clients on emails they send, or forward client emails to others. On the other hand, Answer D is incorrect because it offers an overly narrow reason for why the emails are not automatically privileged. The lack of specific intent to create a confidential communication is certainly a factor in determining whether privilege applies, but automatic disclaimers can be ineffective even when the sender wanted the communication to be private, as when an attorney includes non-clients among the recipients of an otherwise confidential email.

Attorney worked as in-house counsel for a petroleum refinery. One day, after weeks of stalemate in a round of collective bargaining, the refinery workers decided to go on strike to demand higher wages and more vacation time. The workers abandoned their workstations and picketed on the sidewalk in front of the refinery. Late in the evening, some of the picketers moved their protests onto the refinery compound, including some hazardous areas. The usual safety personnel were also on strike, so there was a substantial risk of an explosion at the refinery if the protestors engaged in vandalism. There was also a risk of injuries to the protestors. At midnight, the refinery's manager called Attorney to obtain guidance on hiring a private security force and having all the picketers arrested and removed from the compound. Although Attorney initially suggested that they simply remove the protestors from the hazardous areas, the manager believed that the protesters on the sidewalk would simply migrate over to the same areas to fill the gaps from those removed and then pose an ongoing safety risk. Attorney knew nothing about protester rights or the rights of striking workers. Attorney tried to call one or two lawyers he knew who might know the answer, but nobody answered his calls given the late hour. Without a basis for forming an opinion related to labor laws and dealing with strikers, Attorney advised the manager that it would be fine for the private security personnel to remove the picketers by force. Unfortunately, during an ensuing scuffle with the private security personnel, several workers and security officers were seriously injured. It turned out that Attorney's advice was incorrect given the special circumstances surrounding the collective bargaining. The refinery was subject to substantial liability both to the injured individuals and possible fines imposed by the federal labor board. Is Attorney subject to discipline? (A) No, because the management made the final decision and should bear the full responsibility for it (B) Yes, because a lawyer should provide competent representation to a client, and competent representation requires the legal knowledge, skill, thoroughness, and preparation reasonably necessary for the representation (C) No, because in an emergency a lawyer may give advice or assistance in a matter in which the lawyer does not have the skill ordinarily required, where referral to or consultation or association with another lawyer would be impractical (D) Yes, unless the manager signed a waiver releasing Attorney from responsibility for the advice he rendered on emergency basis

(C) No, because in an emergency a lawyer may give advice or assistance in a matter in which the lawyer does not have the skill ordinarily required, where referral to or consultation or association with another lawyer would be impractical The correct answer is C. Generally, an attorney must refrain from guesswork when advising clients who seek assistance in a field of law that is unfamiliar to the lawyer. However, Comment 3 to Model Rule 1.1 anticipates emergency situations in which the lawyer may give some guidance to a client immediately. For example, when a transactional lawyer's client calls from jail after an arrest (even if the lawyer does not practice criminal law), the lawyer might encourage the client to invoke his right to remain silent and wait for a criminal defense lawyer to be present before answering any questions. This question presents another emergency scenario where a corporate client must deal immediately with a group of protestors on their property. Answer C correctly reflects the guidance set forth in Comment 3 to Rule 1.1. Given the urgency of the situation and the impossibility of the lawyer getting prior advice from another lawyer, it was appropriate for Attorney to make an educated guess. Note, however, that even though Attorney might evade disciplinary action (the question here), it is unclear whether he would escape malpractice liability if the client later brought suit against him—compliance with the ethical rules does not necessarily determine whether a lawyer could be liable for malpractice. Although U.S. jurisdictions use different approaches to the relevance of disciplinary rules in malpractice cases, the rules would likely be considered in consideration of the standard of care. A common approach is for an expert witness to refer to disciplinary rules in opining on the applicable standard of care. Answer A is incorrect because the fact that a client makes the ultimate decision does not discharge the lawyer's duty to render competent advice. Although Answer B correctly states the basic competence rule, it is incorrect because it leaves out the important "emergency exception" that applies to this case. Finally, Answer D is incorrect because the Model Rule 1.8(h)(1) prohibits lawyers from prospectively limiting their liability for malpractice unless the client is independently represented in making the agreement.

Attorney is handling a case for Client, and Client instructs Attorney not to share any of the case details or any of Client's personal details or information with other attorneys in the firm. While preparing for a hearing, Attorney consults with another attorney in his firm about the case. Attorney tells the other attorney that Client would prefer that other attorneys in the firm not be involved in the case. Attorney uses the information and guidance provided by the other attorney to achieve a successful outcome for the client at the hearing. Are Attorney's actions proper? (A) Yes, because lawyers are authorized to make disclosures when reasonably necessary or when it is in the client's best interest, even if the client instructs the attorney otherwise (B) Yes, because a lawyer may discuss with other members of the firm at which he is employed any issues regarding a case that is being handled by that firm (C) No, because lawyers shall not discuss a client's case with other lawyers of the firm handling the client's case if instructed by a client not to do so (D) No, because a lawyer shall not discuss a client's case with anyone, including other lawyers of the firm that is handling the case, unless specifically authorized to do so by the client

(C) No, because lawyers shall not discuss a client's case with other lawyers of the firm handling the client's case if instructed by a client not to do so The correct answer is C. Comment 5 to Model Rule 1.6 adds an important (but also obvious) qualification to the confidentiality requirement—lawyers normally may discuss client matters with other lawyers in their firm. Discussing a matter within the firm does not usually result in any outside disclosure of the client's confidential information because all the lawyers share a duty to prevent disclosure of the information to anyone outside the firm. In addition, such discussions enhance the quality of the lawyers' representation (more experienced lawyers can share their expertise), avoids inadvertent mistakes by a lawyer acting alone, and is necessary to identify conflicts of interest that may emerge even after the representation is underway. Nevertheless, Comment 5 includes an exception to this qualification—if a client expressly instructs a lawyer not to discuss the matter with other lawyers in a firm, the lawyer should honor the client's wishes. This is an infrequent occurrence but still an important provision of the rule. Answer C reflects this exception and is therefore correct. Answer A is incorrect—there is no "client's best interest" exception to the confidentiality rule. Answer B is tricky because it correctly states the general qualification in Comment 5 to Rule 1.6—normally, lawyers are free to discuss client matters with other lawyers in the firm—but it overlooks the exception for cases like the one in this problem, where the client expressly asks the lawyer not to do this. Finally, Answer D incorrectly states the rule in an overbroad way. Normally, a lawyer is free to discuss client information with other lawyers in the same firm, without client authorization, but should refrain from doing so if the client expressly forbids it.

During an interview with the local news station, a prosecutor harshly criticized a few of the local judges, blaming them for the backlog of pending criminal cases and their refusal to reimburse the local police and district attorney's office for expensive investigatory costs. He suggested that the judges had organized crime connections. The prosecutor had no evidence to support these allegations, and he admitted that he was going further than most lawyers would in his assertions. The prosecutor's statements were in fact untrue—none of the judges he mentioned had backlogs on their own dockets, and none of them had received any requests for investigatory reimbursements or disbursement authorizations. The state bar brought a disciplinary action against the prosecutor, but the prosecutor claimed that his statements had First Amendment protection and reflected his actual beliefs. Is the prosecutor correct? D (A) Yes, prosecutors have broad prosecutorial discretion and immunity. (B) Yes, if the prosecutor had a subjective belief that his statements were true, he should not be subject to discipline for making these allegations against public officials. (C) No, because the statements were false, and the prosecutor did not have an objectively reasonably belief, at least for an attorney, that these statements were accurate and true. (D) No, it is improper for attorneys to criticize judges or make any public statements that undermine the integrity or credibility of the judiciary.

(C) No, because the statements were false, and the prosecutor did not have an objectively reasonably belief, at least for an attorney, that these statements were accurate and true. Answer C is correct. ABA Model Rule 8.2 provides that an attorney, including a prosecutor, ". . . shall not make a statement that the lawyer knows to be false or with reckless disregard as to its truth or falsity concerning the qualifications or integrity of a judge, adjudicatory officer or public legal officer, or of a candidate for election or appointment to judicial or legal office." Rule 8.2 embodies the standard set forth by the U.S. Supreme Court in the landmark case Garrison v. Louisiana, 379 U.S. 64 (1964)(a case with facts similar to this question). Note that courts use a different standard for First Amendment protection of public statements by lawyers against judges than they normally use in defamation cases involving public officials—attorneys must have an objectively reasonable belief (the "reasonable attorney" standard) that their public criticisms are true, whereas other individuals can escape liability if they have a subjective belief, even without good reason, in the content of their statements. See, e.g., Lawyer Disciplinary Bd. v. Hall, 765 S.E.2d 187 (W. Va. 2014) (the administration of justice and integrity of the legal profession necessitates use of objectively reasonable standard); In re Dixon, 994 N.E.2d 1129 (Ind. 2013) (adopting objective standard of whether attorney had "any objectively reasonable basis for making the statement at issue"); Ky. Bar Ass'n v. Blum, 404 S.W.3d 841 (Ky. 2013) (adopting objective standard); Disciplinary Counsel v. Frost, 909 N.E.2d 1271 (Ohio 2009) ("no reasonable attorney would accept [the lawyer's] charges of bias and corruption as true"). This means that an attorney's subjective belief that the statements are true is no defense in the context of disciplinary proceedings. See, e.g, In re Riordan, 824 N.W.2d 441 (Wis. 2012) (lawyer's subjective belief could "not relieve him of the obligation to demonstrate a factual basis for his comments to the court"); Disciplinary Counsel v. Shimko, 983 N.E.2d 1300 (Ohio 2012) (lawyer's strong beliefs that judge was not unbiased and was acting improperly were objectively false); Statewide Grievance Comm. v. Burton, 10 A.3d 507 (Conn. 2011) (lawyer's claim that she believed allegations true was insufficient to defend against disciplinary action; lawyer must have objective basis for public allegations of judicial misconduct); In re Madison, 282 S.W.3d 350 (Mo. 2009) (same). Answer A is incorrect because even though prosecutors enjoy almost unfettered discretion in deciding which cases to prosecute and which charges to include, they are still subject to certain constraints on slander or libel against judges and other government officials. Answer B wrongly uses the subjective standard for defamation of public officials set forth in N.Y. Times v. Sullivan, 376 U.S. 254 (1964) and Harte-Hanks Commc'ns, Inc. v. Connaughton, 491 U.S. 657 (1989), which are both First Amendment cases. Courts and state disciplinary authorities have held that lawyers can be subject to higher standards than nonlawyers for a variety of policy reasons. Finally, Answer D is incorrect because it is too broad—in some circumstances, it would indeed be appropriate for a lawyer to make public accusations against a judge, if the lawyer has an objectively reasonable belief, for a lawyer, that the statements are true.

An attorney worked for several years as a city attorney for a large municipality in its employment litigation division, defending the municipality against employment-related lawsuits, including discrimination claims. The attorney then left that position and went to work for a federal regulatory agency, the Equal Employment Opportunity Commission. The EEOC is sometimes an adverse party to the municipality where the attorney formerly worked. Even when not involved in the same matter or litigation, their goals and interests are often adverse, as the city attorneys are usually arguing for limitations on employer liability in discrimination cases, while the EEOC generally seeks to expand protections for workers against discrimination by employers. At her new position, the attorney has no assignments that are the same cases or matters in which she participated as a city attorney, but there are a number of cases pending in the office that are adverse to the interests of her former employer, and some in which they are opposing parties in the same litigation. Must the EEOC screen the attorney from such cases in the same way that a private firm would need to do under the Rules of Professional Conduct? (A) Yes, because the attorney may know confidential government information that would provide an unfair advantage to the lawyers at the EEOC (B) Yes, because when a lawyer is disqualified from representation, no lawyer in the agency with which that lawyer is associated may knowingly undertake or continue representation in such a matter without screening measures in place (C) No, when a lawyer is employed by a city and subsequently is employed by a federal agency, the latter agency is not required to screen the lawyer (D) No, because it is more important, from a policy standpoint, to stop employment discrimination everywhere than it is to protect the legal interests of one municipality against its own employees

(C) No, when a lawyer is employed by a city and subsequently is employed by a federal agency, the latter agency is not required to screen the lawyer The correct answer is C. This question is based on Comment 5 to Rule 1.11—lawyers who move from one government unit to another normally do not need to be screened by the second government employer. This is due to the general similarity in interests between the government entities, all of which are supposed to act in the public interest, or for the public good—perhaps also because of the information sharing that regularly occurs between government agencies and subdivisions (which means confidentiality is less of a concern). Here, Answer C is correct—a municipal lawyer who moves to a legal job in the federal government would probably not need to be screened from cases in the same way that a lawyer entering government service from the private sector would. Answer A is incorrect, even though it focuses on confidential information that would present a concern in other contexts, because there is essentially an exception for government lawyers who move from one government position to another. Answer B merely states the rule for imputation of conflicts of interest without addressing the specific exception that would apply to these facts where the lawyer moved from one government position to another. Answer D is incorrect because it focuses on a policy concern that is not part of the rules for conflicts of interest—the merits or importance of a government agency's mission may be an unstated factor in the exception for government lawyers in Comment 5 to Rule 1.11, but if so, it would presumably apply to most or all government entities. Ultimately, this mission-based rationale is not expressed in the Model Rules, making it speculative, so Answer D is not the best answer to this question.

An attorney and his partner sued Conglomerate Corporation in a Nicaraguan court for injuries sustained by local farmers from the toxic effects of a pesticide made by Conglomerate Chemical Company, a closely held affiliate of Conglomerate Corporation. Both companies have headquarters in Texas. Conglomerate Chemical Company was not a party in the litigation in Nicaragua, and the Nicaraguan court rejected Conglomerate Chemical Company's attempt to intervene in the case. Nevertheless, at the end of the case, the Nicaraguan court was confused and entered a billion dollar judgment against Conglomerate Chemical Company, which had manufactured the pesticide, rather than Conglomerate Corporation, which had participated in the lawsuit. The attorney and his partner then brought an action to enforce the judgment against Conglomerate Chemical Company in Texas courts, because Conglomerate Corporation had become insolvent (judgment proof) in the meantime, though it was the entity that had participated in the litigation in Nicaragua. The trial court dismissed the enforcement action because Conglomerate Chemical Company was not a party to the litigation that ended in the judgment that the attorney sought to execute. The attorney then appealed the decision, still hoping to execute the billion dollar judgment against the wrong legal entity. Are the attorney and his partner subject to discipline for bringing a frivolous action and appeal? (A) No, because they are seeking to enforce a judgment for damages against the company that actually manufactured the pesticides that were the subject of the litigation (B) No, because they are seeking enforcement of a foreign judgment based on the facial reading of the foreign court's entered judgment (C) Yes, because they pursued the attempt to execute a foreign judgment in the United States against a company that had not participated in the litigation (D) Yes, because they should have stipulated to the intervention of Conglomerate Chemical Corporation in the litigation instead of opposing it

(C) Yes, because they pursued the attempt to execute a foreign judgment in the United States against a company that had not participated in the litigation The correct answer is C. The facts in this question are reminiscent of those in a major decision about the ethical rules for bringing meritorious claims: "In re Girardi," 611 F.3d 1027 (9th Cir. 2010). Model Rule 3.1 prohibits lawyers from asserting unmeritorious claims and contentions—those with no basis in law or fact. The easier problems in this subject area typically present examples of legal claims not recognized in the American legal system, or completely fabricated facts that the lawyer knows are untrue. In practice, however, the cases are more nuanced, as in this question that involves corporate affiliates. Normally, as here, only the entities involved in the litigation as parties could be subject to a judgment in the case. Thus, it was appropriate for the trial court in Texas to dismiss the action against a company that was not even a party to the litigation. Answer C correctly reflects the rule in this case—the lawyers violated their ethical duties by pursuing a judgment against the wrong party. The Ninth Circuit held that the lawyers in "In re Girardi" were subject to discipline on similar facts. Note that the facts here suggest this was not an innocent mistake on the part of the lawyers; the lawyers knew that the party that lost the litigation could not pay, so they tried to obtain the funds from an affiliated company. Answer A is incorrect because, even though Conglomerate Chemical could potentially have shared in the responsibility or liability for the tort, it was not a party to the litigation and could not be subject to the judgment. The lawyers should have added Conglomerate Chemical as a defendant to the lawsuit if they wanted to pursue the judgment against them later. Answer B is also incorrect because, regardless of the foreign court's opinion, there is no basis in American jurisprudence for executing judgment on a party that did not participate in the litigation. Answer D is partly correct but partly incorrect—the ethical violation of the attorneys was not their strategic failure to stipulate to the other company's intervention in the case, during the trial phase, but instead was their attempt to execute the judgement on a company when there was no conceivable legal claim that they could assert to support this action.

An attorney represented a client in commercial litigation that required the testimony of experts to assess the economic losses suffered, the costs of mitigating the damage, and so forth. After much effort, the attorney located an expert witness who could substantiate his client's claims and refute the testimony of the opposing party's expert witness. The expert witness, however, demanded a large retainer fee to review the case documents and a fee of $1,000 per hour of courtroom time. Was it proper for the attorney to agree to pay the expert witness a princely sum to testify at trial? (A) No, a lawyer may not offer an inducement to a witness, especially an expert witness, who is supposed to provide a purely objective assessment. (B) No, a lawyer may not hire an expert witness unless he pays the witness a contingent fee that depends on the outcome of the case. (C) Yes, it is proper to compensate an expert witness on terms permitted by law. (D) Yes, as long as the lawyer did not select this expert merely because he expects his testimony to be favorable to his client's position.

(C) Yes, it is proper to compensate an expert witness on terms permitted by law. The correct answer is C. Comment 3 to Model Rule 3.4 prohibits paying occurrence witnesses (e.g., eyewitnesses, character witnesses) any inducement to testify except for the reimbursement of expenses incurred in testifying. Experts, however, may charge fees, as long as they do not receive a contingent fee. In other words, expert witnesses may receive a flat fee, or charge hourly rates (or a combination of these two), but cannot have any compensation that depends on the outcome of the trial, whether a percentage of a verdict or a flat bonus if the expert's side ultimately prevails. Answer C is the correct answer—it is proper for the attorney to compensate the expert, even if the expert is expensive. Answer A incorrectly conflates experts and occurrence witnesses—the rules for compensation are very different depending on the type of witness. Answer B states the opposite of the rule about contingent fees—such fees are forbidden, not required, for expert witnesses. Answer D is incorrect because the Model Rules do not prohibit lawyers from hiring experts they hope will give favorable testimony. The lawyer may not coach the expert witness or encourage the expert to testify falsely, but most lawyers hire experts with the expectation that the expert will support their position.

Conglomerate Corporation, an international manufacturing company with diversified holdings in several industries, hired Attorney Barrows from Boutique Firm to represent it in a personal injury lawsuit in which it was the defendant. During early settlement negotiations, Attorney Barrows told the plaintiff that his client, Conglomerate, did not wish to settle the case for more than a few thousand dollars, even though Conglomerate's Board of Directors had in fact tentatively approved up to a million dollars for settlement, subject to final approval by the Board. Did Attorney Barrows violate his ethical duties by making such untruthful statements during settlement negotiations? D (A) Yes, a lawyer must take care not to communicate the client's position in a way that converts it into a false factual representation that the client did not intend. (B) Yes, a lawyer may not make a false statement of fact or law during negotiations, regardless of materiality, especially if the statement could influence the other party's decisions in the matter. (C) No, during negotiations, a lawyer may permissibly make a false statement of material fact or law to a third person or the opposing party in the matter. (D) No, a lawyer may downplay a client's willingness to compromise, or present a client's bargaining position without disclosing the client's "bottom line" position, in hopes of reaching a more favorable resolution.

(D) No, a lawyer may downplay a client's willingness to compromise, or present a client's bargaining position without disclosing the client's "bottom line" position, in hopes of reaching a more favorable resolution. Answer D is correct. Even though ABA Model Rule 4.1(a) states that lawyers must not "make a false statement of material fact or law to a third person," Comment 2 to Rule 4.1 allows some exceptions: "Under generally accepted conventions in negotiation, certain types of statements ordinarily are not taken as statements of material fact. Estimates of price or value placed on the subject of a transaction and a party's intentions as to an acceptable settlement of a claim are ordinarily in this category. . . ." In addition, ABA Formal Op. 06-439 mentions approvingly the type of lawyer statements we see in this Question: "A plaintiff might insist that it will not agree to resolve a dispute for less than $200, when, in reality, it is willing to accept as little as $150 to put an end to the matter." The Opinion concludes: "[S]tatements regarding a party's negotiating goals or its willingness to compromise, as well as statements that can fairly be characterized as negotiation 'puffing,' are ordinarily not considered 'false statements of material fact' within the meaning of the Model Rules." Answer A incorrectly applies a principle that is normally true to this situation. ABA Formal Op. 06-439 explains: We emphasize that, whether in a direct negotiation or in a caucused mediation, care must be taken by the lawyer to ensure that communications regarding the client's position, which otherwise would not be considered statements "of fact," are not conveyed in language that converts them, even inadvertently, into false factual representations. For example, even though a client's Board of Directors has authorized a higher settlement figure, a lawyer may state in a negotiation that the client does not wish to settle for more than $50. However, it would not be permissible for the lawyer to state that the Board of Directors had formally disapproved any settlement in excess of $50, when authority had in fact been granted to settle for a higher sum. Here, the lawyer in the Question expresses that the Board does not "wish to settle for more than a few thousand," using the same language that the above quote identifies as permissible. Answer B is also incorrect, because it says, "regardless of materiality," while Model Rule 4.1 refers only to "false statements of materialfact" (emphasis added), which means that materiality is dispositive to the application of the Rule. Finally, Answer C simply contradicts (negatives) the prohibition in Model Rule 4.1.

Attorney Bates was a well-known family law practitioner who left his old firm and started his own solo practice. After establishing his own practice, he represented a certain client in an uncontested divorce. Sometime later, the same client hired Attorney Bates to represent her in litigation over the custody of her children. The client was concerned about her older sister, in whom she had confided for many years about the client's struggles with substance abuse and mental illness. Her good relationship with her sister had ended, because of an intense argument over money one of them had borrowed from the other. The client claimed that she had been free from substance abuse for three years, and she was now managing her mental health issues very well. Her sister knew many of her darkest secrets. Attorney Bates located the sister, explained that the client was fighting for custody of her children, and that he expected the ex-husband's lawyer to call her to testify at the hearing about the client's former troubles. Attorney Bates then pleaded with her to show some consideration for the years of good relationship and good memories she shared with the client and to refuse to betray her sister's confidence and reveal all her dark secrets. It was reasonable under the circumstances for Attorney Bates to think that the interests of the sister would remain unharmed if she refrained from giving such information. The sister felt deeply moved by this entreaty and agreed to stay out of the litigation. Was it improper for Attorney Bates to request that the sister withhold information from the court or the other party? (A) Yes, because a lawyer may not request a person other than a client to refrain from voluntarily giving relevant information to another party. (B) Yes, because the lawyer has a duty to think about the best interests of the children in this case, rather than his client's convenience or feelings. (C) No, because in a divorce or custody proceeding, a lawyer may ask various potential witnesses to refrain from disclosing information to keep the proceedings from becoming unnecessarily acrimonious or protracted. (D) No, because a lawyer may request that a relative of the client refrain from voluntarily giving relevant information to another party.

(D) No, because a lawyer may request that a relative of the client refrain from voluntarily giving relevant information to another party. Answer D is the correct answer. Even though ABA Model Rule 3.4(f) generally prohibits lawyers from asking individuals not to testify in litigation matters, the rule contains an important exception that applies here: where "the person is a relative or an employee or other agent of a client," and, "the lawyer reasonably believes that the person's interests will not be adversely affected by refraining from giving such information." Answer A is incorrect, even though it accurately restates the general prohibition in Rule 3.4(f), because it ignores the important exception for family members, which applies to these facts. Answer B misstates the lawyer's duties—the lawyer has a duty primarily to the client, and not to the children, unless the children would be in obvious danger, which is not the case here. Finally, Answer C incorrectly suggests that a lawyer may ask anyone to refrain from testifying in a divorce or custody proceeding, which contradicts the prohibition in Model Rule 3.4(f). The only individuals a lawyer may ask not to testify are relatives and employees (or other agents) of the client.

The local public defender represented a suspect that the FBI arrested on armed robbery charges. The defendant's roommate claimed the defendant was at home with him watching a football game at the time that the police say the crime occurred. The defendant rejected the plea bargain that the prosecutor offered and opted instead to go to trial because he felt he had a credible alibi witness. The public defender interviewed the roommate, however, and learned that the roommate did not want to testify at trial or get further involved in the case. Recognizing that the roommate was his client's only chance at an acquittal, the public defender offered to pay the roommate $1,000 in cash to testify at trial, half before and half after the trial. The roommate agreed because he was concerned about paying rent that month, given that the defendant's half of the rent would not be forthcoming. Would the ethical rules permit the public defender to pay the roommate an honorarium to testify? (A) Yes, because contingency fees are the only kinds of fees not allowed to be paid to witnesses for their attendance and testimony at a hearing or trial; lump sum payments are allowed (B) Yes, because witnesses can be paid for their attendance and expenses incurred for attending and testifying at a hearing or trial (C) No, because an attorney cannot pay for witnesses' attendance at a trial or hearing; rather, the fees must be paid directly from the client to the witness (D) No, because an alibi witness cannot be paid to attend and testify at a hearing or trial

(D) No, because an alibi witness cannot be paid to attend and testify at a hearing or trial The correct answer is D. Model Rule 3.4(b) prohibits lawyers from offering inducements to witnesses that are "prohibited by law." Comment 3 to Rule 3.4 clarifies this rule by distinguishing between two categories of witnesses: "occurrence witnesses" and "expert witnesses." The first category includes every witness that is not testifying as an expert. Examples are eyewitnesses (like the roommate in this question), character witnesses, those who merely authenticate business records or laboratory results, coroners, arson investigators, and so forth. As Comment 3 observes, the common law rule in most jurisdictions is that lawyers may not pay an occurrence witness to testify, besides reimbursing their costs to do so, such as travel and lodging necessary for their trial testimony. Answer D is the best answer because the lawyer here persuaded a reluctant witness to testify by offering to pay him to do so, in violation of Rule 3.4(b). Note that the lawyer could offer to cover any reasonable costs for the roommate to travel to the courthouse (cab fare or even airfare if the trial is in another location), overnight lodging, and meals that must be purchased near the courthouse. The same would apply to pre-trial depositions or sentencing hearings. The lawyer cannot, however, offer money simply as an enticement or inducement to testify. Answer A is incorrect because contingent fees are impermissible for both experts and occurrence witnesses. Answer B is partly wrong and partly right—the lawyer may not pay the alibi witness to attend or testify at trial, but he may reimburse expenses incurred for necessary travel, lodging, etc. Answer Cwrongly suggests that the payment would be acceptable if the client pays the witness instead of the attorney. While the Model Rules govern the conduct of lawyers rather than the parties themselves, presumably the lawyer cannot work around the rule merely by having the payment come from the client. See Model Rule 8.4(a) (noting that it is professional misconduct for a lawyer to violate the Rules of Professional Conduct through the acts of another).

Attorney agreed to represent Client, a foreign national living in the United States. Client explained to Attorney that he was a business owner who operated several small grocery stores catering to immigrants from Client's home country. Nothing seemed suspicious to Attorney until they were about to consummate a deal on the purchase of a small parcel of commercial real estate, and Client insisted on paying with cash, arriving at the closing with duffle bags containing bundles of twenty-dollar bills. The parties completed the sale and title transferred to one of Client's businesses, 7777777 LLC. Attorney became suspicious that Client might be laundering money through such transactions. Would it be proper for Attorney to inform the FBI about the transaction without Client's consent? (A) Yes, a lawyer functions as a gatekeeper to the financial system and has an ethical duty to report any suspicion of money laundering (B) Yes, a lawyer may disclose confidential information to prevent the client from committing a crime or fraud that the lawyer suspects might result in substantial injury to the financial interests or property of another and in furtherance of which the client has used or is using the lawyer's services (C) No, because there is no indication here that the transaction could lead to reasonably certain death or serious bodily injury (D) No, because the Rules of Professional Conduct do not mandate that a lawyer perform a gatekeeper role in this context, and mandatory reporting of suspicion about a client is in conflict with the duty of confidentiality

(D) No, because the Rules of Professional Conduct do not mandate that a lawyer perform a gatekeeper role in this context, and mandatory reporting of suspicion about a client is in conflict with the duty of confidentiality The correct answer is D. This question is based on ABA Formal Op. 13-463. Reporting of clients' money laundering activity is a controversial issue in the American legal profession because some international treaties suggest that lawyers in treaty countries make disclosures to expose money laundering to help in anti-terrorism and anti-trafficking enforcement. The ABA has resisted this policy for lawyers in the United States. Under the Opinion 13-463, Answer D is correct—lawyers do not have an ethical duty to disclose confidential client information just because the lawyer suspects money laundering is afoot. Answer A is incorrect in that it takes the opposite position, suggesting that lawyers have a "gatekeeper function" that the ABA has specifically rejected. Answer B invokes the exception in 1.6(b)(2) and (3) for protecting against financial crimes and fraud but incorrectly applies it to these facts. Money laundering is criminal, and certainly furthers other serious criminal enterprises, but it normally does not result in financial injury to other parties—it merely helps criminals elude detection by law enforcement authorities. Answer C incorrectly invokes the exception for saving a life—as with the previous answer, money laundering is a financial scheme to evade detection of other crimes, so it would not implicate the exception for preventing death or bodily injury.

A manufacturer produces the latest technology in magnetic-resonance imaging machines for medical diagnostics in hospitals. Each machine sells for almost one million dollars apiece. Three years ago, the manufacturer hired an attorney to draft a Purchase and Sale Contract for the manufacturer to use whenever it sells one of the devices to a hospital. The attorney's representation ended after drafting this model contract, and the attorney has done no legal work for the manufacturer since. Recently, St. Luke's Hospital hired the same attorney to handle a dispute with the manufacturer of one of its high-end diagnostic machines. The attorney quickly learned that the faulty device is one of his former client's magnetic-resonance imaging machines, and that the hospital administrator consummated the purchase by signing one of the contacts that the attorney himself had drafted. St. Luke's Hospital now merely seeks to rescind the contract and return the machine for a full refund; the hospital has not yet incurred damages due to the faulty machine, but the device is unusable and was very expensive. Would it be proper for the attorney to represent the hospital in this case? (A) Yes, because the hospital is not seeking any damages besides a refund of the purchase price in exchange for returning the faulty machine, which merely puts the manufacturer back in the same place as if the sale had never occurred; therefore, the attorney's representation of the hospital would not be materially prejudicial to the manufacturer (B) Yes, because the attorney's representation of the manufacturer ended three years ago, so there is no conflict of interest or direct adversity between the attorney's current clients (C) No, because under the Rules of Professional Conduct, if a period of three years or more has elapsed since the termination of representation for a former client, no conflict of interest exists between the former client and new clients the lawyer undertakes to represent (D) No, because under the Rules of Professional Conduct, a lawyer cannot seek to rescind on behalf of a new client a contract drafted on behalf of the former client

(D) No, because under the Rules of Professional Conduct, a lawyer cannot seek to rescind on behalf of a new client a contract drafted on behalf of the former client The correct answer is D. Model Rule 1.9 covers conflicts of interest between successive clients. The question here illustrates a clear point made in Comment 1 to Rule 1.9: "Under this Rule, for example, a lawyer could not properly seek to rescind on behalf of a new client a contract drafted on behalf of the former client." Contract rescission can lead to litigation (especially when a million dollars could be at stake), and the outcome of the litigation will turn on whether the contract was valid in the first place, and if so, whether some provision in the contract created a loophole for a party to escape its obligations under the contract without penalty. Either way, the lawyer who drafted the contract would bear ultimate responsibility for failure to draft a valid contract, or failure to draft it in a way that would adequately protect the interests of the original client. In other words, successful representation of the new client would require the lawyer to attack her own work, the contract the lawyer drafted for the previous client. The lawyer is unlikely to be objective about this, besides the vested interest in avoiding a legal malpractice lawsuit from the original client if the new client prevails. Answer D is therefore correct and reflects the statement made in Comment 1 to Model Rule 1.9—the lawyer cannot represent the hospital in seeking rescission of the lawyer's own contract, drafted for the earlier client (the manufacturer). Answer A is incorrect because even if it were correct that a legal victory for the hospital would not harm the former client (which is also unrealistic), the attorney has a material limitation—the representation would require attacking the lawyer's own document. Answer B is incorrect because is suggests that conflicts of interest arise only between current clients. Model Rule 1.9 addresses conflicts with former or successive client, making it clear that the attorney in this problem still has an issue. In addition, the parties' interests are indeed directly adverse—this is a contract dispute that could easily result in litigation. Finally, Answer C wrongly suggests that there is a statute of limitations for conflicts of interest, when in fact there is not. The passage of time since the representation of the previous client could, if long enough, make the former client more likely to consent to the conflict of interest, but it does not erase the conflict automatically.

An attorney is representing a class of plaintiffs in a class action lawsuit over lethal effects of a popular herbal supplement. The class action will have one named plaintiff and approximately 2,000 unnamed plaintiffs. After the class action lawsuit is underway, the same attorney has an opportunity to represent another plaintiff in a personal injury case over an incident at a sporting event, and the defendant is one of the unnamed members of the class action lawsuit over herbal supplements. Does the attorney need to obtain written consent of the unnamed class member before representing the second client in suing the unnamed class member? (A) Yes, because the lawyer will inevitably learn confidential information about the unnamed member of the class in the herbal supplement lawsuit that he could potentially use to the disadvantage of that member in the lawsuit against the member, such as financial information or a history of litigiousness (B) Yes, because both are technically clients of the same lawyer, and their interests will be directly adverse, and there is no reason that the lawyer cannot seek consent from each client (C) No, because the conflict in this case is nonconsentable, given that one client is suing another client of the same lawyer, albeit in an unrelated matter (D) No, because when a lawyer represents a class of plaintiffs in a class action lawsuit, unnamed members of the class are not clients of the lawyer for purposes of conflicts rules; it is unnecessary to obtain a class member's consent

(D) No, because when a lawyer represents a class of plaintiffs in a class action lawsuit, unnamed members of the class are not clients of the lawyer for purposes of conflicts rules; it is unnecessary to obtain a class member's consent The correct answer is D. Class action lawsuits present a special problem for analyzing conflict of interest problems, because the class of plaintiffs can be very large, and the lawyer representing the class normally does not know most of the class members, making it impossible to screen for potential conflicts. Comment 25 to Model Rule 1.7 resolves this problem with a bright-line exception—neither lawyer (plaintiff or defense) in a class action lawsuit needs to worry about conflicts of interest with the unnamed members of the class, because "unnamed members of the class are ordinarily not considered to be clients of the lawyer" when it comes to the conflicts of interest rules. Answer D is correct—the lawyer here does not need to obtain consent before suing someone who happens to be an unnamed member of a class that the lawyer represents in separate class action litigation. As Comment 25 notes, "the lawyer does not typically need to get the consent of such a person before representing a client suing the person in an unrelated matter. Similarly, a lawyer seeking to represent an opponent in a class action does not typically need the consent of an unnamed member of the class whom the lawyer represents in an unrelated matter." Answer A wrongly focuses on the confidential information of the unnamed class member, even though confidentiality is normally a major concern and factor in conflicts analysis. Answer A is wrong partly because the lawyer is extremely unlikely to learn any confidential information about any of the unnamed class members and also because the ABA excludes unnamed class members as clients for purposes of conflicts analysis. Similarly, Answer B ignores the exemption for unnamed class members in the conflicts rules, plus asserts that there is no reason the lawyer could not obtain the class member's consent, which is unreasonable. With such a large class of unnamed plaintiffs, it would be too burdensome to expect lawyers to contact each one beforehand to request consent to potential conflicts of interest. Answer C is incorrect about nonconsentable conflicts—even if this were not a class action lawsuit, the lawyer is not representing adverse parties in the same litigation, so the clients could potentially consent to the conflict. This is unnecessary, though, because Comment 25 states that unnamed members of the class are not even clients for purposes of the conflict of interest rules.

Attorney Abelard practiced with a prestigious local firm in State A. He was one of the country's best-known experts in the specialized area of corporate mergers and acquisitions. Watts Corp. was a large electricity generator and distributor in the New England states. In the wake of recent industry deregulation, Watts plans to engage in a hostile takeover of Primordial Power, the oldest regional electricity provider in the Gulf Coast region. The legal work necessary to undertake and complete the acquisition would require the physical presence of Watts' attorneys in several states besides State A, the only state where Attorney Abelard has a license to practice. Moreover, the merger of the two corporate entities would eventually require appearances before three or more federal agencies in Washington, D.C.—the Department of Energy, the Federal Trade Commission, the Securities and Exchange Commission, and even (tentatively) the Department of the Interior. Are Attorney Abelard and the other lawyers in his State A firm subject to discipline for undertaking this legal work across jurisdictional lines? (A) Yes, because handling the acquisition work in other states besides State A involves Attorney Abelard practicing law without a license in those jurisdictions, even though it is only temporary. (B) Yes, because Attorney Abelard should at least hire local counsel in each jurisdiction to work as co-counsel on the matter. (C) No, if consummation of the matter includes appearances before federal agencies, federal law would preempt state law licensing requirements, so Attorney Abelard can conduct legal work on the matter anywhere. (D) No, given the federal nature of the legal work and the temporary legal work that is reasonably related to the lawyers' practice in a jurisdiction where they are admitted, Attorney Abelard and other members of his firm may represent Watts in the acquisition.

(D) No, given the federal nature of the legal work and the temporary legal work that is reasonably related to the lawyers' practice in a jurisdiction where they are admitted, Attorney Abelard and other members of his firm may represent Watts in the acquisition. Answer D is the best answer. Even though lawyer may not operate an ongoing law practice (seeking and representing new clients) in jurisdictions where the law does not have a law license, both the ABA Model Rules and the Restatement would permit a lawyer to provide incidental representation under the circumstances described here. Model Rule 5.5(c)(4) allows an exception for legal services on a temporary basis that "arise out of or are reasonably related to the lawyer's practice in a jurisdiction in which the lawyer is admitted to practice." Similarly, the Restatement § 3 (Jurisdictional Scope of the Practice of Law by a Lawyer) in Comment d, permits representation across jurisdictional lines when "the activities of the client involve multiple jurisdictions or the legal issues involved are primarily either multistate or federal in nature," and follows with an illustration that has analogous facts to this Question (Illustration 4). Answer A is incorrect because the rules covering the unauthorized practice of law contain a few specific exceptions, such as pro hac vice appearances and situations like the one described here, which are related to the lawyer's normal practice in the state where he has a law license. Answer B is incorrect, though hiring local counsel is also an exception under Rule 5.5(c)(1) and would certainly be one option for Attorney Abelard, it is not necessary, because the activities here already fall under exception 4 to the same rule. Answer C is incorrect, because the fact that there is a federal component to the overall representation does not avoid the duty to comply with state rules governing legal practice for the parts of the representation pertaining to state law.

Attorney Zarathustra needs to attract more clients to his small law firm. An internet marketing company, GlomOn, advertises "daily deals" and permits users to receive frequent email notifications of daily deals that might interest them. GlomOn makes arrangements with local businesses to offer goods or services at discount rates to GlomOn subscribers. After a certain number of subscribers purchase a particular daily deal, GlomOn splits the proceeds with the local business, and the purchaser receives a code or electronic voucher with an expiration date. Attorney Zarathustra decides to use GlomOn to find new clients, and offers an online deal for $400 off a client's legal fees if they retain the attorney. Attorney Zarathustra honors these commitments and resists the urge to raise his rates for GlomOn clients in order to offset the $400 rebate, so his GlomOn advertisements are not misleading in any way. GlomOn costs Attorney Zarathustra more than other internet advertisers. GlomOn promotes Attorney Zarathustra's message to a large number of subscribers, and GlomOn handles the processing of payments from the coupon purchasers. Most of the clients generated by the daily deal advertisements need very routine legal representation that generates low fees, such as submitting government forms, giving legal advice, or contract review. These tasks often generate fees of $500 or less for the entire representation. Thus, nearly the entire fee paid by GlomOn customers actually goes to GlomOn, not to Attorney Zarathustra. Could Attorney Zarathustra be subject to discipline for marketing his legal services through GlomOn in this way? (A) No, because the Model Rules do not regulate internet advertising for lawyers (B) No, because the fee is reasonable, given the services that GlomOn provides to advertisers (C) Yes, because it is an unreasonable fee for advertising if it is higher than comparable advertisers and most of the initial fee goes to the advertiser (D) Yes, because this constitutes fee sharing with nonlawyers

(D) Yes, because this constitutes fee sharing with nonlawyers The correct answer is D. ABA Formal Opinion 13-465 addressed the newer social media marketing companies like Groupon and others that offer "daily deals," such as the one described in this question. The ABA had two primary concerns in this formal ethical opinion: the truthfulness of the daily deal ads, and the problem of fee sharing with nonlawyers. On the truthfulness side, the problems that seem to have emerged in this area relate to lawyers informing clients who come in for consultations that their case is too complicated and therefore will cost more than the coupon indicated, or lawyers offering discount coupons for initial consultations that are already free at that law firm (making the "discount" misleading). The fee-sharing issue is with lawyers paying a percentage of the income generated by the daily deal advertisement to the marketing company. It is permissible to pay a flat fee (say, monthly or weekly) for the advertisements but not to split the fees. Answer Dis the best answer and reflects the recent position taken by the ABA in its formal ethics opinion. Answer Aincorrectly suggests that the Model Rules do not regulate internet advertising—they do, and some of the Comments for Rules 7.1-7.3 specifically discuss internet banner ads, etc. Answer B misses the point that lawyers are at risk of fee-sharing arrangements due to the cost-payment structures for some of these social media marketing services. Answer C is incorrect because there is no rule prohibiting a lawyer from using an expensive form of advertising—the cost of the advertising does not matter for purposes of the ethical rules governing ads and solicitation of clients.

Attorney Prince is the managing partner of a twelve-lawyer firm that specializes in criminal defense work. He assigns cases to the associate lawyers, who are his subordinates, and supervises their work. In handling workload issues, he frequently must balance competing demands for scarce resources. The firm receives a steady stream of court appointments for representation of indigent defendants, and they also have a constant influx of walk-in clients seeking representation in their misdemeanor and felony cases. Attorney Prince knows that he should monitor the workload of the supervised lawyers to ensure that the workloads do not exceed a level that may be competently handled by the individual lawyers. On the other hand, Prince feels strongly that that indigent defendants are better off receiving some legal help than no help at all, so he is unwilling to turn away any clients. Given that most of the criminal cases result in plea agreements, sometimes taking only a few hours to negotiate and finalize, usually there is a diminishing marginal value in the lawyers expending time and effort on background investigation or legal research for individual cases. In addition, Prince knows that the other lawyers working as his subordinates have an ethical duty to manage their own workloads, so he feels he is not responsible to duplicate that effort. As a result, he continues to assign new cases to associates, even when they complain that they already feel overwhelmed. Is Attorney Prince subject to discipline? (A) Yes, if a supervisor knows that a subordinate's workload renders the lawyer unable to provide competent and diligent representation and the supervisor fails to take reasonable remedial action, the supervisor himself is responsible for the subordinate's violation of the Rules of Professional Conduct. (B) Yes, because it violates the constitutional right to a jury trial and assistance of counsel for lawyers to advise indigent defendants to enter a guilty plea and waive their trial rights. (C) No, most of the defendants would indeed be worse off if they had to represent themselves, so the associates' minimal representation is better for them than nothing. (D) No, a lawyer cannot face disciplinary actions for the unmanageable workload of a subordinate, as the subordinate has a responsibility to decline more cases.

(A) Yes, if a supervisor knows that a subordinate's workload renders the lawyer unable to provide competent and diligent representation and the supervisor fails to take reasonable remedial action, the supervisor himself is responsible for the subordinate's violation of the Rules of Professional Conduct. Answer A is correct. ABA Model Rule 5.1 imposes disciplinary liability on supervisory attorneys for the misconduct of subordinates under some circumstances. Specifically, supervisory lawyers (such as Attorney Prince in this Question) must "make reasonable efforts to ensure that the firm has in effect measures giving reasonable assurance that all lawyers in the firm conform to the Rules" (Rule 5.1(a)) and must "make reasonable efforts to ensure that the other lawyer conforms to the Rules of Professional Conduct" (Rule 5.1(b)). Moreover, ABA Formal Op. 06-441 specifically applies this rule to lawyer supervisors in law offices that represent indigent criminal defendants: "If a supervisor knows that a subordinate's workload renders the lawyer unable to provide competent and diligent representation and the supervisor fails to take reasonable remedial action, the supervisor is responsible for the subordinate's violation of the Rules of Professional Conduct." Answer B is incorrect because it is appropriate to advise an indigent defendant to enter a guilty plea, waive a jury trial, and accept the plea agreement offered by the prosecutor. Indeed, in many cases, a plea agreement is the defendant's best option. Answer C is the wrong answer even if it accurately portrays the dilemma. It may be true that defendants are even worse off with no representation than they are with minimal or incomplete representation, though some empirical studies in the academic literature dispute this, but lawyers cannot evade their ethical obligations under the rules because of some larger social problem, such as the shortage of lawyers available to represent indigent criminal defendants. Answer D contradicts the express provisions of Model Rule 5.1—lawyers can indeed face disciplinary action for the mistakes or misconduct of subordinates under the conditions specified in the Rule.

An attorney represents a large corporation with several offices around the state. After receiving a few employee complaints about workplace discrimination from one office, the company asked the attorney to investigate the situation and advise the company about its potential liability. The attorney conducted a careful investigation and wrote a thorough memorandum summarizing her findings and legal conclusions. The attorney then emailed the memorandum to all the company's human resources managers in each of its offices statewide—39 recipients in all. When litigation eventually ensued over the alleged discrimination, the plaintiffs sought discovery of the attorney's memorandum, but the corporation and their attorney maintained that attorney-client privilege applies to the memorandum. Is this position correct? (A) No, because the attorney was investigating a few separate complaints, so the memorandum did not pertain to any specific lawsuit but was a general inquiry (B) No, because sending the memorandum to so many employees who had no connection to the matter waived the privilege (C) Yes, because the communication was part of a private communication between a lawyer and a client who was seeking legal advice (D) Yes, because the memorandum was prepared in anticipation of upcoming litigation, and therefore qualifies as attorney work product

(B) No, because sending the memorandum to so many employees who had no connection to the matter waived the privilege The correct answer is B. Lawyers and their clients often waive attorney-client privilege inadvertently, by disclosing otherwise privileged information to third parties. Courts have struggled to apply the waiver rules in the context of lawyer communications with large corporate clients because communication with the top management (that is, several individuals) is often necessary to give legal advice to the client, which is an entity. At the same time, most individuals in a corporation are not involved in the company's legal matters, and disclosures to these employees as "third parties" can waive attorney-client privilege. Thus, email communications to a broad audience of mid-level or lower-level managers can waive privilege. For this reason, Answer B is the correct answer to this question—sending the memorandum to 39 individuals across the state would certainly result in waiver under current case law in this area, especially as the investigation involved complaints from only one office. Answer A incorrectly suggests that privilege must pertain to a single potential lawsuit or source of liability. The memorandum in this question would have been privileged if the lawyer had made it a private communication only to the individuals at the corporation who were responsible for the matter. Private client communications with lawyers asking for advice about a variety of legal issues would normally come under the attorney-client privilege. Answer C, on the other hand, accurately states the general rule for privilege, but overlooks the waiver problem presented here. Even if attorney-client privilege would normally apply to the memorandum in this question, the attorney waived privilege by emailing the document to an unnecessary number of individuals in the company. Finally, Answer D wrongly conflates the attorney work product doctrine with attorney-client privilege. The work product doctrine probably would not apply in this case either (the investigation described seems to have occurred long before litigation was on the horizon), but even if it did, the attorney would have waived privilege for the document by disclosing it to so many individuals.

A woman who had recently moved into town made an appointment for a consultation with an attorney at Small Firm to discuss obtaining a divorce from her husband, who was also a lawyer. When the consultation occurred, the attorney asked the woman to "tell him the whole story," that is, to explain her situation and her objectives in the representation. The attorney did not ask for a waiver of confidentiality. The woman went into considerable detail about the problems in her marriage and her motivations in seeking a divorce, and she also discussed the couple's finances and her husband's personal assets and investments. After the consultation, the woman decided not to retain this attorney because she felt his firm's fees were too high, and she hired another lawyer instead. Two weeks later, the same woman's husband, who was a lawyer working for the municipal government, wanted to hire Small Firm to represent him in the divorce proceedings. Which of the following statements is false? (A) The attorney who conducted the consultation with the wife cannot represent the husband, unless the wife now gives informed consent, confirmed in writing. (B) The attorney who conducted the consultation with the wife may indeed represent the husband, because the wife never became a client of the attorney or his firm. (C) If the attorney who conducted the consultation did not share any confidential information with other lawyers in Small Firm, Small Firm can represent the husband if it immediately implements measures to screen the attorney from the matter. (D) The fact that the husband himself is an attorney does not preclude him from hiring Small Firm to represent him.

(B) The attorney who conducted the consultation with the wife may indeed represent the husband, because the wife never became a client of the attorney or his firm. Answer B is the best answer. ABA Model Rule 1.18 imposes some duties to lawyers toward their prospective clients, that is, those who have an initial consultation with the lawyer, even if the person decides not to hire the lawyer or the lawyer declines to provide representation in the matter. These duties mostly involve confidentiality of information the client discloses during the consultation, which would also come under attorney/client privilege protection, but the effect of the rule can be disqualification of that attorney from representing adverse parties, as if there were a conflict of interest. The Restatement (Third) of the Law Governing Lawyers § 15 more or less mirrors the current provisions in Model Rule 1.18 and its official Comment (revisions to the rule have occurred more than once over the years), and this question is reminiscent of one of the examples in the Restatement. Note that disqualification can apply even though the individual who had the consultation never became a client of the attorney or the firm, and this disqualification imputes to the entire firm unless the firm screens the disqualified lawyer in a timely manner from subsequent representation of other parties in related matters. The call of the question here is which of the answer options is FALSE, and that is B—the attorney who learned confidential information during a consultation with the wife cannot subsequently represent the husband in the same divorce proceeding, even though the wife never hired the attorney. This is the only answer among the four that is not true. Answer A is incorrect because it is a true statement—if the prospective client consents to a waiver of confidentiality, the attorney or his firm can proceed with an adverse representation. Answer C is not the best answer because Small Firm can, in fact, screen the attorney from the subsequent representation of the husband so long as the attorney has not shared any confidential information gleaned during the consultation. Similarly, Answer D is not the best answer because the husband's status as a lawyer does not change the duties that the firm owed to the other prospective client (the wife) under the ethical rules.

Attorney Wilson has no litigation experience because she exclusively handles business transactional work. Attorney Wilson has represented Client in a number of her business transactions. In one instance, Wilson prepared a detailed non-compete agreement for Client to use with a nationally known mathematician whom Client hired to work on Client's predictive coding algorithms. After signing the agreement and working with Client's company for a short period of time, the mathematician left Client's company and began working for Client's main business rival, apparently in violation of the non-compete agreement. Client asked Attorney Wilson to bring an enforcement action against the mathematician. Wilson declined to represent Client in the litigation. Attorney Wilson reminded Client that their previous retainer agreement pertaining to the non-compete agreement specifically stated that Wilson's representation would include only the drafting and related transactional work and would not include handling litigation to enforce or nullify the non-compete agreement. Would Attorney Wilson be subject to discipline for including this provision in her agreement to represent Client? (A) Yes, because under the Rules of Professional Conduct, a client has the sole right to determine the scope of the representation (B) Yes, because the Rules of Professional Conduct forbid lawyers from preparing non-compete agreements or similar documents that limit employees' ability to practice in their field (C) No, because a lawyer may reasonably limit the scope of the representation, by informed agreement with the client, at the beginning of the representation (D) No, because Attorney Wilson has no litigation experience and could not competently have represented Client in the enforcement action

(C) No, because a lawyer may reasonably limit the scope of the representation, by informed agreement with the client, at the beginning of the representation The correct answer is C. This question illustrates a common misunderstanding that arises between clients and their lawyers: a client sometimes assumes that a lawyer hired to handle one matter will automatically handle every legal matter that the client encounters. The Model Rules, therefore, encourage lawyers to communicate clearly with clients to define the scope of the representation. Lawyers may limit their representation to a single matter, or even a particular part of a client's matter (such as pre-trial work but not the actual trial, or transactional work but not litigation), provided that the limitation is reasonable under the circumstances. Answer C captures the essence of the rule about limiting the scope of the representation. The attorney here is correct that she has no ethical duty to provide legal services to the client beyond their prior agreement, which was exclusively for transactional work. Answer A is incorrect because the client does not, in fact, have the sole right to determine the representation. A lawyer is free to agree to do certain kinds of legal work (or certain parts of the client's matter) and not others, when the limitation is reasonable. Answer B is incorrect because there is no ethical prohibition that applies to the preparation of non-compete agreements for a client, except that lawyers themselves cannot be subject to such agreements and cannot prepare them on behalf of another lawyer. Answer D is incorrect because Attorney Wilson may indeed start doing litigation work for clients at any time. The Model Rules allow for lawyers to venture into unfamiliar areas of practice on behalf of a client, as long as the lawyer invests the necessary time and research to acquire the necessary knowledge.

As part of his investigation of a client's matter before trial, Attorney Roberts met with several people familiar with the client's situation to gather information and determine if any of the individuals could be a potential witness at trial. One of the client's co-workers told Attorney Roberts several disturbing stories about corruption at the client's workplace. Later, at trial, the opposing party moved to compel Attorney Roberts to disclose the information conveyed by the client's co-worker. Attorney Roberts objects that this information falls under the attorney-client privilege and is therefore inadmissible. Is Attorney Roberts correct in this assertion? (A) Yes, because the stories are confidential information related to the representation (B) Yes, as long as the client wants the attorney to keep the information confidential, because the client is the holder of the privilege (C) No, because the information did not come from the client, and therefore attorney-client privilege does not apply (D) No, because the attorney-client privilege does not apply during trials, but only to communication outside the courtroom

(C) No, because the information did not come from the client, and therefore attorney-client privilege does not apply The correct answer is C. The common law doctrines of privilege and attorney work product overlap somewhat with the modern ethical duty of confidentiality, but a few important features distinguish these rules. All three protections can, and often do, apply to the same information. Privilege is primarily an evidentiary-exclusionary rule—a basis for objection to official requests for information, as from a court or enforcement agency. Confidentiality forbids disclosures made even in informal, social settings (though disclosures made in these settings could ultimately waive privilege when the lawyer tries to assert it sometime later at trial). Another important distinction is that attorney-client privilege applies only to information disclosed by the client herself to the attorney (for seeking legal advice or representation), whereas confidentiality can apply to information from any source. In this question, therefore, Answer C is correct—the information at issue came from a witness, not from the client. Attorney-client privilege does not apply to statements from individuals other than the client or the client's agents. Answer A is wrong because it ignores this important distinction regarding the source of the information—even though the information is confidential (the lawyer generally has an ethical duty to protect the information, regardless of the source), the attorney-client privilege does not apply and does not govern its admissibility in court. Answer B incorrectly suggests that the client's wishes control whether information is privileged. While the client's intentions matter for determining whether the client's own statements are privileged, the client cannot invoke privilege for statements made by others. Finally, Answer Dincorrectly states that the privilege does not apply at trials—in fact, the opposite is true. Attorney-client privilege is a rule of admissibility at trial and applies only in the litigation context.

Attorney Doubletree is general counsel (in-house) for a corporation, but he is also the acting chief financial officer due to his background in corporate finance and economics. Attorney Doubletree's duties are almost evenly divided between legal tasks for the company (contract review, regulatory compliance, and supervising the outside firms that handle the company's litigation) and corporate financial operations. Every week, Attorney Doubletree prepares or reviews financial reports, investment strategy proposals, and various emails or memoranda relating to the firm's financial affairs. An opposing party in antitrust litigation against the corporation seeks to compel production of some of Attorney Doubletree's financial reports and strategy proposals, but Attorney Doubletree claims these are covered by attorney-client privilege, as he simultaneously serves as the corporation's in-house lawyer. Is Attorney Doubletree correct? (A) Yes, because these are internal communications between corporate managers and their in-house counsel (B) Yes, as long as the documents were not available to all the lower-level employees at the company (C) No, because there is no indication that Attorney Doubletree marked these documents as "privileged and confidential" at the time of drafting (D) No, because these are business communications, not legal advice from the lawyer to the client

(D) No, because these are business communications, not legal advice from the lawyer to the client The correct answer is D. It is relatively common for in-house counsel to serve other roles in a corporation, such as the function here (CFO), or human resources director, or other management positions. Serving in these business roles will not automatically transform regular business communications into privileged ones. Instead, most courts find that communications with a business purpose will not trigger the attorney-client privilege—business advice is not legal advice, even if the communication comes from someone who happens to be a lawyer. Answer D is therefore the best answer—the communications mentioned here are business-related and come under Attorney Doubletree's role as CFO, not his role as legal advisor. Answer A is incorrect because not all communications between in-house counsel and managers are privileged, even if they are confidential—privilege applies only to legal advice or representation. Answer B misses the point by focusing only on the recipients instead of the contents of the communications—both are necessary components of attorney-client privilege. Finally, Answer C mistakenly suggests that a failure to mark documents as "privileged" is the reason privilege would not apply. As long as the communications are obviously private or confidential, privilege could apply, even if there is no express assertion at the time that the document comes under the privilege doctrine. The problem is that the documents in the question here are business documents, not legal advice, so attorney-client privilege does not apply.

A major city had a string of murders over a period of three years, and eventually the police realized the murders were the work of a serial killer. A suspect was eventually arrested and was awaiting prosecution. At a press conference, the prosecutor stated that the defendant was single, lived with his mother in a particular apartment complex in the city, and worked as a butcher. Was it proper for the prosecutor to disclose such information about the defendant to reporters? (A) No, because a lawyer in a criminal case may not disclose the residence, occupation, or family status of the accused (B) No, because the defendant is on trial for murder, so special ethical duties automatically apply to the prosecutor's public statements (C) Yes, because a prosecutor represents the people and the public disclosures are necessary communications between a lawyer and his clients, the taxpayers (D) Yes, because a lawyer in a criminal case may state the identity, residence, occupation, and family status of the accused

(D) Yes, because a lawyer in a criminal case may state the identity, residence, occupation, and family status of the accused The correct answer is D. Model Rule 3.6(b) sets forth a list of permissible types of statements that lawyers (including prosecutors) may make to the news media about pending litigation, and Rule 3.6(b)(7)(i) provides that in criminal proceedings, lawyers are free to disclose "the identity, residence, occupation and family status of the accused," among other things. Answer D is therefore correct, as it echoes the language of the rules. The prosecutor here was within bounds. Answer A, in contrast, directly contradicts the rule, and is therefore incorrect. Answer B is more tricky because the Comment to Rule 3.6 suggests that lawyers should be more restrained (and disciplinary authorities should impose higher standards) when the case involves criminal charges or the stakes are high, both of which apply here. Even so, the prosecutor has explicit authorization under the rules to state the accused's name, address, marital status, and occupation. Answer C incorrectly turns the focus from the explicit provisions in the rules to a policy argument about the prosecutor representing the people, which does not align well with the statements in the Comment to this rule and to Model Rule 3.8, both of which suggest that prosecutors should be especially cautious in media statements, out of the need to safeguard due process for criminal defendants.

At 3:00 pm on December 24 an attorney received a small settlement check for a client from the opposing party. All the support staff in the attorney's office had already gone home, and the office was to stay closed until January 2. The attorney was rushing to catch a flight to Europe, where she planned to spend the holidays with her family. On January 2, when the attorney returned and the office reopened, she promptly notified the client that the check had arrived. Did the attorney violate the ethical rules for this delay? (A) Yes, because she should have instructed the opposing party to send the check directly to the client (B) Yes, because she did not notify the client soon enough (C) No, because the office was closed during the entire period, and the attorney notified the client promptly as soon as the office reopened (D) No, because the attorney had to catch a flight, and the client suffered no harm from this delay

(B) Yes, because she did not notify the client soon enough The correct answer is B. Delays in notifying the client about funds is a violation of the Model Rules, which require prompt notification. When funds arrive at the firm, either the attorney or the support staff should notify the client as soon as possible (ideally, within a day). Here, the attorney was leaving on vacation and the office was closed for a week, so she delayed contacting the client for a full week. Holidays and vacations are times when lawyers could miss important deadlines or postpone some necessary communication. Answer B correctly states that the attorney is subject to discipline for this delay. Even though the attorney's behavior was somewhat understandable, given the holiday, the overseas vacation, and the office closure, she neglected a duty to her client, and it would have taken only a moment to call or send a message to the client about the funds (or to have a secretary do this). Answer A is incorrect because there is no duty or requirement that opposing parties send funds directly to the client. Instead, it is customary for the payment to go to the attorney first. Answer C is incorrect because it does not matter that the office was closed—rather, that made it particularly urgent to contact the client immediately because delaying another day meant the communication would not happen until the office reopened. Answer D is incorrect because attorneys must honor their duties to their clients and their obligations under the ethical rules, regardless of their personal schedules. A related issue is that the client property (the check) may not have been properly safeguarded. The best way to safeguard client funds is to immediately deposit the check in a trust account maintained by the attorney. Failure to do so risks the check being lost or the possibility that the check could be returned for insufficient funds. Attorneys should have office procedures in place to address situations such as the one described in the problem.

Attorney Simpson represents several clients in various matters before the Federal Trade Commission. In one proceeding involving one of Attorney Simpson's clients, the FTC adopts a new interpretation of a recently enacted statute about unfair trade practices, and this becomes the rule of the case. Attorney Simpson has some new clients who are at the beginning stages of an FTC inquiry of their business pertaining to the requirements of the unfair trade practices statute. Without mentioning the identity of the other client or the exact nature of the proceedings, Attorney Simpson informs the new clients that the FTC has just adopted a particular interpretation that could be very favorable to the new clients in their interactions with the agency, as long as the clients conduct certain internal audits and recordkeeping. Could Attorney Simpson be subject to discipline for sharing with new clients this information he learned during the representation of the other client? (A) No, a lawyer who learns a government agency's interpretation of relevant legislation during the representation of one client may properly use that information to benefit other clients, because it does not harm the other client (B) No, because the other clients could eventually have discovered the FTC's new interpretation of the statute without the attorney providing the information, given that some written decision of the agency ruling would eventually be available (C) Yes, because it violates public policy for lawyers to exploit government agency interpretations used against one regulatory violator to help other potential violators gain an advantage against the agency (D) Yes, because using information gleaned from representation of a client to the advantage of another client violates the lawyer's duty of loyalty, even when the lawyer uses the information to benefit a third person, such as another client

(A) No, a lawyer who learns a government agency's interpretation of relevant legislation during the representation of one client may properly use that information to benefit other clients, because it does not harm the other client The correct answer is A. Model Rule 1.8(b) prohibits lawyers from using a client's confidential information in a way that harms that client, including giving the information to other clients to help them. Comment 5 to Rule 1.8, however, adds an important qualification that students might overlook—the lawyer violates the rule only if the use of the information somehow harms the original client in the situation. On the other hand, many instances of representation provide a valuable learning experience for the lawyer that enriches the lawyer's knowledge of the law or of legal practice. As lawyers gain experience through representing many clients, their knowledge base grows and they provide future clients with representation that is more effective and more efficient (the lawyer may not have to research a legal question whose answer she already knows from a previous case, for example). Answer A is the correct answer here, because a lawyer learning a government agency's interpretation is very useful for representing the other clients effectively but does not hurt the original client. Answer B misses the point that the lawyer here would not have violated the ethical rules even if the information were not public (such as an unstated current policy of the agency about prioritizing enforcement actions against regulatory violators). The fact that the information is not in fact private or confidential is of course relevant as well, but the question here is focused on what the attorney did with the information and how it affected the clients involved. Answer C is incorrect partly because the public policy argument expressed is not part of the analysis for conflicts of interest (the supposed "unfair advantage" is against an opposing party, not against one of the lawyer's clients), and partly because the Model Rules elsewhere state explicitly that lawyers may inform their clients about the relevant law, even if the client uses that information to find ways around the law. Answer D states the basic thrust of Rule 1.8(b) but ignores the fact that in this question, the lawyer did not use the information to the disadvantage of the client, which is an essential element of this rule.

Parent retains Attorney to represent Defendant, who is Parent's 16-year-old child accused of shoplifting. Because Parent is paying for his services and because Defendant is a minor, Attorney generally communicates with Parent about the proceedings, options for disposing of the case, and other case-related issues. After speaking with Parent about a plea deal that would allow Defendant to do several community service hours and have the case dismissed, Parent advises Attorney that Defendant will take the deal. Attorney contacts the prosecutor who sends the paperwork for Defendant to complete and then cancels the court appearance, advising the court that a plea deal has been reached. Are Attorney's actions proper? (A) No, attorneys are required to continue a normal relationship with their client as much as possible, even if the client has diminished capacity. (B) Yes, minor children are considered incapacitated, and attorneys can deal solely with the parents or guardians of a minor when handling cases for the minor. (C) Yes, as long as the deal is not unreasonable to the minor, attorneys have no obligation to work directly with a client with diminished capacity. (D) No, attorneys are not required to continuously communicate with a client with diminished capacity but are required to allow the client to make the final decision on the client's case, even if client's capacity is diminished.

(A) No, attorneys are required to continue a normal relationship with their client as much as possible, even if the client has diminished capacity. The correct answer is A. Representing minors can present special challenges for the lawyer because the underage client lacks the legal capacity to make certain decisions (that is, legally binding decisions). The Model Rules group minors together with adults having diminished capacity (whether due to old age or mental illness). Even though the relationship is not exactly the same as representing an adult client with full capacity, the Model Rules require lawyers to maintain a normal relationship with underage clients as much as possible. The fear is that lawyers will simply substitute their own judgement for the underage client's, given that the client cannot make legally binding decisions, and leave the client out of the decisions. Lawyers should include the underage client in decisions about the case as much as possible. Answer A correctly reflects this policy—the lawyer here should have consulted with the client instead of making such an important decision on his own. Answer B wrongly states that lawyers can leave underage clients out of all the decisions. The Model Rules require lawyers to include underage clients in the deliberations related to major decisions (such as a plea bargain) as much as possible. Answer C incorrectly suggests that the reasonableness of the outcome for the client justifies the lawyer's actions. Even if the lawyer knows the right course of action to take, there is still an ethical duty to let the underage client participate in the discussion and take part in the decision-making process. Finally, Answer D is incorrect because it creates an incorrect distinction between final decisions and other decisions that affect the representation. The lawyer has an ethical duty to include an underage client in as many decisions about the case as possible (that is, the decisions that any other client would get to make), not merely the final decisions.

A manufacturing firm received a complaint about a consumer who suffered injuries from using one of the manufacturer's products. The manufacturer referred the complaint to its legal department, and the in-house lawyer who investigated the complaint concluded that the complaint posed no legal issues for the company because of a state statute that shielded manufacturers of that type of product from tort liability. The attorney wrote a legal memorandum to the company's management describing his investigation and conclusions. The memorandum included a section of factual findings and another section presenting the legal analysis. Are the facts described in the lawyer's memorandum privileged and protected against discovery at a subsequent trial? (A) No, because attorney-client privilege does not apply to underlying facts, even if those facts are discussed in an otherwise privileged communication (B) No, because the lawyer who wrote the memorandum was in-house counsel at the manufacturer, so the company never communicated with an outside law firm seeking legal advice (C) Yes, because the communication was part of a private communication between a lawyer and a client who was seeking legal advice (D) Yes, because a state statute shielded the manufacturer from liability for injuries from this type of product

(A) No, because attorney-client privilege does not apply to underlying facts, even if those facts are discussed in an otherwise privileged communication The correct answer is A. Attorney-client privilege does not apply to underlying facts discussed in otherwise confidential attorney-client communications. The memorandum's recitation of the facts is not privileged, so Answer A is correct. Answer B wrongly suggests that privilege does not apply to communications between in-house counsel and corporate management; such communications are, in fact, normally within the coverage of privilege. Answer C is incorrect because it ignores an important qualification to the attorney-client privilege—it applies to communications but not to underlying facts. Even though Answer C accurately states the basic idea of privilege, the privilege does not apply to the facts that the lawyer discovered during his investigation. Answer D, on the other hand, relies on an irrelevant point—the statutory preclusion of liability may serve to defeat a claim in court, but it is not pertinent to the question of whether attorney-client privilege prevents the disclosure of the underlying facts. An attorney's investigative fact finding does not become privileged merely because it is part of an attorney's memorandum.

Suzanne hired Attorney Zigler to represent her in a divorce proceeding and custody battle over Suzanne's children. At one point, Suzanne explained to Attorney Zigler that if she loses custody of the children to her estranged spouse, she has detailed plans to murder the spouse and make it look like a suicide so that she can regain custody of her children. Attorney Zigler believes that Suzanne could plausibly carry out this plan successfully, and Attorney Zigler is reasonably certain that Suzanne will indeed lose custody of the children in the current proceeding. May Attorney Zigler immediately warn the estranged spouse, the tribunal, or the police about Suzanne's plan? (A) No, because whenever practicable, the lawyer should first seek to persuade the client to take suitable action to obviate the need for disclosure (B) No, because the far-fetched plan is still contingent on losing custody of the children, and therefore it does not constitute reasonably certain death or bodily injury, and thus fails to trigger the exception to the duty of confidentiality (C) Yes, because a lawyer may reveal information relating to the representation of a client to the extent the lawyer reasonably believes necessary to prevent reasonably certain death or substantial bodily harm (D) Yes, because Suzanne's plan falls outside the scope of Attorney Zigler's representation in the current proceeding, and therefore the information does not come under the duty to protect client confidentiality

(A) No, because whenever practicable, the lawyer should first seek to persuade the client to take suitable action to obviate the need for disclosure The correct answer is A. Comment 16 to Model Rule 1.6 adds an important clarification about how lawyers should approach exceptional situations, that is, cases in which one of Rule 1.6's specific exceptions to the duty of confidentiality could apply. If possible, the Comment explains that the lawyer should first try to persuade the client to make the disclosure in question, or at least authorize the lawyer to do so (for example, to save a life or to prevent financial injury to another party). Of course, clients will often refuse, in which case the lawyer can invoke the applicable exception and make the necessary disclosure, over the client's objection. The point is that clients should first have the chance to do the right thing in the situation, and if the client agrees, it avoids any conflict between the lawyer and the client over the matter. Answer A reflects the guidance in Comment 16—Attorney Zigler should first try to talk sense into Suzanne, before rushing ahead to report the client's secret plan. Answer B does not accurately reflect the facts—the client had a well-developed plan that seemed plausible to the lawyer, and the lawyer expected the client to lose the legal battle and resort to the murder plan. Answer C rightly states the save-a-life exception set forth in 1.6(b)(1) but ignores the commonsense guidance provided in Comment 16 to Rule 1.6, which is that before using one of the exceptions and violating the client's trust, the lawyer should first try to persuade the client to do the right thing. Answer D incorrectly suggests that the client's plan is not related to the representation—in fact, the client has described it to the lawyer as her backup plan if the lawyer's advocacy does not prevail.

Attorney Adler represented a client, Carl, in a small lawsuit against the client's mechanic. One day, Carl asked the Adler for legal advice unrelated to the lawsuit. Carl explained that once a month, he and some friends play cards in Carl's home, and Carl wondered whether it was illegal for those playing to place small bets on the card games (the bets were typically less than fifty dollars). Attorney Adler knew of a penal code section in their state that prohibited gambling, and that a literal reading of the statute would include the type of betting that the client described. On the other hand, Attorney Adler knew that the authorities never prosecuted individuals for placing low-stakes bets on games played in private homes among friends, and that this policy of nonenforcement against games in private homes was a longstanding practice. Would it be proper for Attorney Adler to tell Carl about the nonenforcement policy and practice? (A) Yes, a lawyer may advise a client about enforcement policy in areas of doubtful legality so long as the lawyer does not knowingly counsel or assist the client to engage in criminal or fraudulent activity. (B) Yes, a lawyer's advice to a client about the risk of detection or prosecution for illegal conduct is proper regardless of the lawyer's intentions, so long as the lawyer does not actively participate in the criminal or fraudulent actions. (C) No, such advice is unrelated to the subject matter of the representation and is therefore outside the scope of Attorney Adler's authority under the circumstances. (D) No, because the lawyer's advice amounted to counseling or assisting a client in the commission of a crime or fraud.

(A) Yes, a lawyer may advise a client about enforcement policy in areas of doubtful legality so long as the lawyer does not knowingly counsel or assist the client to engage in criminal or fraudulent activity. Two separate provisions in the ABA Model Rules set the boundaries of what advice lawyers may properly give to their clients: Rule 1.2(d) and Rule 2.1. On the one hand, Rule 1.2(d) provides, "A lawyer shall not counsel a client to engage, or assist a client, in conduct that the lawyer knows is criminal or fraudulent, but a lawyer may discuss the legal consequences of any proposed course of conduct with a client and may counsel . . . [about the] application of the law." Answer A captures the essence of this Rule. Similarly, Rule 2.1 says, "In rendering advice, a lawyer may refer not only to law but to other considerations such as moral, economic, social and political factors, that may be relevant to the client's situation." A longstanding nonenforcement policy of local authorities falls under "other considerations" that supplement purely legal advice about the statutory language. The Restatement (Third) of the Law Governing Lawyers § 94 combines these provisions from the Model Rules into one section, and the facts in this question are similar to one of the examples provided in § 94 of the Restatement, which explains, "Clearly, such advice is permissible when the lawyer knows that nonenforcement amounts to effective abandonment of the prohibition and not simply temporary dereliction on the part of enforcing authorities or ignorance on their part of sufficient facts to bring an enforcement proceeding." Answer B is incorrect because the client's intentions to commit fraud or a crime, if known to the lawyer, can change the lawyer's duty in the situation. As the Restatement says in § 94, "A lawyer's advice to a client about the degree of risk that a law violation will be detected or prosecuted violates the rule . . . when the circumstances indicate that the lawyer thereby intended to counsel or assist the client's crime, fraud, or violation of a court order. The same section adds: "A lawyer's disciplinary liability . . . turns on client activity that the lawyer knows to be criminal or fraudulent or in violation of a court order." Answer C wrongly suggests that lawyers may not answer questions outside the scope of the immediate representation, as ABA Model Rule 2.1 clearly permits lawyers to do so. Answer D is tricky because the statement is technically true (lawyers cannot assist clients in the commission of crime or fraud), but it does not fit with the facts of the question, which put the client's activity outside the purview of the authorities' enforcement policies.

After law school, an attorney worked for two years as a judicial clerk for a federal district judge. A few months before her clerkship was to end, the attorney applied for positions at several law firms in the area and interviewed with them for a position as an associate. Each of these firms had matters pending on the docket of the judge for whom the attorney clerked. During the interviews, neither the attorney nor any of the interviewers brought up the issue of the firm's pending matters before the judge. The interviewers at each firm did ask the attorney general questions about how she liked her judicial clerkship experience. The attorney notified the judge that she was interviewing with these firms, and the judge did not object. The judge wrote a recommendation letter for the attorney, which the attorney submitted to each of the firms. Was it proper for the attorney to seek employment with firms that have pending matters before the judge for whom she clerks? (A) Yes, an attorney serving as a law clerk to a judge may negotiate for private employment in the manner described here (B) Yes, because the firms failed to disclose to her that they had pending matters before the judge for whom the attorney clerked (C) No, because the attorney failed to disclose that she knew the firms had matters pending before her judge and failed to tell the firms what she knew about their cases (D) No, because an attorney working for the government may not negotiate for private employment with any person who is involved as a party or as lawyer for a party in a matter in which the attorney is participating personally and substantially

(A) Yes, an attorney serving as a law clerk to a judge may negotiate for private employment in the manner described here The correct answer is A. Model Rule 1.11(d) generally prohibits lawyers employed by the government from seeking employment with "any person who is involved as a party or as lawyer for a party in a matter in which the lawyer is participating personally and substantially." However, the Rule carves out an exception for judicial clerks: "a lawyer serving as a law clerk to a judge, other adjudicative officer or arbitrator may negotiate for private employment as permitted by Rule 1.12(b) and subject to the conditions stated in Rule 1.12(b)." Judicial clerks normally have one- or two-year terms of employment, after which many switch to private sector jobs. Thus, judicial clerks often interview for positions while they are still clerking to secure future employment for when the clerkship ends. Answer A is therefore correct—the clerk in this case did nothing inappropriate. The judge was aware of the interviews and the clerk did not divulge any confidential information gleaned from the discussions in chambers. Answer B is incorrect because the rules for judicial clerks do not require any action on the part of the firms interviewing the clerk. Furthermore, a clerk would presumably know what firms had pending matters before that judge. Answer C is wrong because the rules do not require that the clerk disclose her knowledge of the pending matters to the firms with whom she interviews—in fact, it would be inappropriate to discuss any information from her work within the judge's chambers with the firms. Answer D, which states the general prohibition against government lawyers negotiating for employment with firms involved in the lawyer's own cases, is incorrect because it ignores the exception for judicial clerks.

Attorney Bates works as a public defender. Because the office is always under-funded, it cannot afford to hire enough staff attorneys to handle the workload. The current public defenders all carry an overload of cases. Attorney Bates feels that she is unable to provide full representation to each client, as she must conduct about seven plea-bargaining sessions for different clients per weekday. Usually she meets the clients for the first time about fifteen minutes before each plea bargain session. Each plea bargain takes about an hour, with short breaks in between. Attorney Bates strongly encourages nearly all of her clients to accept a plea bargain because taking one case to trial means that the public defender's office must turn away about two dozen indigent clients. Attorney Bates and her colleagues believe that it is better for defendants to have a little representation than none at all and that most defendants would lose at trial anyway. Does Attorney Bates have an ethical problem under the Rules of Professional Conduct? (A) Yes, because a lawyer must control her workload so that each matter can be handled competently (B) Yes, because it would be better for clients to have no lawyer at all than to rely upon a lawyer who is providing minimal or inadequate representation (C) No, because if most of the clients would, in fact, fare worse if they went to trial, then Attorney Bates's representation is their best option (D) No, because there is a special exception for public defenders in the Rules of Professional Conduct regarding diligence

(A) Yes, because a lawyer must control her workload so that each matter can be handled competently The correct answer is A. Diligent representation refers not only to the lawyer's actual efforts on the client's matter, but also on the lawyer's availability, attention, and so forth. An overload of clients or cases will necessarily mean that the lawyer will shortchange some or all of them—neglecting some matters for too long, missing crucial details because the lawyer is too rushed or distracted, missing deadlines, or cutting corners. Answer A is correct—it captures the guidance expressed in official Comment 2 to Model Rule 1.3. A lawyer's inability to control her own workload can subject the lawyer to disciplinary action. Answer B is incorrect but cites the typical rationale for taking on too many clients—the idea that clients are still better off with a distracted, overwhelmed lawyer than no lawyer at all. This may or may not be true, but the Model Rules do not excuse lawyers who cannot control their workload just because some clients might have to go unrepresented. Answer C is incorrect because the actual outcome of the clients' cases (and the likely outcome if the lawyer had declined representation) is not the factor used to evaluate the lawyer's diligence. Answer D is simply false—there is no special exception for public defenders, and many academic writers have noted the widespread problem in this area.

Attorney Morgan handled routine legal questions and small matters for a client, including contracts and leases, and obtained necessary licenses and permits for the client's business to operate. After a legal dispute arose between the client and an outside vendor, the client contacted Attorney Morgan seeking representation. Attorney Morgan explained that she is not a litigator and referred the client to a reputable trial lawyer in that locale. The trial lawyer met with the client and agreed to provide representation for a contingent fee. The matter settled quickly. The trial lawyer then sent Attorney Morgan a small share of the contingent fee received in the case. When Attorney Morgan originally referred the case to the trial lawyer, she did not check for any conflicts of interest because she did not do any litigation work and knew she would not participate in representing the client in the matter. For purposes of current or future conflicts of interest, and for potential malpractice liability in the litigation, did an attorney-client relationship exist between Attorney Morgan and the client for the litigation matter? (A) Yes, because a lawyer who refers a matter to another firm, and divides a fee from the matter with the other firm, has undertaken representation of the client (B) Yes, Attorney Morgan could be liable for malpractice if she referred the client to an incompetent trial lawyer, but there is no conflict of interest issue if Attorney Morgan did not do legal work on the matter (C) No, if Attorney Morgan did not perform any work on the matter or agree to represent the client in the litigation, no attorney-client relationship formed (D) No, because once another lawyer was representing the client in the matter, Attorney Morgan's representation of the client ended, as a client cannot simultaneously have two lawyers for the same litigation

(A) Yes, because a lawyer who refers a matter to another firm, and divides a fee from the matter with the other firm, has undertaken representation of the client The correct answer is A. This fact pattern represents a rather cutting-edge issue in legal ethics and was the subject of ABA Formal Opinion 16-474. Lawyers routinely refer clients to other firms, and in some cases, they receive a referral fee from the lawyer who takes the case. If a lawyer is receiving any fee whatsoever related to the provision of legal services to a client, the lawyer has triggered an attorney-client relationship for purposes of the ethical rules, even if the lawyer does not perform any legal work on the matter. Thus, the referral could still create conflicts of interest with other clients or potential legal malpractice liability. Depending on the circumstances, these concerns may be confined by limiting the scope of the lawyer's role in the representation. Answer A is correct because Attorney Morgan is receiving a fee for the referral, and the ABA has concluded that this is sufficient to find that some kind of representation is in place, even if it is limited in scope. Answer B is incorrect because it states that there is no conflict of interest issue; the ABA has opined that fee sharing does indeed trigger the conflicts of interest rules. Answer C is incorrect because the amount of work a lawyer performs on a case is not the sole factor in determining whether an attorney-client relationship exists. For example, receiving even a small fee is enough to create at least a limited relationship and attendant ethical obligations. Answer D is incorrect because the involvement of other lawyers does not automatically end the first lawyer's representation. In fact, it is fairly common for additional lawyers to be added to a case as it proceeds through litigation, appeals, and remand.

An attorney represented a seller in a business transaction involving industrial equipment. When the sale was complete, the purchaser sent the attorney a check for $7,000, the agreed-upon purchase price, with a letter directing the attorney to forward the money to his client (the seller). The attorney notified his client immediately that he had received the check. The client was traveling at the time and asked the attorney to hold the funds until he returned from his trip. The attorney had only recently moved to this jurisdiction and opened a new firm. Because the attorney did not yet have a trust account at a bank in the area, he deposited the check in the trust account in the neighboring state, where he had practiced until recently. He told the client that the funds would be in a client trust account at an out-of-state bank. The client consented. As soon as the check cleared, the attorney wrote the client a check for the full amount to be withdrawn from the out-of-state trust account. Did the attorney act properly in his handling of client funds? (A) Yes, because an attorney may deposit client funds in an out-of-state trust account if the client gives informed consent to this arrangement (B) Yes, because the client asked the attorney to hold the funds temporarily, and the attorney faithfully delivered the entire sum to the client (C) No, because the attorney should not have accepted the check at all, but should have instructed the purchaser to write the check out to the client himself and deliver it directly to the client (D) No, because client funds must be kept in a separate account maintained in the state where the attorney's office is situated, regardless of whether the client consents to such an arrangement

(A) Yes, because an attorney may deposit client funds in an out-of-state trust account if the client gives informed consent to this arrangement The correct answer is A. Model Rule 1.15(a) requires lawyers to deposit client funds in a trust account in the same state. This may present issues for firms with small satellite offices in multiple states. In this case, if the attorney had not first obtained the client's consent, it would have been an ethical violation to deposit the money in an out-of-state bank account, subjecting the attorney to disciplinary action. Fortunately, the attorney in this problem obtained the client's consent beforehand, making Answer A correct. Answer B is incorrect because holding the funds temporarily does not excuse lawyers from the specific requirements of Model Rule 1.15. In fact, lawyers usually hold client funds for only a short time. Answer C is incorrect because the attorney did not have an ethical duty to tell the seller to send the check directly to the client. Finally, Answer D is wrong because the Model Rules allow lawyers to deposit funds in an out-of-state trust account, provided that the client consents.

In a divorce proceeding, the husband retained Attorney Thompson to represent him. Several years later, the husband's ex-wife wanted to hire Attorney Thompson to represent her in a legal action against her ex-husband for unpaid child support. Attorney Thomas called the ex-husband, his former client, and discussed the conflict of interest with the ex-husband. The ex-husband gave consent over the phone to Attorney Thompson representing the ex-wife in the child support matter. The ex-wife also consented to the conflict. Attorney Thompson proceeded to file the child support enforcement case, which the ex-husband immediately settled, paying the full amount owed. Did Attorney Thompson violate the ethical rules regarding conflicts of interest? (A) Yes, because he did not first obtain informed consent, confirmed in writing, from his former client (B) Yes, because attorneys shall not represent persons whose interests would be materially adverse to those of a prior client (C) No, because attorneys are not prohibited from representing adverse parties, as long as both parties give informed consent (D) No, because attorneys may represent a person whose interests are materially adverse to those of a prior client, as long as the attorney advised the person seeking to retain the attorney and makes a reasonable effort to contact the prior client

(A) Yes, because he did not first obtain informed consent, confirmed in writing, from his former client The correct answer is A. The ABA Model Rules pertaining to conflicts of interest require that attorneys obtain informed consent from the affected clients, confirmed in writing. Oral consent by a client to the conflict of interest is insufficient—the consent must confirm the consent in a written document. Here, the lawyer merely obtained oral consent over the phone from his former client—the ex-husband whom he had represented in the original divorce. Thus, Answer A is correct—Attorney Thompson violated the ethical rules regarding conflicts of interest because the consent was not confirmed in writing. Answer B describes one of the basic elements that can constitute a conflict of interest—adverse interests between the clients—but is wrong because it leaves out the fact that clients may consent to many types of conflicts of interest. Answer C, on the other hand, incorrectly overlooks the writing requirement—informed consent is not enough. The lawyer may proceed with the representation only if the clients provide the consent in writing. Answer D is wrong because it is missing even a basic consent requirement—it is not enough for the lawyer to make an effort to obtain consent from former clients. The lawyer cannot proceed with the representation without obtaining informed consent, confirmed in writing, from each client.

A client who was a defendant in a high-profile murder case hired Attorney Finch to represent him. The client was unable to pay the attorney's fee, but he offered Attorney Finch the full media and literary rights to his story of the events and the trial, in exchange for representation. Attorney Finch agreed but included a provision in the retainer agreement stating that the attorney would not use any media or literary rights until the conclusion of the representation. The case proceeded to trial and eventually resulted in an acquittal for the client. Would Attorney Finch be subject to discipline for violating the conflict of interest rules? (A) Yes, attorneys shall not ever accept media and/or literary rights in exchange for services provided to a client. (B) Yes, attorneys cannot make an agreement with a client giving the lawyer media and/or literary rights in exchange for representation before the conclusion of the case. (C) No, attorneys may accept media and/or literary rights, as long as they do not use such rights until after the conclusion of the case. (D) No, attorneys are authorized to accept media and/or literary rights in exchange for services and may immediately use such rights if they are given in exchange for representation.

(B) Yes, attorneys cannot make an agreement with a client giving the lawyer media and/or literary rights in exchange for representation before the conclusion of the case. The correct answer is B. Model Rule 1.8(d) gives a bright-line rule for lawyers accepting literary or media rights in their clients' cases: "Prior to the conclusion of representation of a client, a lawyer shall not make or negotiate an agreement giving the lawyer literary or media rights to a portrayal or account based in substantial part on information relating to the representation." Answer B is therefore the correct answer to this question—Attorney Finch may not accept literary rights in the case at any time before or during the representation of the defendant, even if the client offers, and even if the lawyer agrees not to use the rights until later. The policy concern behind the rule is that the lawyer would then have a perverse incentive to do things in the representation that would make the literary rights more valuable, like discourage the client from accepting a plea bargain or other settlement offer (a trial is more appealing for a book or movie than a quick plea agreement), or making the trial unnecessarily sensational or melodramatic. Such actions by the lawyer would typically run counter to the client's best interest, but the lawyer is less likely to be objective when there is a personal financial interest at stake. Answer A ignores the fact that a lawyer may accept literary rights from a client in some circumstances—for example, it would be acceptable if the client offers the lawyer literary rights in lieu of legal fees in a work that was previously published (the subject is not about the lawyer's own representation of the client). In addition, a client might offer the lawyer literary rights after the representation has concluded, to avoid paying the lawyer's bills, and a lawyer could accept the offer at that time. Answer C is incorrect because the agreement to refrain from using the literary rights until the end of the case does not legitimize the lawyer's behavior or resolve the ethical problem. As stated above, even if a lawyer would not exercise the rights until later, the lawyer might still have an incentive to avoid quiet settlements or plea deals and to make the trial as dramatic as possible. Answer D states the opposite of the rule and is therefore wrong—Model Rule 1.8(d) prohibits lawyers from accepting literary rights in the representation before the representation concludes.

An attorney represented a defendant who faced charges of assault causing bodily injury. The client recounted the events that led to the charges, and the attorney concluded that the client did not commit the assault. The attorney assured the client that he defeated such charges against other clients in the past. The client asked the attorney to get him the best possible plea deal and explained that he did not want to take the case to trial. The attorney contacted the district attorney, who offered a reasonable plea deal for attorney's client, but it included some jail time. The attorney immediately refused the offer and told the client that they would be going to trial because the district attorney did not offer a reasonable plea. Is the attorney subject to discipline? (A) Yes, because lawyers must abide by client decisions regarding plea deals, regardless of the lawyer's opinion about the client's guilt or innocence (B) Yes, because lawyers are not authorized to give advice based on their own opinions about the case (C) No, because a lawyer shall not accept a plea deal in a criminal case if the lawyer believes the client is innocent (D) No, because lawyers are impliedly authorized to refuse plea deals if they do not find them acceptable

(A) Yes, because lawyers must abide by client decisions regarding plea deals, regardless of the lawyer's opinion about the client's guilt or innocence The correct answer is A. The attorney in this question committed two violations. First, he did not consult with the client before rejecting a proffered plea agreement. Second, he disregarded his client's express wish to avoid trial. Answer A is the best answer because it correctly states that lawyers must do what their clients want about plea bargains. Such decisions are entirely up to the client, not the lawyer, although lawyers are free to offer advice (even urging a client to accept or reject an offer). Answer B is incorrect because lawyers are, in fact, supposed to offer advice and opinions to their clients about the case, even though they must abide by the client's decision regarding a plea deal. Answer C wrongly states that lawyers must reject plea deals if they believe their client is in fact innocent. Many innocent clients would be better off accepting a plea bargain with limited punishment rather than risking a hefty sentence if they lose at trial, and unfortunately, sometimes convictions result for innocent defendants because the circumstantial evidence is so incriminating. In addition, defendants in the federal system can enter an "Alford Plea" in which they maintain their innocence but accept the plea deal anyway. Answer D incorrectly suggests that defense lawyers always have permission to reject seemingly unreasonable plea offers, but the Model Rules state the exact opposite—lawyers must always present the offer to a client, unless the client has explicitly instructed the lawyer beforehand to reject or accept particular offers.

Dakota, a new law school graduate, recently moved to a new town and opened a practice there. A prospective client met with Attorney Dakota to discuss representation in a personal injury lawsuit against the town's largest company and major employer. The municipal government and local hospital would be added as co-defendants. The prospective client explained that she had already met with every other plaintiff's firm in the town but that each one declined her case because it presented conflicts of interest for them. Dakota quickly realized that he was likely the only lawyer in town who could represent this client in the matter, having no conflicts of interest with other clients, due to the firm being a new practice. Seeing an opportunity, Attorney Dakota offered to represent this client for four times the usual fee that was customary in the locality for regular legal services. In addition, Dakota explained that the client would have to pay a large non-refundable retainer sum up front, against which the attorney would draw fees. Finally, the client would have to reimburse Dakota for every penny of actual expenses and costs incurred in-house, such as photocopies, plus any costs and expenses incurred from outside services providers, such as court reporters or experts. The client was astonished at the exorbitant fees, but she reluctantly agreed because she felt she had no real choice. Dakota did memorialize their agreement in writing. Is Attorney Dakota subject to discipline for this fee agreement? (A) Yes, because the fee is unreasonably high and Dakota is merely exploiting the client's predicament (B) Yes, because Dakota is forcing the client to reimburse him for in-house expenses, such as photocopy charges (C) No, as long as the fee customarily charged for legal services in that locale is unusually low compared to other parts of the country (D) No, because of Dakota's experience, reputation, and ability

(A) Yes, because the fee is unreasonably high and Dakota is merely exploiting the client's predicament The correct answer is A. Reasonable fees are a requirement for lawyers. Even though a lawyer may charge a higher-than-usual fee for cases that are particularly difficult or time-consuming, or that draw on the lawyer's special expertise, charging a much higher fee merely because the client has no alternative for obtaining representation is unreasonable. This is the equivalent to unethical price-gouging in industries. Even though "reasonable" is a relatively vague term, Model Rule 1.5 provides a list of several factors to consider in determining whether a fee is reasonable—the client being "stuck" with this lawyer is not a factor that makes a fee reasonable. Answer A, therefore, best reflects the Model Rules and the official Comment to the rules about fees. Answer B correctly states that the lawyer would be subject to discipline but gives an incorrect reason—lawyers may seek reimbursement from clients for actual expenses incurred on the client's behalf if the charges are reasonable and the lawyer communicates them to the client at the outset of representation. Answer C is incorrect because the prevailing rate rule (often called the "lodestar rate") focuses on the average or typical fees locally, not nationally. Answer D states one of the valid factors for assessing the reasonableness of a fee, but based on these facts, this factor is inapplicable. Attorney Dakota is relatively inexperienced and has not been practicing long enough to have a reputation (or the expertise) that could justify such an exorbitant fee.

Attorney Barlow was a litigator, but he mostly handled divorces and child custody disputes. On one occasion, a prospective client indicated some hesitation about hiring Attorney Barlow to represent her in her divorce and custody case, due to his high fees. Desperate to garner a new client, Attorney Barlow offered to represent her on a contingent fee basis. The client declined this offer, because she planned to settle the matter as quickly as possible, and she expected that paying fees on an hourly rate would end up being less expensive. The case did not settle quickly, however, and the client soon fired Attorney Barlow and hired another lawyer for the trial. She mentioned to her appellate lawyer that her first attorney had offered to represent her on a contingent fee basis, and now that the matter was dragging on and on, she regretted that she did not agree to that fee structure. Her new lawyer knew that contingent fees in a divorce or custody case would constitute a violation of the Rules of Professional Conduct and reported Attorney Barlow to the state disciplinary authority. When disciplinary proceedings commenced, the Attorney Barlow maintained that he had not actually violated the Rules of Professional Conduct, because the client had brushed off his offer and insisted on paying an hourly fee. Even if he had attempted to violate the Rules, he argued, the client thwarted his attempt, and therefore no actual violation occurred. Is Attorney Barlow subject to discipline merely for attempting to violate an ethical rule, if the attempt never came to fruition? (A) Yes, even an attempt to violate the Rules of Professional Conduct would constitute professional misconduct. (B) Yes, because his sole motivation in offering to represent someone on a contingent fee basis was to overcome the potential client's hesitation about hiring him in the first place. (C) No, because his request merely constituted an attempt to violate the Rules of Professional Conduct, and the Rules do not impose discipline for attempts or inchoate violations. (D) No, because the client refused his offer, and now regrets her decision, so the disciplinary action is not ripe.

(A) Yes, even an attempt to violate the Rules of Professional Conduct would constitute professional misconduct. Answer A is the best answer. ABA Model Rule 8.4(a) says that it is professional misconduct for a lawyer even to attempt to violate the Rules of Professional Conduct. Rule 1.5(d) expressly forbids contingent fees in any domestic relations matter, such as divorce or a child custody dispute. Attorney Barlow offered to violate this rule for the prospective client—even though the client refused, this was an attempted violation, for which he could be subject to discipline. Answer B is incorrect; the lawyer's motivation in violating a rule normally is immaterial to the analysis of whether a violation occurred, especially with bright-line rules like Rule 1.5(d). Answer C wrongly suggests that mere attempts cannot be subject to disciplinary action under the Model Rules, but Rule 8.4(a) states that an attempt to violate Model Rules would constitute professional misconduct. Finally, Answer D is incorrect, because ripeness is not relevant in the content of disciplinary actions for Rule 8.4(a) if a lawyer has already attempted to violate one of the other Model Rules.

A lawyer helps pro se litigants in divorce and custody matters by assisting them in completing self-help forms (court filings) for a nominal fee, giving advice, and reviewing completed forms before the litigants file the forms. To avoid misunderstandings, the lawyer first requires each pro se litigant to sign a written disclaimer providing that no attorney-client relationship exists: "I understand that Attorney ____ has no legal or ethical obligation to provide legal representation to me in this matter." Does an attorney-client relationship exist in these circumstances, even though the pro se litigant signed a form acknowledging that no legal representation will follow? (A) Yes, even if the representation is limited in scope, the lawyer's review of court documents and provision of legal advice about pending legal proceedings constitutes the practice of law by him (B) Yes, because most of the pro se litigants are unsophisticated users of legal services and probably do not understand the significance of the written disclaimer (C) No, because the individual signed an express written disclaimer as to the existence of any attorney-client relationship (D) No, because the lawyer is merely reviewing forms completed by the pro se litigants themselves, which they will file in court on their own behalf

(A) Yes, even if the representation is limited in scope, the lawyer's review of court documents and provision of legal advice about pending legal proceedings constitutes the practice of law by him The correct answer is A. Sometimes it is very clear that no attorney-client relationship exists, but there are also confusing situations, as when a lawyer gives an oral or written disclaimer stating there is no such relationship but then provides legal representation anyway. The ABA addressed this situation in a 2015 Formal Opinion (state ethics opinions, such as Texas Ethics Opinion 635, reach a similar conclusion), finding that a lawyer's services, which were similar to those presented here, constituted the practice of law. Therefore, an attorney-client relationship exists, even if it is limited in scope. The significance of this conclusion is that the lawyer could still be subject to discipline for ethical violations related to the provision of these services, and the limited representation still "counts" for purposes of checking for conflicts of interest. Answer B is incorrect, although a prospective client's legal experience can be relevant in determining whether it was reasonable to rely upon the lawyer for representation. If the lawyer is actually practicing law and providing legal services or advice to the individual, this outweighs whether the client understood the written disclaimer. Answer C is incorrect because an attorney-client relationship can exist even where there is an express disclaimer of such a relationship—otherwise, a lawyer could simply have every client sign a disclaimer in order to avoid potential malpractice liability or duties under the conflict of interest rules and then provide representation anyway. Answer D is incorrect because reviewing documents and advising individuals about the legal significance of their statements constitutes the practice of law, albeit limited in scope. In short, the provision of any legal services creates an attorney-client relationship for purposes of the ethical rules.

Perry Mason was a criminal defense attorney. At one point, Mason represented a client facing drug charges. The client was also an undocumented immigrant—he had entered the country many years before on a tourist visa, and then overstayed his visa, never returning to his home country. The prosecutor offered the client a plea agreement that included minimal time in prison. Attorney Mason was pleasantly surprised by the plea offer, given the seriousness of the drug charges against the defendant and the fact that the charges normally carried a ten-year sentence. Mason recognized this as a much better outcome than the defendant was likely to achieve if they went to trial, so he urged his client to accept the plea agreement. The client then explained his immigration status to Mason and asked if accepting the plea bargain could result in his deportation. Immigration law was not Mason's area of specialty, but he speculated that the plea agreement would have no implications for the client's immigration status, and Mason assured him that his long tenure in the country meant that he would not face deportation. Attorney Mason was correct about their poor odds of prevailing at trial, but he was completely wrong about this; in fact, pleading guilty to these charges resulted in deportation for the client, as a matter of statutory law, after he served his sentence. The client followed the advice and faced deportation. Is the defendant likely to prevail on a claim based on ineffective assistance of counsel? (A) Yes, failing to advise a client that deportation would result from pleading guilty to the charges constitutes ineffective assistance of counsel for Sixth Amendment purposes. (B) Yes, because urging one's client to accept a plea agreement that results in guaranteed prison time constitutes ineffective assistance of counsel for Sixth Amendment purposes. (C) No, because the client accepted the plea agreement voluntarily, and some defendants would be willing to accept deportation in exchange for a significantly shorter prison sentence. (D) No, because lawyers do not have to be knowledgeable about every area of law in order to provide representation to a client, as long as the lawyer is competent in the subject area of the representation, such as criminal defense work.

(A) Yes, failing to advise a client that deportation would result from pleading guilty to the charges constitutes ineffective assistance of counsel for Sixth Amendment purposes. Answer A is correct. In a recent line of cases, the U.S. Supreme Court has held that failing to advise a client that nearly certain deportation will result from a guilty plea constitutes ineffective assistance of counsel. See, e.g., Lee v. U.S., 137 S. Ct. 1958 (2017); Padilla v. Kentucky, 559 U.S. 356 (2010). Note that the defendant in such a case is not challenging his conviction or sentence, but instead is trying to revoke his guilty plea and waiver of trial, due to the severe collateral consequences; this makes the two-part Strickland test applicable mostly by analogy. Answer B is incorrect because lawyers may, and usually do, counsel their clients to accept favorable plea agreements, and doing so does not constitute ineffective assistance of counsel most of the time. Answer Cwrongly suggests that the apparent voluntariness of the guilty plea precludes an ineffective assistance of counsel claim on appeal. Instead, the Supreme Court has held that defendants cannot enter an informed, rational, voluntary guilty plea if their lawyer misinformed them—or even inadequately informed them—of the direct and collateral consequences of such a decision. Answer D is partly correct and partly incorrect—even though lawyers do not have to have specialized knowledge about every area of law, a minimally competent lawyer should be able to check for the correct answer when the client asks a question with such far-reaching consequences or the lawyer should consult immigration counsel. Most public defender offices and legal aid recourse centers now have immigration counsel who can be consulted.

A nonprofit corporation worked to preserve the environment. The nonprofit corporation retained an attorney to sue Conglomerate Corporation for violating pollution regulations. The attorney prevailed in the case, winning an injunction against Conglomerate, as well as the payment of several millions of dollars into a fund for cleanup of polluted areas. The court also ordered Conglomerate to pay attorney fees to the prevailing party in the case. The attorney had previously agreed to share the fees with the nonprofit corporation, with 85% of the fees going to the nonprofit corporation. Is it proper for the attorney to share the fees with the nonprofit corporation? (A) Yes, sharing court-awarded fees with a nonprofit entity is proper when the nonprofit itself employed or retained the lawyer in the matter. (B) Yes, the fee-sharing arrangement is proper so long as the attorney's share is the same as if he charged the nonprofit corporation a reasonable hourly rate for the time spent on the matter. (C) No, the fee-sharing arrangement is improper because the attorney gave most of the money to the nonprofit corporation, rather than splitting it evenly. (D) No, the fee-sharing arrangement is improper because the nonprofit corporation is not a law firm or owned by lawyers, so this action constitutes sharing legal fees with nonlawyers.

(A) Yes, sharing court-awarded fees with a nonprofit entity is proper when the nonprofit itself employed or retained the lawyer in the matter. Answer A is correct. ABA Model Rule 5.4(a) prohibits fee-sharing with nonlawyers, with four narrow exceptions: contracts for payments of death benefits to a law firm partner's spouse or estate, purchasing a law firm pursuant to Rule 1.17, inclusion of nonlawyer employees in compensation or retirement plans, or sharing court-awarded fees with a nonprofit that the lawyer represented (the situation here). Answer B is incorrect, because a lawyer is free to give most or all the fees to the nonprofit, even if that means that the lawyer received less than his normal hourly rate. Note, however, that the lawyer could not collect unreasonably high fees from a nonprofit under 5.4 (the "reasonableness" requirement of Rule 1.5 would still apply). Answer C is incorrect because Model Rule 5.4 does not require lawyers to share fees with a nonprofit evenly, so the 85-15 ratio here is permissible. Finally, Answer D is incorrect as the nonprofit corporation falls under the exception in Rule 5.4(a)(4), although normally it would be impermissible for a lawyer to share legal fees with a for-profit corporation.

A client paid his legal fees to his attorney in cash. The total fees were $11,100, and the client paid the attorney in bundles of twenty-dollar bills. The Internal Revenue Code 26 U.S.C. § 6050 requires that lawyers disclose, through Form 8300, the identities of clients, amounts, and payment dates of all cash fees in excess of $10,000. The client had forbidden the attorney to disclose the information to the IRS. Must the attorney disclose on the Form 8300 the client's name, the amount, and the dates of payment? (A) Yes, the Internal Revenue Code supersedes the Rules of Professional Conduct regarding the duty of confidentiality, so the lawyer should make such disclosures as are necessary to comply with the law, after informing the client. (B) Yes, because payment of the fee is normally not confidential client information and normally could not be prejudicial to the interests of the client in the representation. (C) No, a lawyer must comply with the client's express wishes regarding the disclosure, as the punishment for failing to file Form 8300 will probably fall on the client, not the lawyer. (D) No, the Rules of Professional Conduct permit but do not require disclosure to comply with other law, so the attorney may file Form 8300, but it is not correct to say attorney "must" do so.

(A) Yes, the Internal Revenue Code supersedes the Rules of Professional Conduct regarding the duty of confidentiality, so the lawyer should make such disclosures as are necessary to comply with the law, after informing the client. The correct answer is A. Rule 1.6(b)(6) provides another of the narrow exceptions to the confidentiality rule—lawyers can make disclosures of otherwise confidential information to comply "with other law," which includes state and federal statutes as well as regulatory requirements. Comment 12 to Model Rule 1.6 adds that other laws may supersede the ethical requirements of Rule 1.6, and the tax code provision in this problem explicitly requires lawyers to make the delineated disclosures. The federal tax code supersedes Rule 1.6, so the lawyer must make the disclosures or else withdraw from the representation before filing anything. Answer A is therefore correct—the lawyer will have to make the disclosures over the client's objections and could face sanctions for failing to do so. The lawyer would not be violating Rule 1.6 in making these disclosures, as they are required by law. Answer B incorrectly suggests that client fee payments are not confidential information—in fact, they are, and lawyers would normally need client authorization to disclose any information about payment of legal fees. Answer C is wrong because it ignores the exception in Rule 1.6(b)(6) and also the fact that the regulation imposes this requirement on the lawyers, even if clients will bear the primary responsibility for tax fraud. Finally, Answer D incorrectly states that complying with other law is optional for lawyers—Comment 12 indicates that when other law supersedes Rule 1.6, as is the case here, the lawyer should comply with the law.

Two business partners had a bitter dispute that led to the dissolution of their partnership and eventually to litigation over the assets and obligations of the partnership. One of the partners hired Attorney Grover to represent him in the litigation. During a pre-trial deposition of Attorney Grover's client, the client gave answers that Attorney Grover knew were false. Attorney Grover said nothing and allowed his client to continue giving false testimony. Later, at trial, the opposing party submitted evidence that proved conclusively that Attorney Grover's client had lied during his deposition, and it was an easy inference that Attorney Grover must have known that his client's statements were untruthful, given the circumstances. The judge filed a grievance against Attorney Grover for allowing his client to give false testimony and failing to rectify it for the court. Attorney Grover objects that his alleged ethical violation took place during a deposition, not at trial, so the duty of candor to the tribunal did not apply, as the testimony did not take place before the tribunal. Could the disciplinary authority reprimand Attorney Grover even though the incident occurred outside of court, at a pre-trial deposition? (A) Yes, the ethical duty to take remedial measures when a client offers false statements applies in depositions as well (B) Yes, unless it appears that opposing counsel already knows that the statements are false and is planning to impeach the witness (C) No, because the client was testifying in a deposition, which is merely an ancillary proceeding to a trial, rather than committing perjury during the trial itself (D) No, because a lawyer has a duty to protect client confidentiality and a duty of loyalty to the client that prohibits such a disclosure

(A) Yes, the ethical duty to take remedial measures when a client offers false statements applies in depositions as well The correct answer is A. Comment 1 to Model Rule 3.3 says that the Rule applies even in situations that are "ancillary proceedings conducted pursuant to the tribunal's adjudicative authority, such as a deposition." Thus, a lawyer cannot allow his own witnesses to submit false testimony in a deposition. Note that witnesses testifying in depositions are put under oath, and that the deposition often becomes part of the trial record in the proceedings. Answer A is correct as it accurately captures the provision in Comment 1 about ancillary proceedings. Attorney Grover could be subject to discipline for violating the duty of candor to the tribunal. Answer B is incorrect because the opposing counsel's awareness of the falsity of the evidence submitted (a fact that is not even part of the question) does not change the lawyer's duty of candor to the tribunal. The lawyer has a duty to take remedial measures to correct the false statements, even if it appears that the other party knows the statements were false. Answer C incorrectly states that the duty of candor to the tribunal does not apply in depositions. The duty extends to all ancillary proceedings if they are conducted pursuant to the tribunal's adjudicative authority. Answer D wrongly states that the duty to protect the client's confidentiality outweighs the duty of candor to the tribunal—the reverse is true. The duty of candor to the tribunal requires disclosures that would otherwise be impermissible under the rule of client confidentiality.

Two co-plaintiffs agree to representation by the same lawyer in a civil action, and both provide written informed consent to the potential conflicts of interest. Halfway through trial, the opposing party unexpectedly makes a settlement offer that one client wants to accept. The other client wants to reject it, as he believes that a favorable jury verdict is a certainty and will be much higher. The lawyer strongly encourages them to accept the settlement offer, which he believes is generous, given his perception that their chances for a favorable jury verdict are slim. Unable to reach a consensus on whether to accept the settlement offer, one of the clients revokes his consent to the conflict of interest. Assuming there are no other special circumstances in this situation and that the court would not prohibit withdrawal, must the lawyer seek to withdraw from representation? (A) Yes, the lawyer must petition the court to withdraw from representing both clients. (B) Yes, the lawyer must withdraw from representing the client who did not revoke consent and continue to represent the client who revoked consent. (C) No, given that both clients consented in writing at the outset of representation, and that the trial is already underway, the lawyer can and must continue to represent both clients. (D) No, the lawyer can continue to represent the other client who did not revoke consent, even if the revoking client terminates the representation and hires another lawyer.

(A) Yes, the lawyer must petition the court to withdraw from representing both clients. The correct answer is A. Clients who want joint representation will usually give written consent to the potential conflicts of interest because sharing a lawyer may save money and can simplify the proceedings. Even so, each client who gives consent to a conflict of interest can revoke that consent at any time, which means the lawyer will have to withdraw from the representation of at least one, if not all, of the clients. Once a trial is underway, the lawyer must ask the court for permission to withdraw, but courts almost always grant the request. In this question, Answer A is the best answer—the lawyer will presumably have to withdraw from representing both clients. Sometimes, a lawyer may be able to withdraw from one and continue representing the other, but given the late stage of the proceedings here and the fact that one client's decision puts the other in an unfavorable position, the lawyer probably cannot represent either. Answer B, in contrast, is not the best answer under these facts where the trial is well underway and the clients are in disagreement as to how to proceed—although the lawyer might have been able to continue representing the other client, if the first client had revoked consent very early in the representation and very little confidential information had been shared that could be prejudicial to either client. Answer C wrongly overlooks the fact that clients can revoke consent at any time in the representation, at which point the conflict of interest would preclude the lawyer from representing both clients. Answer D has the same problem as Answer B, although it switches the clients—given the stage of the proceedings and how far the representation has already gone, and how much confidential information the lawyer must have by this point, the lawyer will have to withdraw from representing both clients.

The directors of a large corporation requested that its lawyer investigate allegations of wrongdoing. During that investigation, the lawyer conducted interviews of the organization's employees and managers. The interviews revealed that the alleged wrongdoing involved only one employee, whom the corporation immediately terminated. After resolving the matter, the lawyer authored a series of blog posts about the amusing anecdotes he heard during his interviews, illustrating the petty internal politics, the low morale in certain departments, and a few of the office romances that had ended badly. Did the lawyer violate his ethical duties to his client? (A) Yes, when a corporation asks its lawyer to investigate an internal problem or wrongdoing, interviews made during that investigation between the lawyer and the client's employees come under the client-confidentiality rules (B) Yes, because when a lawyer represents an organization, every constituent of that organization is a client of the lawyer, and he owes a duty of confidentiality to each one (C) No, because the employees were not the lawyer's clients, if he represented only the organization, and the disclosed information was personal information, not institutional information (D) No, because after the representation ended, the lawyer no longer had a duty of confidentiality to anyone in the organization

(A) Yes, when a corporation asks its lawyer to investigate an internal problem or wrongdoing, interviews made during that investigation between the lawyer and the client's employees come under the client-confidentiality rules The correct answer is A. Comment 2 to Rule 1.13 explains that whenever "an organizational client requests its lawyer to investigate allegations of wrongdoing, interviews made in the course of that investigation between the lawyer and the client's employees or other constituents are covered by Rule 1.6." That means that the duty of confidentiality applies to corporate clients just as it would to individual clients, and the lawyer may not disclose confidential information about clients, even after the matter ends, without client consent or some other exception that would allow the disclosure. The best answer to this question, therefore, is Answer A—the lawyer violated the duty of confidentiality by sharing the embarrassing information about employees of the corporation. Answer B is wrong because it asserts that each of these individual employees is a client of the lawyer, which is not the case. In fact, Comment 2 goes on to say, "This does not mean, however, that constituents of an organizational client are the clients of the lawyer." The lawyer has a duty of confidentiality, but it is not to these individuals, but rather to the corporation itself. Answer C, on the other hand, observes correctly that the employees themselves were not clients of the lawyer, but this answer wrongly suggests that the information therefore did not come under the confidentiality rule. The disclosures would be embarrassing to most corporations, and the lawyer's duty to the corporate client meant that the information should have remained confidential. Answer D incorrectly suggests that the duty of confidentiality ends when the representation ends, which is not the case—the lawyer's duty of confidentiality continues indefinitely (perpetually), even after the representation has concluded.

Twenty years ago Attorney Abrams represented Husband in a transactional matter that involved incorporating a small business that Husband later sold. The proceeds from the sale went to fund college tuition for Husband's grown children. Wife recently retained Attorney Abrams to represent her in a divorce action against Husband. Husband and Wife both provided Attorney Abrams with informed consent, confirmed in writing, waiving any conflicts of interest. Husband barely remembered Abrams. In fact, the previous representation has no bearing on the current divorce or marital property. Wife then told Abrams, in confidence, that Husband had an extramarital affair the previous year, and this was a major factor in her deciding to file for divorce. She explained that it was very important to her that Husband's affair be a centerpiece in the divorce proceedings, as it would give her closure and help both her parents and her grown children to understand why she felt compelled to end the marriage. Attorney Abrams believed her, but found this allegation unsavory and did not want to embarrass Husband, especially given that Husband is a former client. Abrams halfheartedly filed the divorce petition without any mention of the affair, stipulated to Husband's request for a sealed record without discussing this move with Wife, and resolved the matter as discreetly as possible. Ultimately, Wife accepted the settlement recommended by Attorney Abrams but was deeply disappointed that the affair had been kept secret, with the result that her family would not believe her about the reasons she sought a divorce. Is Attorney Abrams's conduct proper? (A) No, because Wife was disappointed at the end of the case, though she consented to the final settlement (B) No, because a lawyer must act with commitment and dedication to the interests of the client and with zeal in advocacy upon the client's behalf (C) Yes, because a lawyer has an ethical duty to preserve the privacy of opposing parties (D) Yes, because a lawyer is not bound to press for every advantage that might be realized for a client

(B) No, because a lawyer must act with commitment and dedication to the interests of the client and with zeal in advocacy upon the client's behalf The correct answer is B. The attorney in this case ignored his client's stated goals in the representation. The client wanted a divorce, but she also wanted her family members to know the reason she was seeking a divorce. Answer B reflects the rule that Attorney Abrams violated in this case—he did not act with dedication to Wife's interests or provide her with zealous advocacy. Answer A is incorrect because a lawyer's diligence is not measured by the client's satisfaction or disappointment at the end of the representation. Client satisfaction might have nothing to do with whether the lawyer complied with the ethical rules. Answer C is incorrect because, even though attorneys are indeed supposed to treat other parties as respectfully as possible, in this case (a divorce) it was appropriate to bring up Husband's marital infidelity. Answer D correctly restates part of the Comment to Rule 1.3, but this hypothetical was not about the lawyer pressing for every conceivable advantage. Rather, addressing the adultery was an important goal that the client communicated to the attorney. The attorney's failure to diligently pursue the client's goals also points to a conflict of interest. Although the previous representation of the husband on the incorporation matter was unrelated to the current representation of the wife in the divorce, the attorney still faced a concurrent conflict of interest. Under the pertinent part of Model Rule 1.7(a), a lawyer shall not represent a client if there is a significant risk that the representation of one or more clients will be materially limited by the lawyer's responsibility to a former client. Notwithstanding such a conflict, a lawyer may represent a client if the lawyer reasonably believes that the lawyer will be able to provide competent and diligent representation of each affected client, and the affected client gives informed consent, confirmed in writing. Although Attorney Abrams obtained consent, there are two problems under these facts. First, there is a question as to whether he could "reasonably believe that he would provide competent and diligent representation," given that Abrams knew that he intended to not disclose the adultery, a goal important to the client. Second, there is a question as to whether Wife's consent to the settlement was informed because the facts do not state that the attorney disclosed his intention not to disclose the divorce.

Attorney Anderson had recently taken on many new clients and was having trouble managing her time and meeting deadlines. She had not missed any deadlines in any client matter however. At one pre-trial hearing, opposing counsel asked for a one-month postponement of the previously scheduled hearing, to Attorney Anderson's great relief. Anderson readily agreed because the postponement would enable her to attend to other urgent client matters and give her more time to prepare for the hearing. Based on facts, there was no indication that postponing the hearing would prejudice the client. Anderson called her client to notify him about the trial postponement. The client was upset about the postponement because he wanted the matter resolved as quickly as possible. The client accused Attorney Anderson of putting her own scheduling needs ahead of his interests. Anderson explained that they had not lost anything through the postponement and that she would now have more time to prepare for the hearing. She suggested that the delay was probably advantageous for the client's case, both from a strategic and a preparation standpoint. The client accepted Attorney Anderson's answer but still felt disappointed that Anderson did not ask him first because he would have expressed his will to oppose the postponement. Was Attorney Anderson in compliance with her ethical duties under the Rules of Professional Conduct? (A) Yes, because a lawyer has an ethical duty to accommodate opposing counsel's request for a postponement or continuance, as long as the delay will not prejudice the client in the final outcome (B) Yes, a lawyer's duty to act with reasonable promptness does not preclude the lawyer from agreeing to a reasonable request for a postponement that will not prejudice the lawyer's client (C) No, because Attorney Anderson failed to ask the client about the postponement before agreeing to it in the courtroom (D) No, because any postponement without client consent may cause a client anxiety and undermine confidence in the lawyer's trustworthiness

(B) Yes, a lawyer's duty to act with reasonable promptness does not preclude the lawyer from agreeing to a reasonable request for a postponement that will not prejudice the lawyer's client The correct answer is B. The ethical rules require that lawyers act with reasonable promptness and avoid creating unnecessary delays, especially in litigation. Sometimes lawyers postpone and procrastinate simply because they have taken on too many cases, and sometimes lawyers intentionally cause delays merely to gain some strategic advantage (or to put another party in a difficult position, as when key witnesses eventually become unavailable). The Model Rules contain an explicit exception, however, for lawyers agreeing to accommodate requests for postponement (sometimes called a continuance) sought by another party. Such requests (due to unavoidable scheduling problems) are very routine and entirely reasonable. Answer B correctly states the sentiment expressed in Comment 3 to Model Rule 1.3 The postponement in this question was reasonable and a lawyer could normally agree to it without checking first with the client. Answer A is incorrect because the lawyer does not have any duty to accommodate opposing counsel's request, though it is customary etiquette to do so. The lawyer may accommodate such requests but does not have a duty to do so. Answer C is incorrect because such requests often need an immediate answer in the courtroom and do not change the client's situation in any substantial way. Barring unusual circumstances, a lawyer can agree to a routine postponement without checking first with a client. Answer D is incorrect, even though it borrows language from one of the other Comments to Rule 1.3. While it is true that unreasonable delays may upset the client, the delay in this question was reasonable and falls under the exception in Comment 3.

A criminal defense attorney represented a defendant who was facing charges for armed robbery. While preparing for trial, the defendant told the attorney that he intended to lie on the witness stand. The attorney tried to dissuade him, explaining that committing perjury could subject the client to additional criminal changes, and that the prosecuting attorney was likely to expose the client's untruthfulness during cross-examination. The attorney must either disclose the contemplated perjury to the tribunal or should ask the court for a recess to counsel and remonstrate with the client. Should the attorney allow the client to testify under these circumstances? (A) No, the attorney must withdraw from representation before the testimony occurs. (B) No, the attorney must either disclose the contemplated perjury to the tribunal, or should ask the court for a recess to counsel and remonstrate with the client. (C) Yes, because the attorney fulfilled his ethical duty by trying to dissuade his client from perjury, and the opposing lawyer will have an opportunity to cross-examine the client to catch him in his lies. (D) Yes, as long as the untruthful testimony is not material to the case and is unlikely to affect the outcome of the litigation.

(B) No, the attorney must either disclose the contemplated perjury to the tribunal, or should ask the court for a recess to counsel and remonstrate with the client. The correct answer is B. Model Rule 3.3(b) prohibits lawyers from offering evidence they know to be false, including the testimony of a client. A lawyer cannot call a witness to testify, or conduct the direct examination of a witness at trial, knowing that the testimony offered is false. Criminal cases present a difficult situation because defendants have a constitutional right to testify at their own trials. Answer B is the best answer to this question—either the lawyer will have to breach the duty of client confidentiality and reveal to the court that the client's testimony is a fabrication (the duty of candor outweighs the duty of confidentiality), or, if the local rules permit it, the attorney may ask the court to have the client testify in a narrative mode without the lawyer's participation in the testimony. Answer A is not the best answer—although the lawyer might have the option of withdrawal, doing so during trial is usually prejudicial to the interests of the client and disruptive to the proceedings so this should not be the lawyer's first choice. Answer C is incorrect because it is not enough for the lawyer merely to try to persuade the client to tell the truth—the lawyer cannot submit evidence knowing that it is untruthful. Answer D is wrong because Rule 3.3 does not apply only to material evidence; lawyers cannot offer any evidence at trial that the lawyer knows to be false.

Attorney Breyer has been representing a client in an adoption of an orphan from Zimbabwe. Acme Corporation subsequently hires Attorney Breyer to defend it against a defective products lawsuit brought by Victim. During the discovery phase of litigation, Victim's lawyers disclose the list of witnesses they plan to call at trial to testify on behalf of Victim against Acme. Attorney Breyer's adoption client is on Victim's witness list. The adoption client is unaware that Attorney Breyer is representing Acme Corporation. The adoption client, who is a friend of Victim, will testify about Victim's character traits of caution and care and the suffering Victim has endured since the incident with Acme's defective product. Attorney Breyer was not aware that the adoption client even knew Victim, and therefore had no previous reason to obtain consent from the adoption client, Acme Corporation, or Victim about his representation of Acme in the litigation. Will the court grant a motion to disqualify Attorney Breyer from representing Acme Corporation in the products liability case? (A) Yes, because a lawyer who handles adoptions is probably not competent to represent a corporation in litigation over defective products (B) Yes, a directly adverse conflict may arise when a lawyer is required to cross-examine a client who appears as a witness in a lawsuit involving another client of the lawyer (C) No, because neither the client nor the lawyer were aware, nor could have been aware, that the client would end up testifying in a case in support of a party to whom the lawyer would be opposing counsel (D) No, because the client is merely a witness in the matter and not a party to the litigation, so the client's interests are not directly adverse to the lawyer's other client, Acme Corporation

(B) Yes, a directly adverse conflict may arise when a lawyer is required to cross-examine a client who appears as a witness in a lawsuit involving another client of the lawyer The correct answer is B. The type of conflict described here—where one client might have to be an adverse witness in another client's case—comes up in the Comment to Model Rule 1.7 and is a type of conflict that law students often overlook. If a lawyer's client must serve as a witness for the opposing party in litigation, the lawyer will end up having to cross-examine one of her own clients in order to represent the other client as a party in the litigation. If the testimony will be damaging to the client who is represented in the lawsuit, the lawyer will need to impeach the witness's credibility, which normally involves portraying the witness either as untruthful, unintelligent (unreliable), or both. The client called as a witness for the opposing party is likely to feel betrayed if her own lawyer conducts the cross-examination so that she appears to be a liar or otherwise unreliable, and the client who is a party to the litigation will feel let down if the lawyer representing her goes easy on a hostile witness because the witness is also the lawyer's client. Answer B is therefore the correct answer because the clients' interests become directly adverse when one of the clients must testify against the other at a trial in which the lawyer would be cross-examining the client-witness. Note that one of the remedies available in cases of conflicts of interest is disqualification by the tribunal. Disqualification puts the client in a difficult position of having to scramble to find a new lawyer for litigation that is already underway, and it is embarrassing for the disqualified lawyer. If the lawyer is disqualified, the client may refuse to pay the lawyer's fees, especially when the lawyer should have recognized the conflict before representation commenced. Answer A is incorrect because conflicts of interest do not depend on the lawyer's expertise in a particular area of law. Besides that, many lawyers would be competent to handle a wide variety of client matters. Answer C incorrectly suggests that conflicts do not matter if the lawyer is not aware of them—in fact, lawyers have a duty to screen thoroughly for potential conflicts of interest like this at the outset of the representation and to remain watchful for unanticipated conflicts that might arise after the representation is underway. Finally, Answer D incorrectly suggests that a client serving as a hostile witness is not a significant enough situation to create a conflict of interest that would be an ethical violation for the lawyer. The ABA Comment to Model Rule 1.7 is very clear about this point—a client serving as a hostile witness in another client's litigation matter presents the lawyer with a conflict of interest that requires informed, written consent from each client.

Freemore hired Attorney Costle to provide the legal services necessary to set up her professional business. Subsequently, a dispute arose between Freemore and Attorney Costle over the fees, and the fee dispute turned into litigation. To support his claims and defenses in the fee dispute, Costle had to disclose to the tribunal exactly what he did for Freemore and the complexity of the issues involved, which necessarily involved the disclosure of confidential information. Was it proper for Attorney Costle to disclose this confidential information about Freemore merely to prevail in a fee dispute? (A) Yes, because the representation of Freemore ended when the fee dispute began, so Attorney Costle has no remaining duty of confidentiality to Freemore (B) Yes, a lawyer may reveal information relating to the representation of a client to the extent the lawyer reasonably believes necessary to establish a claim or defense on his or her behalf in a controversy between the lawyer and the client (C) No, because there is no exception to the confidentiality rule for fee disputes between a lawyer and client (D) No, because Attorney Costle owed a duty of confidentiality to Freemore, even after the represented ended

(B) Yes, a lawyer may reveal information relating to the representation of a client to the extent the lawyer reasonably believes necessary to establish a claim or defense on his or her behalf in a controversy between the lawyer and the client The correct answer is B. This question tests the exception contained in Rule 1.6(b)(5)—lawyers may disclose otherwise confidential client information "to establish a claim or defense on behalf of the lawyer in a controversy between the lawyer and the client ... or to respond to allegations in any proceeding concerning the lawyer's representation of the client." This exception to the confidentiality requirement is important because fee disputes between clients and lawyers are common, and the lawyer needs to provide evidence about the time and effort spent on the client's matter, and the nature of the lawyer's activities behind the scenes, even if this information would normally be confidential. The exception also covers lawyers defending themselves in legal malpractice suits by clients or disciplinary actions by the bar. Answer B is therefore the best answer. Answer A incorrectly suggests that the confidentiality requirement ends at the close of the representation, which is not the case—the duty is permanent. In other words, lawyers must continue to protect the confidentiality of information for their former clients as well as their current clients. Answer C is incorrect because Rule 1.6(b)(5) does cover situations where there is a fee dispute between lawyer and client. Answer D is incorrect because it ignores the "self-defense" exception for lawyers, even though it correctly acknowledges that the duty of confidentiality generally survives the termination of the representation.

Attorney Cabot, a partner at Big Firm, attended a networking reception. She found herself in conversation with a former law school classmate who was a managing lawyer at Boutique Firm. After a few drinks, the other lawyer started bragging about how the advantage of being managing partner at a smaller firm was that she was able to manipulate her firm's bookkeeping to understate the earnings of the firm and the salaries of the lawyers there for purposes of tax reporting. Attorney Cabot listened to this story and felt a mix of envy and disgust because it sounded lucrative but illegal. She kept the information in mind and gossiped about it to some of her friends, but she took no action to report the other lawyer to the authorities. Could Attorney Cabot be subject to discipline for not reporting this information to the state bar disciplinary authorities? (A) Yes, because if she does not report the tax fraud, she has become an accessory to a crime, under a theory of accessory liability. (B) Yes, a lawyer who learns of another lawyer committing tax fraud has a duty to report the ethical violation to the appropriate professional authority. (C) No, she does not have a duty to report potential ethical violations by lawyers who do not work at her firm, and the facts state that the other lawyer worked for another firm. (D) No, the rule for reporting ethical violations by other lawyers is permissive, not mandatory, so she could not be subject to discipline for a failure to report.

(B) Yes, a lawyer who learns of another lawyer committing tax fraud has a duty to report the ethical violation to the appropriate professional authority. Answer B is correct. ABA Model Rule 8.3(a) imposes an obligation on a lawyer "who knows that another lawyer has committed a violation of the Rules of Professional Conduct that raises a substantial question as to that lawyer's honesty, trustworthiness or fitness as a lawyer in other respects" to report the violator to the state bar disciplinary authority. Moreover, Comment 2 to the following Model Rule, Rule 8.4, expressly mentions tax fraud as the type of offense that reflects adversely on the perpetrator's fitness to practice law. Answer A is incorrect, because it is not clear from the facts in the Question that Attorney Cabot would be subject to criminal liability merely due to something someone told her in casual conversation at a reception. Answer C wrongly suggests that the duty to report applies only to an attorney's own firm—instead, lawyers may find themselves obliged to report a lawyer at another firm or corporate law department. Answer D is incorrect because reporting serious misconduct of other lawyers (like tax fraud) is not optional under Rule 8.3—it is mandatory.

Attorney Grimes recently moved laterally to a new firm. His previous firm represented Conglomerate Corporation, and Attorney Grimes occasionally worked on some of Conglomerate's legal matters. Attorney Grimes's new firm recently decided to represent the plaintiffs in a lawsuit against Conglomerate, and the cause of action arises from a new consumer protection statute that the state legislature passed in its last session. Attorney Grimes had left the previous firm before the new lawsuit began, and will not work on the new lawsuit at all. If Grimes knows some confidential information about Conglomerate that is material to the new matter, would it be proper for Grimes's new firm to represent the plaintiffs in an action that is directly adverse to Conglomerate? (A) Yes, assuming the new firm obtains written informed consent from Conglomerate, and screens Grimes from the case, providing written notice to Conglomerate about its screening procedures, and periodic certifications of compliance with the screening protocols (B) Yes, assuming the new firm screens Grimes from the case, and provides written notice to Conglomerate about its screening procedures, as well as periodic certifications that the firm is indeed following the screening procedures regarding Attorney Grimes (C) No, because Attorney Grimes has enough confidential information from working on Conglomerate's previous legal matters that there is a substantial risk that the firm will have an unfair advantage in the litigation (D) No, unless the firm obtains written informed consent from both the plaintiffs and from Conglomerate and declines representation of the plaintiffs in this case

(B) Yes, assuming the new firm screens Grimes from the case, and provides written notice to Conglomerate about its screening procedures, as well as periodic certifications that the firm is indeed following the screening procedures regarding Attorney Grimes The correct answer is B. This question tests the basic rule about imputed conflicts of interest, found in Model Rule 1.10, which centers around screening an individual lawyer who has a conflict that would otherwise not involve the rest of the lawyers in the firm. Note that if the firm does not follow proper screening procedures for the attorney who has the conflict, the entire firm has a conflict and cannot proceed with representing the new client. If it proceeded anyway, the firm would then be subject to disqualification by a tribunal, or a disciplinary action by the bar, or a legal malpractice lawsuit (or all of these). Answer B is the correct answer as it captures the basic rules about imputation and screening—the firm can proceed, but only if it effectively screens Attorney Grimes from the matter. Answer A incorrectly conflates this rule with the other conflict of interest rules that require informed consent—conflicts for clients at a lawyer's previous firm require only screening measures, not permission from the opposing party. Answer C is incorrect because the attorney's confidential information would be unavailable to the other lawyers in his firm if the firm effectively screened him. Answer D is incorrect for the same reason as Answer A—conflicts affecting only one lawyer, due to association with a previous firm, can be resolved by screening and notification protocols, without obtaining consent from the affected clients.

Attorney Andrews agreed to represent Client Charles and obtained Charles's written consent to divide the fees with a lawyer in a neighboring state where the trial was to occur. All the discovery and pre-trial work was to occur in the state where Client Charles and Attorney Andrews reside. The other lawyer, a well-known litigator and courtroom advocate, was to handle the actual trial, but Attorney Andrews was to handle the discovery and pre-trial motions. The case settled the day before the trial was to begin, so the other lawyer did not have to do anything except his usual trial preparation. Attorney Andrews had agreed beforehand with the other lawyer to divide the fees in half between them and that Attorney Andrews would take full responsibility for the representation overall. Client Charles had agreed in writing to this arrangement beforehand. To conclude the representation, Attorney Andrews sent Client Charles the expected bill, with half the fee going to Attorney Andrews and half the fee going to the other lawyer. Is Attorney Andrews, or the other lawyer, subject to discipline for this fee arrangement? (A) Yes, because Attorney Andrews should not have asked Client Charles to agree to pay another lawyer in another jurisdiction whom Client Charles never met (B) Yes, because Attorney Andrews and the other lawyer did not assume joint responsibility for the work (C) No, because Client Charles agreed to the arrangement beforehand in writing (D) No, because the other lawyer's reputation as a trial advocate may have influenced the opposing party to settle the case before going to trial

(B) Yes, because Attorney Andrews and the other lawyer did not assume joint responsibility for the work The correct answer is B. This question presents another common scenario in which multiple lawyers play a part in representing the same client—when the matter has components in multiple states (jurisdictions) and lawyers from each state have to play a role in the case. With regard to legal fees, the Model Rules recognize that lawyers in such cases may share fees generated from the case, but Rule 1.5(e)(1) requires that the lawyers share the fees in proportion to the work each performs or assume joint responsibility for the work. In this case, Answer Bcorrectly applies the rule to the facts because the lawyers did not divide the fees in proportion to the work performed nor did they assume joint responsibility for the work. As stated in the problem, the second lawyer ended up doing little or no legal work on the case but collected half the fees that the client paid. Lawyers who split fees in this way can face discipline. Answer A is incorrect because there was nothing wrong with Attorney Andrews asking the client to consent to retaining local counsel in the other jurisdiction where the trial would occur. Answer C is incorrect because the client's consent is only part of the analysis. Even when the client consents to the fee-sharing arrangement, the fee arrangement must be proportional to the work each lawyer performs. Otherwise, the lawyers have violated the rules and could face discipline. Answer D is incorrect because the lawyer's reputation alone is not enough to satisfy the requirement that the fee sharing reflect the proportion of work each lawyer performs. It is possible the other lawyer is entitled to some compensation for bringing an impressive reputation into the case, as this would certainly influence the decisions of the other party in the case, and could prompt a pre-trial settlement. Even so, the fact that he received half of the total fees after the first attorney did all the discovery and pre-trial work is disproportionate and would violate the rules.

Two litigants were represented by counsel. One of the attorneys had to testify briefly at her client's trial about a point that was uncontested, but nevertheless necessary, as an antecedent point for the issues in the case. The attorney testified to facts that she believed were true at the time she testified. Later, before the conclusion of the proceedings, the client discharged the attorney, and then informed the attorney of previously unknown facts that compel the conclusion that the attorney's testimony was incorrect. Does the attorney have a duty to take remedial measures to rectify the false statements? (A) Yes, because the client discharged the attorney and no duty of confidentiality remains after the termination of representation (B) Yes, because a lawyer must correct a false statement of material fact or law previously made to the tribunal by the lawyer (C) No, because the attorney was not aware at the time that the statements were false and, therefore, did not knowingly mislead the tribunal (D) No, because the lawyer has a duty of confidentiality that continues even after a client discharges the lawyer

(B) Yes, because a lawyer must correct a false statement of material fact or law previously made to the tribunal by the lawyer The correct answer is B. Model Rule 3.3(a)(3) requires lawyers to take remedial measures if the lawyer has offered material evidence and the lawyer later comes to know of its falsity. Here, that would probably mean disclosing the information to the tribunal and the opposing party, given that the false statements came from the lawyer herself. Answer B is the best answer to this question—even though the lawyer did not intentionally lie, now that she knows the truth, she must correct her earlier statements. Note that attorneys normally would not be a witness in a case in which they were also representing one of the parties, but these facts fit into one of the exceptions to the attorney-as-witness rule: an uncontested point for which the attorney's testimony is seemingly necessary in order to proceed to the real issues in the case. Also note that if the lawyer learned the truth only after the ultimate conclusion of the proceedings, there would be no duty to correct the earlier statements. Answer A is incorrect because the duty of candor to the tribunal is unaffected by the client discharging the lawyer. Lawyers have an ongoing duty of confidentiality to the client even after a client fires the lawyer, and the lawyer's duty of candor to the court also continues—and outweighs the duty of confidentiality. Answer C contradicts Rule 3.3(a)(3), which explicitly requires correction of false statements that the attorney believed were true at the time the statements were made. Answer D is wrong about the relationship between the duty of candor and the duty of confidentiality; in general, the duty of candor to the tribunal outweighs the duty of client confidentiality, when the two rules are in tension.

A client retained Attorney Stevens to handle a criminal matter. On Friday afternoon, the client delivered a check to Stevens's office. The amount of the check only covered the work Attorney Stevens anticipated completing the following Monday. Because the following Monday was a banking holiday, if Attorney Stevens deposited the retainer check into his client trust account on Friday afternoon, he would not have had access to the funds until Tuesday. Stevens deposited the client's check into his business checking account and paid himself with those funds on Friday before the firm closed. According to the Model Rules, did Attorney Stevens violate his ethical duty to his client by depositing the funds in this way? (A) Yes, because attorneys shall not accept amounts paid in advance for criminal matters (B) Yes, because attorneys shall deposit amounts paid in advance into a client trust account and the attorney shall not withdraw the funds until fees are earned or expenses are incurred (C) No, because if an attorney believes the funds will be earned within a short period, the attorney may deposit the amount he anticipates will be earned directly into his business account (D) No, because when an event out of an attorney's control, such as a bank holiday, causes the funds to be unavailable when the attorney anticipates he will need to withdraw them, the attorney may deposit the amount he reasonably believes will be earned or needed for expenses into his business account instead of the client trust account

(B) Yes, because attorneys shall deposit amounts paid in advance into a client trust account and the attorney shall not withdraw the funds until fees are earned or expenses are incurred This question is about temporarily commingling client funds. A lawyer may mistakenly believe that as long as the lawyer eventually separates the client's funds into another account, it is harmless to commingle the funds briefly for the sake of convenience. The Model Rules do not permit this practice. Even a temporary commingling of client funds with the lawyer's funds violates the Rules and could trigger disciplinary action. Thus, Answer B is the best answer because it states the correct rule: lawyers must promptly deposit client funds into a trust account, separate from the lawyer's business or personal accounts. Answer Aincorrectly states that lawyers cannot accept advance fees for criminal cases. On the contrary, advance fees in criminal cases are permissible and are the norm, except for court-appointed counsel for indigent defendants. Answer C incorrectly states that a lawyer may reasonably commingle client funds with the firm's funds for a short time. Such commingling violates the Rules. Answer D suggests that there is an expediency exception to the prohibition against commingling funds, but there is not. The Model Rules do not provide for bank holidays or other inconveniences. The rule is absolute.

A former client sued Attorney Andrews for legal malpractice, alleging that Andrews improperly withdrew from the representation on the eve of trial, thereby harming the client. Attorney Andrews claimed in defense that her withdrawal was due to an unforeseen conflict of interest that arose after the representation was underway, and that the conflict was serious enough to compel withdrawal from the representation. The client insists that there was no real conflict of interest that could have required Attorney Andrews' abrupt withdrawal. Should a judge allow expert testimony to help persuade the factfinder that the state's ethical rules concerning conflicts of interest indeed required withdrawal? (A) Yes, because compliance with the ethical rules is an absolute defense against legal malpractice and dispositive to the matter. (B) Yes, because compliance with the requirements of the ethical rules is relevant and material to the question of the attorney's malpractice, even if it is not dispositive. (C) No, raising this defense is immaterial and irrelevant, because withdrawal alone cannot furnish the basis of a malpractice action, even if the client suffered losses as a result. (D) No, because compliance with the ethical rules is irrelevant as a defense against a claim of legal malpractice.

(B) Yes, because compliance with the requirements of the ethical rules is relevant and material to the question of the attorney's malpractice, even if it is not dispositive. Legal malpractice actions are usually either a tort negligence claim or a claim for a breach of fiduciary duty, and most of our guidance for rules in this area comes from case law and the Restatement (Third) of the Law Governing Lawyers § 52 (This Question has facts that are analogous to Illustration 2 in § 52). Note that the ABA Model Rules do not furnish the basis for a malpractice claim against an attorney—the Model Rules are disciplinary rules, or rules of professional conduct for licensing purposes. This means that a violation of the Model Rules does not automatically mean that a client will succeed in a legal malpractice action, as a client would normally have to prove causation and/or damages, as well, and some clients would not be able to do this. Conversely, a successful malpractice claim against a lawyer does not necessarily mean that the lawyer would also be subject to discipline by the state bar. See ABA Formal Op. 18-481 (". . . a lawyer's error may impair a client's representation even if the client will never be able to prove all of the elements of malpractice'). At the same time, a violation of the ethical rules is certainly relevant and material to the question of whether a lawyer has breached a duty of care (or a fiduciary duty) to the client, even if the violation is not itself dispositive. In other words, a violation of the rules does not automatically trigger malpractice liability, but it should be admissible as persuasive evidence that the lawyer's conduct constitutes malpractice, and evidence of compliance with the rules, while not automatically shielding a lawyer from malpractice liability, is certainly relevant to the case. As the Restatement says in § 52, "the trier of fact may consider the content and construction of a relevant statute or rule, for example, a statute or a rule of professional conduct designed for the protection of persons in the position of the claimant." The section then continues: Such a provision tends to show how lawyers conduct themselves and how the promulgating authority concludes they should act. Typically, such rules are formulated on the basis of extensive consideration of what conduct is practical and desirable for lawyers, including consultation involving the bench and bar and comparison with similar standards adopted in other jurisdictions. The rules state minimum standards with which all lawyers should comply and often implement preexisting legal standards applicable to lawyers and other fiduciaries. . . . The use of the rules in malpractice litigation can also protect lawyers, for example when showing that a lawyer was compelled by rule to act in the way challenged by the plaintiff. Were the rules inadmissible in evidence, jurors would have no guidance in applying the duty of care except for often-conflicting expert testimony (sometimes inconsistent with the rules) as to how lawyers do or should act. Id. Answer A is incorrect because compliance with the Model Rules is not an affirmative defense to malpractice liability; that is, it does not automatically defeat a malpractice claim, even though it might be relevant in showing that the lawyer's conduct was not negligent. Answer C is wrong because an unjustified withdrawal could indeed furnish the basis for malpractice liability, as withdrawals (especially on the eve of trial) can be extremely prejudicial to the client, who must rush to find a new lawyer and bring that lawyer up to speed on the case. Finally, Answer D is incorrect because compliance or noncompliance with the Model Rules may be relevant and informative to a factfinder deciding whether the lawyer breached a duty of care to a client, even if it is not dispositive; that is, it does not guarantee the outcome of the malpractice case either way.

An attorney opened her own law practice and decided to attract new clients through advertising. She advertised her legal services on billboards emblazoned with the logo, "LOWEST LEGAL FEES IN THE CITY!" The billboard contained the firm's name, address, phone number, and website, but no disclaimers or qualifications about the claim regarding its legal fees. Approximately 10,000 lawyers practice in that city, and a legal aid clinic provides free legal services for homeless or indigent clients. The billboards are located on roads with very high rates of accidents and traffic fatalities, so the billboards are often visible to those who have just had an accident. Has the attorney violated the ethical rules governing lawyer advertising with these billboards? (A) Yes, because the billboard constitutes solicitation of clients, as it is directed to the general public (B) Yes, because it includes an unsubstantiated comparison of the lawyer's fees with the fees of other lawyers, presented in a way that would lead many people to think that the comparison could be substantiated (C) No, because the lawyer is merely exercising her First Amendment rights to free speech, and advertising falls under the Supreme Court's "commercial speech" doctrine (D) No, because no reasonable person would rely on this billboard and think that this attorney truly has lower fees than other attorneys

(B) Yes, because it includes an unsubstantiated comparison of the lawyer's fees with the fees of other lawyers, presented in a way that would lead many people to think that the comparison could be substantiated The correct answer is B. In previous eras, many states banned lawyer advertising entirely, but the U.S. Supreme Court held in a series of cases that the First Amendment protects the rights of lawyers to use advertisements, such as billboards, as long as they are not false, deceptive, or misleading. The ABA modified the Model Rules to reflect the Supreme Court's holdings in this area. Model Rule 7.1 prohibits communications that are false or misleading. The Rule requires strict truthfulness in lawyer advertising, as well as the name and office address of at least one lawyer or law firm responsible for the content of the advertisement. Model Rule 7.1 states "A communication is false or misleading if it contains a material misrepresentation of fact or law, or omits a fact necessary to make the statement considered as a whole not materially misleading." Comment 3 to Rule 7.1 adds, "Similarly, an unsubstantiated comparison of the lawyer's services or fees with the services or fees of other lawyers may be misleading if presented with such specificity as would lead a reasonable person to conclude that the comparison can be substantiated." Such an unsubstantiated comparison is found in the lawyer's advertising in this question. Given the number of lawyers in the city, it is impossible that the lawyer verified that each one of them charges more than she does for the same services; moreover, if other lawyers sometimes waive part or all of their fees (as when doing pro bono work), she would not have lower fees in that case since she cannot charge less than zero. Many non-lawyers would see the billboard and conclude that the attorney was indeed the least expensive lawyer in that urban area. Answer B is the best answer because the billboards make an unsubstantiated comparison with the fees of other lawyers. Answer A incorrectly conflates advertising with solicitation; the Supreme Court and the ABA distinguish between advertisements like billboards and direct solicitations, such as a lawyer approaching an individual in person and offering representation. The latter—in-person solicitation—is generally impermissible (with a few important exceptions), and the ban on solicitation does not violate the First Amendment, according to the Supreme Court. Answer C wrongly suggests that the First Amendment protects this type of advertising. Even though the U.S. Supreme Court did hold that lawyers have a First Amendment right to advertise, the Court allowed the state bars to impose some requirements and limitations on lawyer advertisements, including an absolute truthfulness requirement, which means the ABA and states can prohibit unsubstantiated comparisons with other lawyers. Answer D is factually incorrect—even if many people would view the billboard's claim with skepticism, many others would assume it is an accurate statement that the lawyer could verify or substantiate, so the advertisement is misleading.

Amanda was in her third and final year of law school. She applied for admission to practice law. The application forms for admission instructed applicants to list every place of employment for the previous seven years. Amanda decided to leave out the job she had for a year between college and law school, because she was embarrassed about it—she had worked for a year as a rent-a-clown for children's birthday parties. Worse, the job came to an end not because she started law school, but because the police arrived during the last party where she worked as a clown to arrest the foster care parents for ongoing child abuse. As a clown present in the home when the police and child protection services arrived, Amanda found herself under arrest, though the district attorney later dropped the charges when they realized she was merely present for a birthday party and was not part of the abuse of the foster children. At the same time, the authorities were upset that Amanda had previously performed as a clown at other birthday parties for the same foster parents, and she should have been aware that the children were suffering from abuse, but she had not reported it and continued to accept engagements to be a clown at their parties. Traumatized by the whole ordeal, Amanda could not continue working as a clown, so she took a month or two off before starting law school. Because the entire incident and the job seemed unrelated to the practice of law, she thought it was appropriate to omit it entirely from her application for admission. The state admissions authority, however, obtained her arrest record, and pieced together that she had worked as a party clown after college, though she did not include it in her employment history. She also had omitted a brief summer job during college because that employer never paid her, and the employer had gone to jail for operating a pyramid scheme. Is it proper for the state admissions authority to deny her admission for her failure to report these prior jobs? (A) Yes, because each of the jobs relates directly to her qualification or eligibility to practice law. (B) Yes, because omitting a material fact on an admissions application, such as a prior place of employment when the form instructs applicants to list every past job, violates the ethical rules and counts as a misrepresentation to the state disciplinary authorities. (C) No, because an omission of information cannot constitute a misrepresentation, because she did not state anything that was untruthful, and because she had no intention of harming anyone by deceiving or misleading the authorities. (D) No, because these were not legal jobs and are therefore not material facts for her application for admission.

(B) Yes, because omitting a material fact on an admissions application, such as a prior place of employment when the form instructs applicants to list every past job, violates the ethical rules and counts as a misrepresentation to the state disciplinary authorities. ABA Model Rule 8.1 prohibits lawyers, or would-be lawyers, from making false statements or misrepresentations when applying for admission to the bar. Moreover, Rule 8.1(b) specifically includes "failure to disclose" and "failure to correct a misapprehension" as material misrepresentations for purposes of the rules, so in this Question, Amanda has violated the rules. Answer A is incorrect, because applicants have a duty to disclose all prior employment if the application requests this information. Even if the jobs did not involve legal services, either the job or the applicant's reason for leaving that job could reflect on the applicant's character and fitness to practice law. Answer C is wrong because the Model Rules expressly cover failure to disclose facts necessary to correct a misapprehension and failure to respond to a lawful demand for information from an admissions authority. The applicant's intentions do not matter—the fact that she knowingly withheld information to avoid embarrassment or indirect association with criminals is enough to constitute a violation. Finally, Answer D is incorrect because, as stated before, the state bar can require applicants to submit information that does not necessarily relate to the applicant's legal skills or knowledge but may relate (even indirectly) to the applicant's character and fitness to practice law.

A defendant in a business-related lawsuit hires an attorney to represent him in the matter. The plaintiff is one of the client's vendors. The attorney explains that she needs $10,000 from the client at the outset of the representation, $2,000 of which is a non-refundable availability retainer, with the remainder to be used to cover expenses, court costs, and the attorney's hourly fees. These fees, costs, and expenses will be deducted as they accrue throughout the representation. Should more funds prove necessary, the client can provide more at the time; if any of the original $10,000 remains unused at the end of the representation, the attorney will refund the balance to the client. The client provides a check for $10,000 that the attorney deposits in his firm's primary bank account, carefully noting the deposit in the bank ledger and using the funds exactly as described. The attorney immediately used $2,000 to pay the monthly rent for her office space and began her representation of the client in the matter. The matter settled quickly, and the attorney returned $3,000 in unused funds to the client, with a full accounting of the costs, fees, and expenses deducted. Is the attorney subject to discipline? (A) Yes, because she used $2,000 of the client's funds to pay her rent immediately, before doing any work for the client (B) Yes, because she deposited the client's funds in her firm's regular bank account, rather than a special trust account containing only client funds (C) No, because the client consented to the entire arrangement, including that $2,000 would be non-refundable (D) No, because this is not a contingent fee, so the attorney can do whatever she wishes with the funds

(B) Yes, because she deposited the client's funds in her firm's regular bank account, rather than a special trust account containing only client funds The correct answer is B. The Model Rules strictly prohibit commingling client funds with the lawyer's own funds. This means lawyers must maintain a separate bank account that is exclusively for holding client funds—a client trust account. At the end of the representation, a lawyer must promptly return unused funds to the client. Answer B best describes the lawyer's violation of the rule, which in turn could result in a disciplinary action (a reprimand, license suspension, or permanent disbarment). The lawyer did most things right in this situation, but she deposited the client's money in her firm's regular bank account, commingling client funds with the firm's funds used for operating expenses. The Rules require that the money be deposited in a separate trust account. Answer A might sound like a violation (the lawyer immediately spending some of the money), but the $2,000 in question was a non-refundable availability retainer. Such a retainer need not be deposited in the trust account. Answer C is incorrect because client consent does not excuse the violation of the rule against commingling funds—unlike other ethical rules, there is no exception to this rule for a client who consents. Finally, Answer D is incorrect because lawyers must keep client funds in a separate account even in contingent-fee arrangements.

An attorney represents the defendant in a lawsuit over the breach of an oral agreement to sell a breeding cow. The cow had turned out to be already pregnant at the time the parties made their agreement. The attorney's client had agreed to sell the cow to another rancher, and received payment, but before delivery of the cow to the purchaser, the client discovered that the cow was pregnant. The client then reneged on the agreement because delivering a pregnant cow to the purchaser would be like giving the purchaser one animal (the expected calf) for free. The client suggested to the attorney that they assert a mistake of fact defense to the oral contract, claiming that neither he nor the purchaser knew or could have known that the cow was pregnant at the time of sale or would somehow become pregnant in the short time between payment and delivery. The attorney researched past court decisions and concluded that mistake of fact claims usually lose in scenarios like this. Moreover, in his various discussions with the client, the story has changed a little each time. The attorney now suspects that the client either is lying or is so confused that he will not be a credible witness at trial. The attorney would like to withdraw before filing an answer to the lawsuit asserting a defense of mistake of fact, because he knows they will probably not win, and he is not even sure if his client is telling the truth. Nevertheless, the client insists that the attorney should file the answer before withdrawing from the case, so that the client does not miss the deadline and face a default judgment, but does not mind if he must find another lawyer to handle the discovery and trial phase. Would it be proper for the attorney to file the answer to the pleadings, asserting a mistake of fact defense? (A) Yes, because a lawyer should follow the client's wishes about what to file or include in the pleadings (B) Yes, because the client's defense has some basis in fact and law, even if it seems improbable in both regards (C) No, because the attorney's research has led him to the conclusion that courts usually disfavor such defenses as a rule (D) No, because the attorney suspects his client is either lying or is confused about the facts

(B) Yes, because the client's defense has some basis in fact and law, even if it seems improbable in both regards The correct answer is B. Model Rule 3.1 prohibits lawyers from bringing frivolous claims or asserting frivolous issues in litigation, that is, an issue for which there is no basis in law or fact. Yet this does not mean that every claim or argument must be absolutely certain as the rules allow a lawyer to raise a weak argument, or one that has a small chance of succeeding, as long as there is some basis for it. In this question, the attorney's research has indicated that the "mistake of fact" defense will probably not succeed at trial. Nevertheless, it does not appear to be out of the question, as mistake of fact can be a valid defense in other situations, and it is possible that a court would apply the rule here, after the presentation of all the facts at trial. Similarly, doubts about the client's reliability does not mean the lawyer lacks any basis in fact—often clients retell their story a little differently every time, even when they are not trying to deceive anyone. Even if the client is unreliable, an examination of the cow by veterinary experts during discovery could date the start of pregnancy before the sale agreement and thereby validate the mistake of fact defense. Thus, Answer B is the best answer—even improbable or weak arguments can have some basis in law and fact and are therefore permissible under Rule 3.1. Answer A is incorrect because an attorney may not simply do whatever the client asks—lawyers must represent their clients within the parameters set forth in the ethical rules, and the facts here trigger the applicability of some rules that override the client's instructions. Answer C wrongly suggests that unfavorable precedent automatically triggers the "no basis in law" provision in Rule 3.1, but this is not the case; "usually disfavor" is not the same as "no basis in law." Similarly, Answer D is incorrect because it makes the rule overly broad. A lawyer can raise a factual issue even if it is uncertain or doubtful, as long as there is some basis for it. The fact that the cow turned out to be pregnant, and was early enough in the pregnancy for the other party not to notice at the time of the agreement, suggests that there is some basis for this claim, even if it is a long shot.

Executives at a large pharmaceutical manufacturer discovered that one of the company's foreign subsidiaries had bribed local government officials to obtain lucrative government contracts. During the internal corporate investigation of the matter, the corporation's directors asked their in-house general counsel to send written inquiries to dozens of the corporation's foreign managers about whether similar payments or bribes were occurring elsewhere. After reviewing the responses to these inquiries and following up with phone calls and meetings, the corporate directors self-reported any questionable transactions to the Internal Revenue Service (IRS). The IRS then demanded to see all the original written responses to these internal inquiries. The corporate directors and general counsel refused, claiming that the information was privileged. Are the responses to the internal inquiries privileged communications? (A) Yes, because the information was provided to an attorney and is therefore privileged (B) Yes, because the information was provided at the request of the corporate directors to the in-house attorney, and the information was work-related and was necessary for obtaining legal advice (C) No, because lower-level employees at a corporation, who play no part in controlling the corporate decision-making, do not count as part of the "client" for purposes of attorney-client privilege (D) No, because the inquiries and written responses are underlying facts in the case, and therefore not covered by attorney-client privilege

(B) Yes, because the information was provided at the request of the corporate directors to the in-house attorney, and the information was work-related and was necessary for obtaining legal advice The correct answer is B. The facts described here are reminiscent of those in Upjohn v. United States, 449 U.S. 383 (1981), in which the U.S. Supreme Court redefined the parameters for attorney-client privilege when the client is a large corporation. The Court's holding became known as the "Upjohn doctrine" and applies the privilege to communications with lower-level corporate employees if 1) the communications were made to in-house counsel at the direction of corporate directors, 2) the information concerned matters within the scope of the employee's workplace duties, 3) the corporate directors had no other way to obtain the information about the internal operations at the company, and 4) the employees who provided the information were aware that the corporation was seeking legal advice or representation about the matter in question. Answer B correctly captures the Upjohn factors. The corporate directors asked their in-house counsel to send the inquiries and solicit the answers, and the employees sent answers to the in-house lawyer knowing that there was a legal investigation underway. Answer A is incorrect because it is too broad—not all information provided to an attorney is privileged, and communications by lower-level employees to the employer's lawyer might not be privileged if it does not satisfy all the factors of the Upjohn test. Answer C is wrong because it states the "control group" test applied by the lower courts in the Upjohn case, which the Supreme Court reversed. Even though the "control" test or "need-to-know" standard still operates as an underlying premise in some jurisdictions, in analyzing attorney-client privilege for corporate clients, communications made at the behest of the top management to in-house counsel about work-related information (information not otherwise available) would in fact come under the protection of attorney-client privilege. Answer D incorrectly suggests that written inquiries and answers used in internal corporate audits or investigations are "underlying facts" not covered by privilege. For purposes of attorney-client privilege, the documents themselves would normally count as privileged "communications," not "facts," although the facts discussed in the documents (such as whether bribery occurred) would not be privileged.

A husband and wife met with Attorney Lewis about obtaining legal guardianship and power-of-attorney for the wife's elderly mother, who was suffering from early-stage senile dementia. The couple needed to be able to help manage the assets and finances of the wife's elderly mother because they had taken her into their home. Attorney Lewis said he would charge a flat fee for his legal services. The couple agreed to pay a reasonable fee. As the couple was leaving the meeting, Attorney Lewis said he would formalize their fee arrangement in a written document and send them a copy. Unfortunately, an urgent issue in another unrelated case distracted him so he forgot to prepare a written fee agreement. Eventually, Attorney Lewis drafted and filed the necessary documents to place the elderly mother under the legal custodial care of the couple. At the conclusion of the matter, Lewis sent the couple a bill. The bill included the lawyer's flat fee, as the couple had agreed, plus administrative filing fees and court costs. The amounts were accurate and reasonable. Is Attorney Lewis subject to discipline for his actions? (A) Yes, because the lawyer neglected to formalize the fee agreement in writing at the outset of the representation (B) Yes, because the lawyer did not explain to the clients that they would be responsible for administrative fees and court costs in addition to his legal fees (C) No, because the oral fee agreement was satisfactory since written fee agreements are only required when an attorney charges a contingent fee (D) No, because all of the fees and costs were reasonable, and the central tenet of the Rules of Professional Conduct pertaining to fees is that they be reasonable

(B) Yes, because the lawyer did not explain to the clients that they would be responsible for administrative fees and court costs in addition to his legal fees The correct answer is B. Model Rule 1.5 covers the lawyer's ethical duties regarding legal fees and begins with a basic requirement that fees be "reasonable." In addition, the Rule and its Comments require lawyers to clarify for clients whether the client is responsible for fees and court costs in addition to compensating the lawyer for time spent on the case. Clients often assume that the lawyer's stated "fee"—whether hourly, contingent, or a flat fee—includes everything, and they are surprised when they receive a bill at the end of the representation that lists various costs (hiring experts, copying costs, and so on) and court fees on top of the lawyer's fee for providing legal services. Answer B accurately reflects the requirement of the Model Rules. It is essential for lawyers to communicate clearly to clients their responsibility for court fees and other costs. Answer A wrongly implies that the lawyer needed to formalize the fee in writing. The Model Rules require written fee arrangements only for contingent fees and merely recommend written agreements for flat fees or hourly fee arrangements. Answer C is incorrect because the fee agreement fails to specify whether the clients would have to pay court fees and other costs. Answer D is also incorrect. Even though the fee here was reasonable, the lawyer needed to communicate certain information to the clients in order to avoid common misunderstandings.

An attorney moved to a new firm. Her previous firm has represented Conglomerate Corporation for many years, and the attorney occasionally worked on some of Conglomerate's matters, in the course of which she acquired confidential information about the company. The attorney's new firm represents a client in an action against Conglomerate. Conglomerate's lawyers at the attorney's previous firm move to disqualify the attorney's new firm from representing the client in the case against Conglomerate. The partners at the attorney's new firm were unaware that this attorney's previous firm represented Conglomerate, first learning of this when they received the motion to disqualify their firm. The firm immediately implemented screening procedures to keep the attorney from working on the case or receiving or sharing any confidential information about the case or about Conglomerate's other legal matters. The firm provided notice to opposing counsel about the screening procedures and plans to provide periodic certifications of compliance as well. Should the attorney's new firm be subject to disqualification? (A) Yes, unless the client provides written informed consent to waive the potential conflict of interest (B) Yes, because the new firm did not implement the screening procedures soon enough (C) No, because at this point disqualification would be very disruptive to the litigation and prejudicial to the client (D) No, because they implemented appropriate screening procedures as soon as they learned of the attorney's conflict of interest and provided notice to the opposing party

(B) Yes, because the new firm did not implement the screening procedures soon enough The correct answer is B. When an attorney in a firm has a conflict of interest that could disqualify the rest of the lawyers in the firm, the firm must implement timely screening procedures to prevent the disclosure (in either direction) of any confidential information. In this question, the firm did not catch the conflict soon enough, and therefore the screening procedures were probably not timely. The court in the case could therefore disqualify the entire firm from representing the party that is litigating against Conglomerate. Answer B is correct—the firm was too late in implementing the required screening measures, which shows the importance of screening for conflicts of interest beforehand. Answer A is not the best answer here, although it is possible that an opposing party could acquiesce to a conflict of interest by not filing a motion for disqualification. Answer A focuses on consent of the affected parties, but the rules governing imputed conflicts focus on screening and notification procedures, rather than obtaining consent. Answer C is also not the best answer, although disruption to the proceedings is one of the factors courts consider when deciding whether to disqualify a firm. If the motion to disqualify were filed before trial, the disruption would not be very serious. Answer D is incorrect because screening a lawyer with a conflict of interest after the representation is well underway means the firm should probably be disqualified—it is not enough that the firm responded quickly to the opposing party's motion.

An attorney represents a wealthy executive in a divorce case. In the course of the representation, the attorney learns that the client intends to purchase and develop several parcels of land in an undeveloped area on the outskirts of the city. It happens that the attorney also represents a physician in an estate planning matter. The attorney and the physician have a longstanding relationship, so the attorney mentions something to the physician about the parcels of land that are for sale on the outskirts of the city, which another client brought to the attorney's attention. The attorney recommends that the physician also try to buy one of the parcels of land as an investment for the estate, knowing that the area will soon see development and the property values will increase. Even in the short term, the attorney knows that the physician could probably make a quick profit by buying a parcel and selling it to the other client when that client gets around to purchasing the parcels. The attorney did not mention to the physician that his other client was the individual planning to purchase and develop the parcels. Would the attorney be subject to discipline for giving the physician this offhand tip? (A) Yes, because the attorney should have given the first client's contact information to the physician so that the physician could inquire more directly about the plans for development and see if the client would be interested in purchasing the parcels together (B) Yes, because using information gleaned from representation of a client to the disadvantage of the client violates the lawyer's duty of loyalty, even when the lawyer uses the information to benefit a third person, such as another client (C) No, because the attorney is merely recommending to another client that he buy a parcel of land, using information from one client to help another client, and there is no conflict of interest here (D) No, because the attorney was careful not to divulge the client's identity or any confidential information about the client's case and the attorney is acting in the physician's best interest, not engaging in self-dealing

(B) Yes, because using information gleaned from representation of a client to the disadvantage of the client violates the lawyer's duty of loyalty, even when the lawyer uses the information to benefit a third person, such as another client Model Rule 1.8(b) proscribes, "A lawyer shall not use information relating to representation of a client to the disadvantage of the client unless the client gives informed consent, except as permitted or required by these Rules." Lawyers often learn valuable information from clients that would enable other clients to make lucrative investment decisions, but disclosure of such information could be harmful to the client from whom the lawyer learned it, as when prices go up due to other clients acting on the lawyer's disclosures of the confidential information (the lawyer's investment tips). Answer B is the correct answer and draws from the guidance in Comment 5 to Rule 1.8. The physician buying parcels to resell at a higher price to the other client obviously puts the client in a worse position than if the lawyer had kept the information confidential. Even if the physician buys other nearby parcels besides the ones that the first client intends to purchase, basic concepts of supply and demand would predict that land prices would go up as a result, again making it more difficult for the first client to purchase the parcels he wants. Answer A is incorrect because disclosing one client's contact information and investment plans to another client clearly violates the first client's confidentiality, and the recipient of the information could still infer enough about the situation to buy the parcels behind the other client's back, in an attempt to make a quick profit. Answer C also wrongly ignores the risk of harmful action that could be taken by one client against another once in possession of the information. Rule 1.8(b) prohibits these disclosures even where the attorney would not benefit personally, because of the potential harm to the first client and the breach of trust or loyalty involved. In addition, the attorney could have a subtle personal conflict of interest by wanting to curry favor with the physician, or to induce the physician to continue using the lawyer's legal services. Answer D is incorrect because it is not enough that the lawyer protected the first client's identity—the lawyer disclosed the client's information in a way that is likely to be disadvantageous to the client.

An attorney represented a client in an estate planning matter. The client was showing early signs of Alzheimer's dementia: forgetfulness, abrupt changes in the conversation, and repeating the same story or information within the same consultation. At the beginning of each conversation, the client would ask the attorney to remind her of the attorney's first name and how he liked being a lawyer, though she always recognized the attorney as her legal representative. Home health aides visited the client twice a week, but otherwise the client lived alone and took care of her daily needs without incident, though she no longer drove a car. The attorney found it frustrating to discuss the more difficult legal questions in the estate plan with the client, who would frequently repeat a question twenty minutes after the attorney had given a lengthy explanation to the same question. The client's overall objectives were clear—she wanted to divide the estate equally among the surviving heirs. At some point, the attorney started to follow his own judgment about the subtler questions, such as the creation of a spendthrift trust, the liquidation of real property rather than bequeathing the title to one or more heirs, and so on. All the attorney's actions in the matter were standard practices among estate planning lawyers and were objectively competent. In fact, the attorney fulfilled the client's objectives and protected the client's interests, even though he decided not to bother the client with the details. Could the attorney be subject to discipline for following his own judgment on the practical questions of estate planning, without including the client in these decisions? (A) Yes, when lawyers represent elderly clients in estate planning matters, they should consult with the family and the heirs about how best to structure the estate for the convenience everyone involved. (B) Yes, lawyers have an ethical duty to maintain, as much as possible, a normal client-lawyer relationship with the client, even when the client suffers from diminished capacity. (C) No, because the lawyer's actions were objectively reasonable and competent, and he protected the legal interests of the client. (D) No, when a lawyer realized that a client suffers from diminished mental capacity, the lawyer may substitute his own reasonable judgment and not include the client in some of the decisions, so long as the lawyer fulfills the client's overall objectives.

(B) Yes, lawyers have an ethical duty to maintain, as much as possible, a normal client-lawyer relationship with the client, even when the client suffers from diminished capacity. The correct answer is B. ABA Model Rule 1.14 provides, "When a client's capacity to make adequately considered decisions in connection with a representation is diminished, whether because of minority, mental impairment or for some other reason, the lawyer shall, as far as reasonably possible, maintain a normal client-lawyer relationship with the client." The Restatement (Third) of the Law Governing Lawyers § 24 includes similar verbiage. In this case, it is not clear how much the client's early-stage dementia impairs her ability to participate in the decisions, except that the attorney needs to exercise extra patience and explain the same points a few times. As Comment 1 to Rule 1.14 concludes, "So also, it is recognized that some persons of advanced age can be quite capable of handling routine financial matters while needing special legal protection concerning major transactions." Thus, the attorney in this question should have discussed decisions with the client regarding the creation of a trust or the liquidation of certain assets from the estate. Answer A is incorrect because lawyers must maintain client confidentiality as much as possible, even for clients who have diminished mental capacity, except when the lawyer is making disclosures necessary to take protective actions for an incompetent client, which is not the case here. Normally, as here, a lawyer should not disclose the client's estate planning decisions to the relatives or heirs, unless the client expressly authorizes such disclosures. Answer C is wrong because Rule 1.14 requires lawyers to consult with clients to the extent possible. Even when a client suffers from a mental impairment, it is unethical to ignore a client's wishes, or exclude a client from important decisions about the representation, regardless of the soundness of the lawyer's own judgments or the outcome that results. Answer C is exactly what Rule 1.14 and § 24 of the Restatement forbid—well-intentioned lawyers substituting their own judgment for the client's whenever a client suffered from occasional forgetfulness or confusion.

An attorney has represented a particular client several times in various civil litigation and transactional matters. The client and the attorney decide to purchase a piece of property together, which has an appraisal value of $4 million. The lawyer and client each contribute 50% to the purchase price, but the client receives a 5% interest in the property and the attorney receives a 95% interest in the property, by agreement of the parties. The greater ownership share for the lawyer is due to some unpaid legal fees that the client owed the attorney, totaling around $2,000. Even so, the value of the representation is far less than the value of the additional interest that the attorney received in the property. The client was represented by independent legal counsel for the property purchase. Could the attorney be subject to discipline for this lopsided deal with the client? (A) Yes, attorneys cannot accept an interest in an asset of a client as a fee for representation. (B) Yes, the business transaction must be fair and reasonable even when the client obtains representation by independent legal counsel, though representation by counsel will be a factor in determining the fairness of the transaction. (C) No, attorneys can enter into business transactions with clients, as long as they are unrelated to the current matter for which the attorney is representing the client. (D) No, any requirement that the business transaction be fair and reasonable is eliminated when the client is represented by independent legal counsel, as the independent counsel can advise the client on any unfairness and discourage transactions that are unfavorable to the client.

(B) Yes, the business transaction must be fair and reasonable even when the client obtains representation by independent legal counsel, though representation by counsel will be a factor in determining the fairness of the transaction. The correct answer is B. Model Rule 1.8(a) covers lawyers' business transactions with their clients (apart from the usual fee agreements for legal representation of the client). The Rule provides that one of the core requirements of such transactions is that they be fair and reasonable, to prevent lawyers from taking advantage of their own clients. Here, the deal is not fair or reasonable—the client contributed $2 million and received only a 5% ownership share, compared to the lawyer's 95% share for an identical contribution toward the purchase price. In other words, the outcome of the transaction for the client is to forfeit over a million dollars of equity in the purchased property merely to clear a $2,000 debt to the attorney. Answer B is thus the correct answer. Even though it is a relevant factor that the client had another lawyer representing him for this transaction (see Comment 4 to Model Rule 1.8), the transaction here is still unfair and is therefore unethical under the Rules. Answer A wrongly suggests that lawyers cannot accept ownership shares of assets as payment for legal fees—lawyers can in fact do this, as long as the transaction complies with all the formal requirements of Rule 1.8(a). Answer C is incorrect because it ignores the requirements that Rule 1.8(a) imposes on lawyers for any business transaction with the client, regardless of the connection with the original representation. Answer D overstates the exception in Comment 4 to Rule 1.8—while independent legal representation does eliminate some of the formalities normally required under Rule 1.8(a), the transaction must still be reasonable and fair to the client.

Bonnie and Clyde are co-defendants, facing felony charges for a string of armed robberies. Attorney Abbott represents the two of them together. The District Attorney has a theory that Bonnie planned the crime and was the only one carrying a weapon, while Clyde helped collect the money during the robbery and drove the getaway car. This prompts the District Attorney to offer Clyde a plea agreement—if Clyde will plead guilty to a misdemeanor and then testify against Bonnie, he will receive a minimal amount of jail time. Attorney Abbott is loyal to both clients, so he insists that Clyde reject the District Attorney's offer. Which of the following is true regarding Attorney Abbott's ethical duties? (A) Attorney Abbott should balance the interests of both clients, and taken together, should discourage Clyde from testifying against Bonnie if the potential increase in Bonnie's sentence would exceed the reduction in Clyde's sentence. (B) Attorney Abbott can ethically represent Bonnie and Clyde with the informed consent, confirmed in writing, of both clients. (C) As soon as plea negotiations involving Clyde's options started, Clyde should have had independent counsel representing him. (D) Attorney Abbott should advise Clyde to testify against Bonnie, because as Bonnie's counsel, he should be able to offset the unfavorable testimony coming from his other client, Clyde.

(C) As soon as plea negotiations involving Clyde's options started, Clyde should have had independent counsel representing him. Answer C is the best answer; the lawyer has an insurmountable conflict of interest. Conflicts of interest are one of the main ethical problems facing criminal defense lawyers, because many defendants lack financial resources to hire separate lawyers, and co-defendants often want to "share" a lawyer to save on legal fees. The facts in this Question are reminiscent of one of the illustrations in the Restatement (Third) of the Law Governing Lawyers § 129 (Conflicts of Interest in Criminal Litigation). As soon as the prosecutor began a plea negotiation with one of the co-defendants, it was foreseeable, if not inevitable, that this would involve asking one defendant to testify against the other. This immediately puts the lawyer representing the two of them in a difficult bind—it would be in the best interest of the first client to accept the deal and testify against the other defendant, so the lawyer should advise him to take the deal, but this would simultaneously harm the lawyer's other client—the other defendant. Note that unlike the Restatement, the ABA Model Rules do not have a separate section for conflicts in criminal litigation, but the same rule is part of Model Rule 1.7, and the Comment to Rule 1.7 applies the rule to both civil and criminal litigation, reaching the same result as the Restatement. Answer A is incorrect because the ethical rules forbid lawyers from balancing their client's interests in this manner (merely trying to ensure the benefit to one exceeds the harm to the other)—a lawyer who has to engage in such balancing of interests should decline or withdraw from the representation of one or both co-defendants. Answer B is incorrect but admittedly tricky, because both the Restatement § 129 and Model Rule 1.7 normally allow lawyers to proceed with joint representation if both clients give informed consent, confirmed in writing. Nevertheless, both the Model Rules and the Restatement would deem this type of conflict "nonconsentable," that is, the interests of the two clients are so adverse that it is unreasonable to think the same attorney could provide adequate representation to both at once. Answer D is wrong because a lawyer cannot advise one client to take a position that would be so adverse to the interests of the other client. Note that according to the Restatement, Attorney Abbott could indeed represent both defendants, with their informed consent, if neither was testifying against the other. The Comment to Model Rule 1.7 strongly discourages joint representation in criminal cases, but it stops short of expressly forbidding it.

Attorney Elkins spent several years working for Big Firm in its business litigation division. While there, Attorney Elkins represented a client in an action against Conglomerate Corporation alleging unfair trade practices and antitrust violations. Attorney Elkins eventually left Big Firm and accepted a position at a federal regulatory agency. There, Attorney Elkin's first assignment was to bring an enforcement action against Conglomerate for violating antitrust laws and unfair trade practice laws. Attorney Elkins obtained written informed consent from his previous client to pursue a related matter against Conglomerate, but not from the agency itself or from Conglomerate. Is it proper for Attorney Elkins to represent the government in an enforcement action against his prior opponent, if the matter is substantially related? (A) Yes, because the conflict of interest rules apply to attorneys leaving government service for private practice, but here, Attorney Elkins has done the opposite, going from private practice to government service (B) Yes, because the interests of Attorney Elkin's previous client and the agency align, rather than being adverse, so there is no conflict of interest (C) No, because Attorney Elkins pursued a claim on behalf of a private client, so he may not pursue the claim on behalf of the government, unless the appropriate government agency gives its informed consent, confirmed in writing (D) No, because Attorney Elkins did not obtain written informed consent from Conglomerate at the outset of the new enforcement action, even though the action is directly adverse to Conglomerate

(C) No, because Attorney Elkins pursued a claim on behalf of a private client, so he may not pursue the claim on behalf of the government, unless the appropriate government agency gives its informed consent, confirmed in writing The correct answer is C. Comment 3 to Model Rule 1.11 governs the facts here—the consent of the government agency is required in order for Attorney Elkins to proceed with the matter on behalf of the government. Of course, the government is likely to consent to an enforcement action by one of its own employees. Answer C is correct because Attorney Elkins already pursued this claim on behalf of a private client, and would have obtained confidential information in the process about the parties involved. The government agency must be informed of this situation before he can proceed. Answer A incorrectly asserts that the conflicts rules pertain only to lawyers leaving government service, but in fact Rule 1.11 covers both directions of career movement—lawyers entering government service from the private sector, and lawyers entering the private sector after working for the government. Answer B is incorrect because the question of Attorney Elkin's conflict does not merely turn on the alignment of the agency's interests, but rather on the agency being aware of the situation and consenting to it. Answer D is incorrect because Model Rule 1.11 requires consent from the government agency, rather than the private party involved.

An attorney managed her own practice as a sole practitioner and needed to attract more clients. One day, she heard that John Smith, a former classmate from law school, was facing disciplinary action by the state bar for making live telephone solicitations of prospective clients. The attorney called Smith and offered to represent him in his hearing before the grievance committee for a fee of $400 per hour, which is higher than the usual rate for such representation. The attorney's motivation was primarily for pecuniary gain, not concern for her former classmate. Could the attorney be subject to discipline for making this live telephone solicitation of her former law school classmate? (A) Yes, because the attorney offered to represent the prospective client for a higher-than-average fee (B) Yes, because the attorney solicited professional employment when a significant motive for the doing so was pecuniary gain (C) No, because the person she solicited as a prospective client was also a lawyer (D) No, because the prospective client is an acquaintance from law school

(C) No, because the person she solicited as a prospective client was also a lawyer The correct answer is C. Model Rule 7.3 forbids most in-person solicitation of potential clients, including telephone calls and "live chats" on social media. Note that the rules for solicitation (approaching an individual to offer legal representation) are much stricter than the rules for advertising, that is, communications directed at the general public rather than an individual. The ABA and the U.S. Supreme Court have concluded that solicitation presents far more danger of coercion, manipulation, or exploitation by lawyers. Nevertheless, Rule 7.3 includes some important exceptions: other lawyers and persons with whom the lawyer has a family, close personal, or prior professional relationship. In this question, the attorney clearly engaged in telephone solicitation when she called a former classmate—not necessarily a close friend—but the individual was himself a lawyer, so the exception in Rule 7.3(a)(1) would apply. Answer C is the correct answer—because John Smith, the target of the attorney's solicitation, was himself a lawyer, the attorney did not violate the rules. Answer A is incorrect because the fee itself does not affect the solicitation rule, unless the attorney is offering to do the case pro bono; it was not a violation to offer to represent Smith for a high-than-usual fee, as Smith could simply decline the offer if he wanted to hire someone at the usual rate, or he could represent himself. Answer B states the general prohibition on soliciting and mentions the pecuniary motive, but it ignores the exception in 7.3(a)(1) where the target of the solicitation is also a lawyer. Answer D incorrectly suggests that the target being an acquaintance would make the solicitation proper. Instead, Rule 7.3(a)(2) would require that Smith have a "close personal relationship" with the attorney who made the solicitation, which the facts in this question do not indicate.

Attorney Ellsberg represents a chemical manufacturer. A regional vice president recently informed Attorney Ellsberg that there was a chemical spill that released hundreds of gallons of toxic substances into a stream that ran into the town's nearby water supply reservoir. The spill occurred because a newly hired employee turned the wrong valve during a training exercise at the plant. Attorney Ellsberg explained that the corporation could face civil liability in either tort actions or regulatory actions by governmental entities at the state and federal levels and urged the vice president to report the spill immediately, if it was still unreported. The vice president replied that the company could not afford the negative publicity and the impact it would have on its share price. He reminded Attorney Ellsberg that the upper management of the company received most of its compensation in the form of preferred stocks and options, so it seemed unfair to penalize them through a loss in share price. Attorney Ellsberg explained that he would have to withdraw from representation and would report the incident to the necessary public health officials, which he did, despite the vice president insisting that this was confidential information. Did Attorney Ellsberg violate his ethical duty of confidentiality to the client? (A) Yes, because the company's conduct may not have been criminal and did not yet result in anyone's death or serious bodily injury (B) Yes, because Attorney Ellsberg disclosed confidential information and betrayed his duty of loyalty to the client (C) No, because Attorney Ellsberg believed the company's disposal of waste products was likely to cause serious injury to others (D) No, because Attorney Ellsberg believed that the bad publicity and decrease in share price would be even worse if it emerged that there was an attempted cover-up after the chemical spill

(C) No, because Attorney Ellsberg believed the company's disposal of waste products was likely to cause serious injury to others The correct answer is C. ABA Model Rule 1.6(b)(1) includes an important exception to the confidentiality rule: "to prevent reasonably certain death or substantial bodily harm." Comment 6 to Rule 1.6 offers an example reminiscent to the facts in this question—a lawyer whose client had polluted a local water supply—though this question adds a substantial number of additional details. Note that lawyers should first try to persuade the client to do the right thing (that is, make the necessary disclosure, authorize the lawyer to make the disclosure, or take action to eliminate the danger). The lawyer in this question did so. Answer C is the correct answer here because the lawyer reasonably believed that his client's disposal of waste products would poison numerous unsuspecting victims, and the disclosure was necessary to prevent deaths or serious health consequences from drinking contaminated water. Thus, the lawyer's disclosure fell squarely under the first exception to Rule 1.6 and did not violate his ethical duty to the client. Answer A incorrectly suggests that the client's conduct would have to be criminal in order for the lawyer to be free to make the disclosure—but the "save a life" exception can apply even to non-criminal incidents, like accidents or the discharge described here. Answer B is wrong because it ignores the important exception to the duty of confidentiality—when disclosure is necessary to prevent reasonably certain death or serious bodily injury. Answer D is wrong because it relies on the wrong reason or rationale—the lawyer's concerns about the client's reputation and share price would not be enough to justify such a disclosure. The confidentiality rules have no exception for protecting the client's own financial or reputational interests.

Attorney Groves was a well-known criminal defense lawyer and he agreed to represent Famoso, a celebrity who is a defendant in a high-profile murder case. Attorney Groves filed the proper notice with the court and the prosecutor's office that he was representing Famoso. Attorney Groves also filed a motion to exclude Famoso's confession that he gave to the police on the night of the murder while Famoso was somewhat intoxicated; the attorney's motion concedes the intoxication and contends that this nullifies the voluntariness of the confession for Fifth Amendment purposes. The news media learned that Groves was representing Famoso, and news commentators began to speculate that Famoso must be guilty if he hired such a notorious defense lawyer. Famoso was furious that anyone knew that he had hired a lawyer, which he claimed was confidential. Did Attorney Groves violate the ethical rules of confidentiality by making these disclosures without his client's express authorization? (A) Yes, because Famoso is already in the public light as a celebrity and therefore has a lower reasonable expectation of privacy, and he knew he was hiring a well-known defense lawyer (B) Yes, because a lawyer has an ethical duty to obtain express authorization from a client before taking any action that could disclose a client's confidential information (C) No, because a lawyer is impliedly authorized to make disclosures about a client when appropriate in carrying out the representation (D) No, because Attorney Groves was raising a constitutional issue in the motion that has greater importance than the duty of confidentiality to a client

(C) No, because a lawyer is impliedly authorized to make disclosures about a client when appropriate in carrying out the representation The correct answer is C. Model Rule 1.6 governs confidentiality, and the first part of the confidentiality requirements includes a qualification "unless ... the disclosure is impliedly authorized in order to carry out the representation." In addition, Comment 5 to Model Rule 1.6 says, "In some situations, for example, a lawyer may be impliedly authorized to admit a fact that cannot properly be disputed or to make a disclosure that facilitates a satisfactory conclusion to a matter." If a client retains a lawyer to defend him in litigation, this automatically implies that the lawyer will have to sign the pleadings and motions filed in court, thereby disclosing the fact that legal representation is underway. Note that if a client hired a lawyer merely to provide legal advice, without further action, even the fact that the lawyer is representing the client would be confidential information that a lawyer must safeguard. In this problem, Answer C is the correct answer—Famoso hired Attorney Groves to defend him against criminal charges, and it was appropriate for Attorney Groves to notify the court and the prosecutor that he was providing representation and to file the preliminary motion mentioned here. Even if the client had clearly forbidden the lawyer from letting anyone discover the representation, it would have been impossible for the attorney to provide a legal defense without identifying himself on court documents. Answer A, however, is incorrect—even though "public light" can be a factor for celebrities in defamation (tort) actions, it is not relevant in determining whether a lawyer has an ethical duty to keep certain information confidential. Answer B wrongly states that lawyers must have express authority for any disclosures whatsoever. The confidentiality rule (1.6) clearly allows for "implied authorization" of disclosures necessary to carry out the representation, besides a number of other exceptions. Answer D is wrong because there is no balancing test for constitutional issues versus confidentiality—the crucial factor here is that the disclosure was implied (though unstated) in what the client asked the lawyer to do, not that the lawyer needed to raise a serious legal issue.

Lex Luther committed a spectacular crime that garnered media coverage, and his arrest and prosecution led to even more media attention. Atticus Finch was the criminal defense lawyer representing Lex Luther. When a reporter asked Attorney Finch for a comment on the case, Finch replied, "I'm sure the only one guilty of anything here is the media. Everyone knows my client is innocent, and that the police framed him." Were Finch's comments proper under the Model Rules? (A) Yes, because it was unlikely to have a materially prejudicial effect on an adjudicative matter (B) Yes, because a lawyer may make a statement that a reasonable lawyer would believe is required to protect a client from the substantial undue prejudicial effect of recent publicity not initiated by the lawyer or the lawyer's client (C) No, because a lawyer should not publicly express any opinion as to the guilt or innocence of a defendant or suspect in a criminal case or proceeding that could result in incarceration (D) No, because a lawyer participating in a criminal proceeding shall not make any extrajudicial statement that the lawyer knows or reasonably should know will be disseminated by means of public communication

(C) No, because a lawyer should not publicly express any opinion as to the guilt or innocence of a defendant or suspect in a criminal case or proceeding that could result in incarceration The correct answer is C. Comment 5 to Model Rule 3.6 lists several types of statements or topics that lawyers should avoid when talking to the media about pending litigation and includes "any opinion as to the guilt or innocence of a defendant or suspect in a criminal case or proceeding that could result in incarceration." In fact, Comment 5 is one of the few places in the Comments to the Model Rules that specifies the amount of weight that a disciplinary authority should give to these factors—that they are "more likely than not to have a material prejudicial effect on a proceeding." This means that these types of comments would presumably outweigh lesser countervailing factors, such as the need to rebut preexisting media attention (Comment 7). It seems counterintuitive to many students that defense lawyers should not argue for their own client's innocence in the media, but that is what the Comment says (note there is a difference between the lawyer reporting his client's position—"my client maintains that he is innocent"—and phrasing it as the lawyer's own opinion about the client's innocence). A lawyer publicly insisting on his confidence in his own client's innocence could backfire when the jury later hears some evidence of the client's guilt. It could also narrow the defense lawyer's options later in terms of recommending that the client plead guilty to lesser charges, which might end up being the client's best option. Answer C is the best answer here—Attorney Finch should not have expressed his own opinion about his client's guilt or innocence. Answer A is incorrect because Comment 5 maintains that such statements are indeed "more likely than not to have a materially prejudicial effect on the proceeding." Answer Bmay be the second-best answer—in other situations, Comment 7 would apply and would permit the lawyer to make statements necessary to rebut negative media publicity about the defendant, but the language in Comment 5 says that declaring a defendant to be guilty is one of the items that is likely to have a prejudicial effect, thereby creating a presumption that the lawyer's statement still appears to violate the ethical rules. Answer D wrongly states a categorical ban on lawyers talking to the media; the Supreme Court previously struck down such sweeping restraints on lawyer speech on First Amendment and void-for-vagueness grounds.

Two co-plaintiffs in a personal injury lawsuit hired Attorney Johnson to represent them in the matter. The litigation promised to become very complex, with multiple issues pertaining to liability and multiple potential defendants. Attorney Johnson had each client sign a detailed "waiver of present and future conflicts of interest" form and carefully explained the specific types of conflicts that can arise between co-plaintiffs in tort litigation, such as indemnification claims, cross-claims, adversarial positions in response to counter-claims from defendants or third-party interveners, and so on. In addition, Attorney Johnson encouraged both clients to consult with separate lawyers before consenting to the conflicts, and both did so. Moreover, both plaintiffs were themselves lawyers and were very familiar with the potential conflicts of interest that could arise from this common representation. Attorney Johnson reasonably believed that she would be able to provide competent and diligent representation to each affected client. As the trial date approached, a counter-claim by one of the defendants forced one plaintiff to file a cross-claim for indemnification against the other. Will the written, informed consent to potential conflicts that each client signed be effective in this situation, so that Attorney Johnson does not have to withdraw from the representation? (A) Yes, because both clients are lawyers and have sophisticated knowledge of potential conflicts of interest that could arise, and they consented in writing to the dual representation (B) Yes, because each client had the benefit of independent counsel advising them about the risks of consenting to such conflicts of interest in litigation, and Attorney Johnson carefully informed them as well (C) No, because new circumstances materialized that make the conflict nonconsentable, namely, the assertion of a claim by one client against another client represented by the lawyer in the same litigation (D) No, because it violates the Rules of Professional Conduct for a lawyer to ask a client to waive future claims such as a conflict of interest, and a lawyer cannot withdraw from representation in the middle of litigation

(C) No, because new circumstances materialized that make the conflict nonconsentable, namely, the assertion of a claim by one client against another client represented by the lawyer in the same litigation The correct answer is C. This question is tricky because the facts portray the lawyer seemingly doing everything right to obtain valid consent to future conflicts of interest—not only did the clients receive adequate explanation, but the clients themselves were lawyers and were represented by lawyers—providing three layers of protection against any misunderstanding. The attorney did nothing wrong with obtaining consent beforehand. The problem is that the specific conflict that arose—making the clients adverse parties in the same litigation, with the same lawyer representing both of them—is not "consentable" according to Model Rule 1.7. When the lawyer is in the position of being both the plaintiff's and defendant's lawyer in a lawsuit, it does not matter if the lawyer obtained informed written consent from the clients—this is a "nonconsentable conflict." (The ABA Model Rules use the terms "consentable" and "nonconsentable" in the official Comment to Rule 1.7.) Comment 22 to Rule 1.7 mentions this problem, when it notes, "advance consent cannot be effective if the circumstances that materialize in the future are such as would make the conflict nonconsentable under paragraph (b)." This means that Answer C is the correct answer to this question—even though the lawyer validly obtained informed consent in writing from the clients to specific future conflicts of interest, the counter-claim by one client against the other rendered the lawyer's conflict of interest nonconsentable. Answer A is incorrect because it ignores the fact that the conflict became nonconsentable, although normally it would be very relevant that the clients were themselves lawyers. Similarly, Answer B focuses on a fact would be relevant in most situations—that the clients each had independent counsel advise them about the waivers—but when a nonconsentable conflict arises, consent from a client is not enough, regardless of the factors that would otherwise matter. Answer D is wrong because it incorrectly states the rule too broadly—lawyers can, in fact, sometimes obtain valid waivers for future conflicts of interest, if the conflicts are otherwise consentable.

A lawyer agreed to represent a tenant who was facing eviction for nonpayment of rent. The lawyer formalized his representation agreement with the tenant and filed an appearance in the local housing court. The court docket had the tenant's hearing scheduled for one month later. Four days after filing his appearance, the lawyer received a phone call from the tenant saying she no longer wanted him to represent her because she wanted to represent herself instead. She conceded that he had done nothing wrong. The lawyer tried to persuade her to change her mind, but she was insistent so the lawyer said he would send her all the documents from her case. The lawyer then drafted a letter acknowledging the termination of representation and sent it along with copies of the court documents he had pertaining to the client's case. The letter was returned three days later to the lawyer, marked "UNDELIVERABLE: Not at This Address." The lawyer tried calling the client, but her phone number was no longer in service. On the date of the tenant's scheduled hearing, the lawyer appeared in person to notify the judge that the tenant had discharged him and to seek to withdraw from the case. The tenant did not appear at the hearing. The judge refused to permit the lawyer to withdraw from the case, and ordered him to proceed with the representation, because otherwise a default judgment would enter against the tenant for failure to appear. The hearing then proceeded as scheduled, in the tenant's absence, with the lawyer presenting the same defense for nonpayment of rent that he would have presented if client had not discharged him. Ultimately, the court ruled in favor of the landlord and ordered the eviction of the tenant. Was it improper for the lawyer to represent the tenant at the hearing, even after the tenant had discharged him? (A) Yes, because a client has a right to discharge a lawyer at any time, with or without cause, subject to liability for payment for the lawyer's services (B) Yes, because the lawyer failed to ensure that the client had received proper documentation of the discharge and had failed to notify the court before the hearing of the termination of representation (C) No, because the court refused to grant the lawyer to withdraw from the case, despite the client's attempt to discharge the lawyer (D) No, because the court ruled against the tenant anyway, so the lawyer's representation did not make any difference either way

(C) No, because the court refused to grant the lawyer to withdraw from the case, despite the client's attempt to discharge the lawyer The correct answer is C. Model Rule 1.16(c) includes a provision regarding this type of scenario: "When ordered to do so by a tribunal, a lawyer shall continue representation notwithstanding good cause for terminating the representation." Judges have broad authority and discretion over the proceedings in their own courtrooms, including the lawyers who appear in their court. Judges usually allow lawyers to withdraw from representation—especially if the client has purportedly discharged the lawyer—but in some instances, a court may order a lawyer to continue for the time being to preserve the integrity of the proceedings. Note that clients normally have almost absolute authority to discharge their own lawyer, but in the facts here, the client was not present to confirm the discharge or state on the record a preference for representing herself. Answer C, therefore, is the best answer in this case—if a judge orders a lawyer to continue with the representation, the lawyer should do so, even against the client's wishes. At the least, the lawyer would not be violating the rules if he complied with the court's order and continued with the representation. In fact, a lawyer who refuses to comply with a court order could be held in contempt and face sanctions from the court. Answer A is not the best answer, although it is true that the client normally has the right to fire a lawyer. Even so, from the lawyer's standpoint, it is safer to comply with the court's order, in terms of what is proper. Answer B is incorrect because a lawyer does not necessarily have to notify the court in advance of a decision to withdraw from the case. In addition, it appears the lawyer in this problem did make reasonable efforts to provide the client with documentation of the discharge. Answer D is incorrect because the outcome of the case, or its inevitability, does not determine whether a lawyer may stop representing the client, though it would be one relevant factor if the problem were about the lawyer withdrawing, as opposed to the client discharging the lawyer and the court ordering the lawyer to continue. The ultimate outcome of the case is not a factor once the judge has ordered the lawyer to finish the proceedings at hand.

An attorney represented a medium-sized company in an enforcement action brought by the National Labor Relations Board (NLRB) over violations of the laws protecting unionized workers. The attorney met with the employees of his client, in groups of four or five at a time, and explained that there was litigation pending, that government lawyers are representing the NLRB, and that they should decline to discuss the case with anyone, especially lawyers from the government. Was it proper, under the Model Rules, for the attorney to ask the employees not to talk to the other party? (A) Yes, because each of those individuals is still free to ignore the lawyer and talk to whomever they want about the case or about the company (B) Yes, the Rules of Professional Conduct permit a lawyer to advise employees of a client to refrain from giving information to another party, for the employees may identify their interests with those of the client (C) No, because the lawyer could not reasonably believe that the employees' interests will not be adversely affected by refraining from giving such information (D) No, because the Rules of Professional Conduct require a lawyer to encourage every potential witness to talk openly and honestly with the lawyers on both sides of the case

(C) No, because the lawyer could not reasonably believe that the employees' interests will not be adversely affected by refraining from giving such information The correct answer is C. Model Rule 3.4(f) generally prohibits lawyers from asking any potential witnesses to withhold information from the opposing party. Important exceptions, delineated in Rule 3.4(f)(1), are family members or employees/agents of the client. Normally, a lawyer may ask these individuals, due to their special relationship with the client, to refrain from disclosing information to the other side in litigation. Yet there is a qualification added in Rule 3.4(f)(2)—one might call it the exception-to-the-exception—that lawyers may not ask family members and employees/agents to withhold information unless "the lawyer reasonably believes that the person's interests will not be adversely affected by refraining from giving such information." In this question, the employees, as members of the union, are effectively the opposing party in the dispute, because the dispute is about protecting the legal rights of the workers. Asking the workers to refrain from discussing the case with lawyers from the NLRB goes against the workers' own interests in the immediate legal dispute. Thus, Answer C is correct—the lawyer could not reasonably have believed that it would be in the employees' own best interest to refuse to talk to the government agency intervening to protect their rights against the management. The lawyer violated Rule 3.4 by asking the employees not to talk to the lawyers from the government in this case. Answer Ais incorrect because the fact that individuals could ignore the lawyer's request does not get around the rule. In fact, in most situations where a lawyer would ask someone to withhold information from the other party in litigation, the individual would be free to disregard the lawyer's request, and the lawyer would still be in violation of the rules even if the person disclosed the information and testified on behalf of the opposing party. Answer B uses the language of the usual exception to Rule 3.4(f) for employees (the Comment to the rule notes that often the employees' interests in fact align with the employer's interests), but Answer B ignores the exception-to-the-exception found in Rule 3.4(f)(2). The employees' interests are clearly adverse to those of the corporate client (this is a dispute that required intervention by the NLRB). Answer D incorrectly asserts that lawyers must encourage potential witnesses to share information with the opposing party. Lawyers cannot ask someone to refrain from sharing information, but they have no obligation to encourage individuals to disclose anything, either.

After being served with process in a personal injury action, a business owner made an appointment with Attorney Davis, a local lawyer. The business owner met with Attorney Davis and explained that she needed defense counsel for the personal injury suit. Attorney Davis explained that she only handled commercial real estate litigation and probably had too many cases pending to agree to handle another matter, especially one involving an unfamiliar type of litigation. Nevertheless, Attorney Davis said she would look into it, think it over, and let the business owner know if she could provide the representation but that she was disinclined to take the case. The business owner left the firm and notified the plaintiff's lawyers that they should direct all further communication to Attorney Davis. Has an attorney-client relationship been formed between the business owner and Attorney Davis? (A) Yes, because Attorney Davis did not explicitly decline to represent the business owner, leaving the possibility open (B) Yes, because the business owner's understanding at the conclusion of the consultation was that Attorney Davis would function as her advocate in the matter, at least for the time being (C) No, because there was no express agreement to provide any representation, and it was not reasonable for the business owner to think that Attorney Davis would represent her (D) No, because even though Attorney Davis tentatively agreed to consider the representation, the representation was not confirmed in writing

(C) No, because there was no express agreement to provide any representation, and it was not reasonable for the business owner to think that Attorney Davis would represent her The correct answer is C. It is important to assess whether an attorney-client relationship exists before proceeding to evaluate the lawyer's ethical behavior. Sometimes, there is ambiguity about whether the lawyer has agreed to represent the prospective client, as well as situations where a prospective client decided not to hire the lawyer, but the lawyer misunderstands the client and proceeds with the representation anyway. This fact pattern describes the former scenario, where a lawyer did not actually agree to provide representation. Answer C is correct because Attorney Davis never agreed to represent the business owner, and it was not reasonable for the business owner to presume that Attorney Davis would do anything, or to tell opposing counsel that Attorney Davis was the defense counsel for the matter. Answer A is incorrect because the burden is not on the lawyer to make an express refusal of representation—the question is whether a reasonable person in the prospective client's situation would have thought Attorney Davis agreed to proceed. Answer B is incorrect because the prospective client's subjective impression is not the only factor in these situations—rather, it is whether a reasonable person might have thought that the lawyer was providing representation. Answer D is incorrect because an agreement in writing is not necessarily required. Even when nothing is in writing, a lawyer could create an attorney-client relationship with an oral promise to provide representation in the matter.

Attorney Addison was a second-year associate at Big Firm, under the direct supervision of a partner there. The partner assigned a new client matter to an associate, instructing him to draft and file a product liability lawsuit on the client's behalf against Conglomerate Corporation. As the associate proceeded to work on the matter, however, he realized that the client had no basis in law or fact for the claim. Conglomerate Corporation was not the manufacturer or seller, and the type of product came under one of several federal statutes that shield manufacturers from product liability. The associate approached the partner about his concerns, but the partner told him to file the claim anyway, because the client was a longtime client of the firm and because there was a chance that Conglomerate Corporation would offer to settle the matter quietly without contesting the claim in court, in order to save legal costs. Would it be proper for the associate to proceed as the partner instructed? (A) Yes, because subordinate lawyers may follow the instructions of their supervisors, if the supervisor is a lawyer, as supervisory lawyers bear the responsibility for any ethical violation that they order subordinates to commit. (B) Yes, because the reasons that the partner gave for proceeding make this an arguable question under the ethical rules, and the partner's solution is reasonable. (C) No, the associate must follow the requirements of the ethical rules regardless of the directions of a client or supervisor, and the lawsuit in this case is frivolous. (D) No, because it is unethical for a lawyer to file a lawsuit in hopes of obtaining a quick settlement from an opposing party who needs to avoid costly litigation.

(C) No, the associate must follow the requirements of the ethical rules regardless of the directions of a client or supervisor, and the lawsuit in this case is frivolous. ABA Model Rule 5.2(a) provides, "A lawyer is bound by the Rules of Professional Conduct notwithstanding that the lawyer acted at the direction of another person." Similarly, the Restatement (Third) of the Law Governing Lawyers § 12 states that "a lawyer must conform to the requirements of an applicable lawyer code even if the lawyer acted at the direction of another lawyer or other person," paraphrasing Model Rule 5.2. Here, the lawsuit is frivolous because it has no basis in fact or law, and it is therefore unethical for the associate to file it (see Model Rule 3.1), given that the associate knows of the problem. Answer A is incorrect, even though the partner would indeed share in the responsibility or liability for the ethical violation, because Model Rule 5.2 requires subordinates to exercise independent judgement regarding clear ethical obligations. In other words, there is no "Nuremburg defense" for subordinate lawyers. Answer B is incorrect, even though Rule 5.2(b) permits subordinates to follow instructions in arguable cases where the supervisor's solution is "reasonable," because in this scenario, the ethical question is not arguable—filing the lawsuit clearly violates Model Rule 3.1. There is no exception for "reasonable" solutions from supervisors in clear cases, only in arguable or close cases. Answer D is wrong because it overstates the rule—if the case indeed was meritorious, having some basis in law and fact, then it would not be unethical to file it even if the client hopes to obtain a quick settlement due to the defendant's aversion to litigation.

An attorney previously represented a developer in securing environmental and construction permits to build an apartment building. Various government agencies granted the necessary permits. Funding shortfalls unrelated to the attorney's representation delayed the construction of the apartment building after the attorney's representation of the developer ended. Eventually the developer completed the apartment building, decided to manage the property instead of selling it, and leased 80% of the units. Three years later, one of the tenants was unable to pay rent for her unit for two consecutive months, so the property manager commenced eviction proceedings. The tenant hired the same attorney to represent her in the eviction proceedings. The apartment's owner filed a motion to have the attorney disqualified due to the substantial relationship between his previous work in securing environmental permits for the building and the present eviction action against the tenant. Should the court grant the motion to disqualify the attorney from representing the tenant in this matter? (A) Yes, because the confidential information the attorney learned while securing permits prior to construction would certainly be substantially related to the nonpayment of rent by a tenant in the same building later on (B) Yes, because the lawyer previously represented the developer who constructed the apartment building and is now representing a party with directly adverse interests to the former client (C) No, the matters are not substantially related because they do not involve the same transaction or legal dispute, and confidential information learned while obtaining construction permits prior to construction are unrelated to the nonpayment of rent by a tenant sometime later (D) No, because as a public policy matter, it is difficult for renters to find and afford legal representation, especially when facing something as potentially devastating as an eviction

(C) No, the matters are not substantially related because they do not involve the same transaction or legal dispute, and confidential information learned while obtaining construction permits prior to construction are unrelated to the nonpayment of rent by a tenant sometime later The correct answer is C. This question presents a situation in which the connection between the matters handled by the same lawyer is too attenuated to be substantially related. See Comment 3 to Model Rule 1.9. This question also highlights a difference between the conflict of interest rules for current clients versus successive clients—a lawyer would probably have a conflict of interest that would require informed consent of both clients if the developer were still a client, but not if the developer is a previous client. Answer C is the correct answer because the work of securing environmental and construction permits is not related at all to the eviction case against a tenant years later for nonpayment of rent. The result might be different if the tenant had stopped paying rent because of flooding or some environmental issue, but the question presents merely a nonpayment eviction case. The lawyer can therefore represent the tenant against the former client because the matters have no legal or factual overlap. Answer A is wrong because it suggests the exact opposite—that the two matters are closely related, when it is obvious that the tenant's nonpayment of rent and eviction matter having nothing to do with the application for government permits before construction began. Answer B is incorrect, even though it is true that the current and former clients have directly adverse interests, because the developer is a former, not a current client. Thus, there would have to be substantially related matters in the representation of each client in order for the lawyer to have a real conflict of interest, which is not the case here. Answer D wrongly focuses on policy concerns for the economic disadvantage faced by many renters in eviction cases; while this concern might underlie some of the substantive law in the area of landlord-tenant relations (such as statutes proscribing eviction procedures), such a policy concern is not relevant to the analysis of the lawyer's conflicts of interest and would not be a consideration in a motion for disqualification.

An attorney works as in-house counsel for a large international corporation and has daily contact with higher-level executives and managers. One day, a senior executive mentions casually to the attorney that he has offered lucrative stock options, worth millions of dollars, to a foreign government official who has agreed to give the corporation an exclusive contract to provide certain goods and services to the foreign state. The executive seems to think this is normal and good for the company, but the attorney believes it constitutes bribery of foreign officials, which would violate the Foreign Corrupt Practices Act (FCPA) and could subject the corporation to enormous fines and penalties. The attorney explains her concerns to the executive, including that the executive could face personal criminal charges in addition to bringing liability on the corporation, and she reminds him that she represents the corporation, not him personally. The executive is dismissive of her concerns, even though she approaches him several times about the matter. How must the attorney proceed? (A) She should report the matter immediately, in writing, to the Department of Justice and tell no one in the company that she has done so. (B) She should keep her conversations with the executive confidential but try to document everything that she knows about the situation in case the Department of Justice brings an enforcement action. (C) She should approach the executive's immediate corporate superior, advising those next up the chain of authority to stop the transaction and take appropriate actions against the executive. (D) She should immediately notify the company's Board of Directors, advising them about the potential liability, and threaten to report the activities to the Department of Justice if they take no action.

(C) She should approach the executive's immediate corporate superior, advising those next up the chain of authority to stop the transaction and take appropriate actions against the executive. The correct answer is C. In this question, the attorney has already taken the appropriate first step in addressing wrongdoing that could subject the organizational client to millions of dollars in fines and other penalties under federal law (the FCPA). That first step was to confront the individual wrongdoer, the executive, and try to persuade him to remedy the situation immediately—but this has proved fruitless. Model Rule 1.13, with an associated Comment, instructs lawyers in that case to proceed to the next step up the corporate chain of command. Answer C is therefore correct. Answer A wrongly suggests that the lawyer should skip directly to the whistleblower stage, that is, turning the executive over to government enforcement officials in order to save the corporation. This action would be valid only as a last resort, after exhausting every other option within the organization. Answer B, in contrast, suggests a course of action that could put the corporation in jeopardy, and therefore would violate the lawyer's duty to protect the corporate client's interests. Answer D is wrong for the same reason as Answer B—this course of action skips some intermediate steps the lawyer should take in this situation before going to the top of the organization, that is, the Board of Directors.

An attorney represented a criminal defendant in a murder prosecution, in which the defendant could receive the death penalty if convicted. The attorney assiduously pursued pretrial discovery and motions to exclude prosecution evidence. Nevertheless, the defendant confessed to the murder, against the attorney's specific advice. A sense of hopelessness set in for the attorney, and he began to curtail his efforts in the representation. The situation deteriorated even further when the defendant again ignored his attorney's advice and waived his right to a jury trial, and then entered a guilty plea to all charges, including the capital murder charge. The attorney then instructed the defendant to request an advisory jury at his capital sentencing hearing, which was permissible in that jurisdiction, but the defendant again rejected the advice and opted for sentencing by the trial judge without a jury recommendation. At this point, the attorney made only token efforts to prepare for the sentencing hearing, and he gathered no evidence regarding the defendant's character or emotional state. The defendant himself argued for leniency based on his lack of prior convictions, but this proved unsuccessful, and the judge sentenced him to death. On appeal, the defendant claimed that he had received ineffective assistance of counsel. Which of the following best describes the test an appellate court would use in deciding whether to grant postconviction relief? (A) The defendant must show that his counsel did not use the best possible strategy to refute the evidence of guilt and that that there is a reasonable probability that, but for counsel's unprofessional errors, the result of the proceeding would have been different, that is, it undermines confidence in the outcome. (B) The defendant must only show that his lawyer's performance did not meet the highest standards of excellence in the legal profession, and that there is some possibility that his lawyer's "less-than-the-best" performance impacted the ultimate outcome of the case, including the sentence received and results on appeal. (C) The defendant must show that counsel's representation fell below an objective standard of reasonableness, using a strong presumption that counsel's conduct falls within the wide range of reasonable professional assistance, and the defendant must show that there is a reasonable probability that, but for counsel's unprofessional errors, the result of the proceeding would have been different, that is, it undermines confidence in the outcome. (D) The defendant must show that counsel's representation fell below an objective standard of reasonableness, using a strong presumption that counsel's conduct falls within the wide range of reasonable professional assistance, and the defendant must show that he might have received a better result if his attorney had applied more effort, or if another attorney had represented him.

(C) The defendant must show that counsel's representation fell below an objective standard of reasonableness, using a strong presumption that counsel's conduct falls within the wide range of reasonable professional assistance, and the defendant must show that there is a reasonable probability that, but for counsel's unprofessional errors, the result of the proceeding would have been different, that is, it undermines confidence in the outcome. Answer C is correct. The facts in this Question come (loosely) from the landmark Supreme Court decision on the subject, Strickland v. Washington, 466 U.S. 668 (1984). Note that this Question oversimplifies the facts or the original case—in Strickland, the defendant was facing three separate counts of murder and numerous other charges, and the procedural history was labyrinthine. Courts now apply the two-part Strickland test in all ineffective assistance of counsel appeals. See, e.g., Sexton v. Beaudreaux, 138 S. Ct. 2555 (2018); Buck v. Davis, 137 S. Ct. 759 (2017). The defendant must first show that counsel's performance fell below that of a reasonable lawyer and must overcome a presumption of that counsel's performance was reasonable. Courts should avoid the skewed perspective of hindsight and should not second-guess judgment calls or strategic decisions merely because they were arguable or turned out to be unavailing. In addition, the defendant must show a high probability that the attorney's incompetence affected the trial outcome—and this is the point on which most appellants lose. In other words, even if the trial lawyer was egregiously incompetent or ineffective, an appellate court will uphold a conviction if it believes that the trial court would have convicted the defendant anyway, that is, with a decent lawyer. Given the "probable cause" burden of pretrial indictment and other prosecutorial selection effects, most murder convictions will indeed have a record showing robust evidence of guilt, and this means most defendants will lose on the second prong of Strickland, even if their attorney's performance was deficient. Answer A incorrectly states the first prong of the test—a defendant must show far worse "ineffectiveness" of counsel than merely being "less than the best." A defendant does not necessarily have a constitutional guarantee of a lawyer who always uses the best possible strategy, but instead, a lawyer who uses a minimally competent strategy. Similarly, Answer B misstates the first prong—again, "ineffective assistance" means something more like "far below average," not simply "imperfect." Answer B also misstates the second prong of the test—the threshold is not "some impact" but rather "reasonable probability" of prejudicial effect. Finally, Answer D has the second prong of the test incorrect (too low of a threshold), even though it has the first prong right.

A client committed perjury on the witness stand during his trial, but her attorney did not know it at the time. The client won her case and there was no appeal of the verdict. Sometime later, the client boasted to her attorney that she successfully lied to the court and won the case as a result. Does the attorney have an ethical duty to disclose to the tribunal that the perjury occurred? (A) Yes, because when a lawyer represents a client in an adjudicative proceeding and knows that a person has engaged in fraudulent conduct related to the proceeding, the lawyer shall take reasonable remedial measures (B) Yes, because if a witness called by the lawyer has offered material evidence and the lawyer comes to know of its falsity, the lawyer shall take reasonable remedial measures, including, if necessary, disclosure to the tribunal (C) No, unless the judge at some later time specifically asks the lawyer if his client committed perjury (D) No, because a lawyer's duty to take remedial measures after perjury occurs continues only to the conclusion of the proceeding

(D) No, because a lawyer's duty to take remedial measures after perjury occurs continues only to the conclusion of the proceeding The correct answer is D. Model Rule 3.3c provides that the duties of candor under Rule 3.3(a) and (b) continue to the conclusion of the proceeding. Comment 13 to Model Rule 3.3(c) explains that the "conclusion of the proceeding is a reasonably definite point for the termination of the obligation." Furthermore, the Comment notes that a proceeding has concluded "when a final judgment in the proceeding has been affirmed on appeal or the time for review has passed." While other rules, such as the rule of client confidentiality, continue indefinitely, even after the representation ends, the duty of candor to the tribunal ends when the case is final. Apart from some special rules for prosecutors who learn of wrongful convictions, lawyers do not have an ethical duty to correct false evidence if they learn of its falsehood only after the proceedings conclude. Answer D is therefore the correct answer—the lawyer has no duty to disclose the perjury to the tribunal if the verdict is final and there are no further appeals. Note, however, that if the client's perjury comes to light some other way, the client could still face criminal charges for perjury. The terminal point of the lawyer's ethical duty of disclosure does not mean the client is immune from prosecution later. Answer A states the general rule for candor to the tribunal but ignores the fact that the rule does not apply after the conclusion of the proceedings. Answer B is wrong for the same reason, even though it correctly states a provision of Rule 3.3 that compels the lawyer to make disclosures that would otherwise be impermissible under the client confidentiality rules. Answer Boverlooks the fact that the proceedings have concluded, which triggers the provision in Comment 13 about the duty of candor terminating at that point. Answer C is tricky because in other situations a lawyer would have to answer questions from the judge truthfully, even then the lawyer had no duty of disclosure otherwise.

On a contingent fee basis, a criminal defense lawyer offered to represent a defendant charged with murder. The client would pay only if the lawyer won an acquittal and would pay nothing if the case resulted in a conviction. The client was eager to do this and consented to the arrangement, in writing. Which of the following best describes the lawyer's situation? (A) The lawyer is not subject to discipline because the client consented and confirmed it in writing. (B) The lawyer is subject to discipline for failing to include a third option, a partial fee if the case ends with a plea bargain to a lesser charge that requires no jail time. (C) The lawyer is subject to discipline for charging a contingent fee in a criminal matter. (D) The lawyer's arrangement would constitute ineffective assistance of counsel in a criminal case.

(C) The lawyer is subject to discipline for charging a contingent fee in a criminal matter. The correct answer is C. The Model Rule 1.5 includes a mix of general guidelines and a few specific, bright-line rules related to fees. One of the specific, bright-line rules is that contingent fees are not permissible in criminal cases—a lawyer cannot offer to represent a criminal defendant on the basis that the fee applies only if the lawyer obtains an acquittal (or a lighter sentence). Answer C accurately states the rule. Answer A is incorrect because the client's consent does not matter for certain fee arrangements that the Model Rules expressly prohibit. Answer B proposes an alternative arrangement (a hybrid fee accounting for plea-bargain results) that would also be impermissible under the Rules because it still would have a contingent-fee component in a criminal matter. Answer D is wrong, partly because contingent fees and other violations of specific prohibitions do not necessarily constitute ineffective assistance of counsel. The Supreme Court has held that the ethical violation would need to be egregious (something no competent lawyer would do), and that the lawyer's misconduct probably affected the trial outcome.

Attorney represents Client in a litigation matter. Client was not present during the last pre-trial hearing at which the lawyers argued about whether certain experts on each side could testify at trial. The trial was to start the following week. At the end of the hearing, the opposing counsel asked the court to have the record sealed in the upcoming trial and to have reporters banned from the courtroom. He explained that the testimony at trial would necessarily reveal some of his client's trade secrets, and it was important to the client to keep the trial records sealed. The judge was amenable to this suggestion and asked Attorney if he had any objections. Attorney tried to call Client, but Client did not answer his phone right then. Attorney agreed to the sealing because he could not think of a compelling reason for Client to oppose the motion. The judge set the matter for a sealed-record trial. Client never returned Attorney's call, and Attorney did not explain what had transpired until they arrived at the court for the first day of trial. Client was dismayed because he had planned to use this litigation as a test case for subsequent litigation over the same type of issue, but Attorney explained that it would now be difficult to get the judge to reverse course on this point. Is Attorney subject to discipline in this case? (A) No, because the opposing party's request was reasonable, and even if Attorney had asked Client and Client disapproved, Attorney could not have ethically objected to the request (B) No, because it was proper for the lawyer to defer to the judge on this question, lest he risk angering the judge or unnecessarily inconveniencing the opposing party (C) Yes, because even when an immediate decision must be made during a hearing or trial, and the exigency of the situation may require the lawyer to act without prior consultation, the lawyer must tell the client about it as soon as possible (D) Yes, because the attorney had a duty to consult the client prior to agreeing to the sealing of the record because of the importance of the action under consideration and the feasibility of consulting with the client

(C) Yes, because even when an immediate decision must be made during a hearing or trial, and the exigency of the situation may require the lawyer to act without prior consultation, the lawyer must tell the client about it as soon as possible The correct answer is C. Sometimes, attorneys must make decisions during a trial or hearing when the client is absent and there is no time for consultation beforehand. Even though the exigent circumstances justify the lawyer going ahead and making a decision in these situations, the lawyer must notify the client about what transpired (and the decision the lawyer made without the client) as soon as possible. Answer C correctly states this principle, which is found in Comment 3 to Model Rule 1.4. Answer A incorrectly focuses on the reasonableness of the decision—it was indeed reasonable for the lawyer to make that choice in the client's absence—but ignores the fact that the lawyer still has an ethical duty to notify the client about it as soon as possible. Answer B is wrong for a similar reason—certainly it was appropriate for the lawyer to defer to the judge, but that does not relieve the lawyer of the duty to tell the client about it so that the client is informed. Answer D, on the other hand, incorrectly states that the lawyer impermissibly made the decision without the client in this situation. It was permissible for the lawyer to make the immediate decision during trial—the lawyer then had a duty to contact the client as soon as possible thereafter.

An employee in the sales and marketing department of a large corporation experienced harassment from a supervisor and was (wrongly, she believed) demoted to a lower position. She retained counsel and filed a claim for workplace discrimination. While preparing for a trial, her attorney decided to contact some night shift workers in the corporation's offsite warehouse to learn about the day-to-day operations of the corporation. The attorney was hoping to hear some revealing gossip that would uncover a larger pattern of discrimination by corporate management, and by the corporation's human resources department. The attorney did this without permission from the corporation's lawyer. If asked, the defendant corporation's lawyer would certainly have forbidden it and would have told the warehouse workers not to talk to the plaintiff's attorney at all. Was this communication by the plaintiff's attorney proper? (A) No, because the acts or omissions of the workers may be imputed to the organization for purposes of liability (B) No, because he is mostly looking for gossip about the company's management, which could only serve to embarrass the managers or marketing department at trial (C) Yes, because the warehouse workers are not involved in the matter, do not report directly to the firm's in-house counsel, and lack authority to bind the organization in the matter (D) Yes, because he knows that opposing counsel would improperly forbid the warehouse workers from talking to him, so the Model Rules allow him to take action to counteract this inappropriate potential behavior by the corporation's lawyer

(C) Yes, because the warehouse workers are not involved in the matter, do not report directly to the firm's in-house counsel, and lack authority to bind the organization in the matter The correct answer is C. Model Rule 4.2 forbids lawyers from communicating about the subject of the representation with the opposing party who is represented by counsel in the matter, unless opposing counsel gives consent, or a law or a court order specifically authorizes the communication. Comment 7 to Rule 4.2 adds a clarifying provision for situations where the opposing party is an organization, such as a corporation: the lawyer may not communicate with "a constituent of the organization who supervises, directs or regularly consults with the organization's lawyer concerning the matter or has authority to obligate the organization with respect to the matter," or whose act or omission in connection with the matter may be imputed to the organization for purposes of civil or criminal liability. In this question, it would be a violation of the rules for the lawyer to contact the corporate executives or managers who have authority to make decisions on behalf of the organization, and whatever individual supervises the lawyer who represents the organization in the matter. If the corporation has hired outside counsel to handle the litigation, normally the person supervising those lawyers would be the corporate in-house counsel. If the in-house counsel is handling the litigation itself, this would be the corporate manager (say, a president or vice president) who supervises the lawyers there. The attorney in this question, however, did not contact those individuals; instead, he contacted the warehouse workers who presumably have no decision-making authority in the organization. Thus, Answer C is the best answer—the attorney acted properly and within the boundaries proscribed by Rule 4.2 and Comment 7. Answer Ais incorrect because opposing counsel's disapproval does not matter if the contacted individual is outside the scope of the prohibition, as in this question. Note that the corporation's lawyer is free to instruct these warehouse workers not to talk to the plaintiff's attorney, but corporate counsel cannot prevent the attorney from trying to engage these workers in conversation. Answer B wrongly suggests that the lawyer's intentions determine the outcome of the rule. Rule 4.2 does not contain any provisions about the attorney's subjective intentions, except for excusing lawyers who did not know (had no reason to know) that the opposing party had hired counsel. In addition, it is proper for an attorney to do some investigation to explore the possibility of expanding the claim. Answer D is also incorrect because it is appropriate for opposing counsel to ask all the employees of the corporation to stop talking to other lawyers, and if there were a violation of the rule (say, if the attorney had talked to someone in senior management in the marketing department), the other lawyer's actions would not create a justification.

Attorney Avery represented a defendant in a criminal case involving serious felony charges. After the defendant rejected all proffered plea bargains from the prosecutor, he insisted upon a jury trial and volunteered to testify at his trial to assert his innocence. Attorney Avery knew that it would be a strategic mistake for his client to testify, for several reasons. First, the defendant had initially confessed to the crime, but Attorney Avery managed to have the confession excluded due to a technical defect in the Miranda warnings. Avery was now worried that the otherwise excluded confession would become admissible for impeachment purposes if the client testified and presented a contradictory version of the facts. The defendant also had a long record of prior convictions involving fraud and larceny, which would otherwise be inadmissible at trial but would become admissible to impeach the defendant's credibility if he testified. Even worse, Attorney Avery had confidential information that the client committed several related crimes to those charged in the case, and the prosecutor might elicit testimony implicating the client in these additional crimes during cross-examination. The prosecutor in the case was notorious for aggressive cross-examination of witnesses at trial and even teaches special training courses to other litigators on how to conduct merciless, devastating cross-examination. Finally, the client is not very articulate; he constantly uses street slang, gratuitous profanity, and incorrect grammar when speaking. This conduct would likely alienate some jurors. Attorney Avery explained all of this to the defendant, and then reminded the defendant that he already rejected several generous plea offers and would forfeit any chance of winning at trial due to his insistence about testifying. Attorney Avery concluded by saying, "There is no way I will allow you to testify in this case; it would be malpractice on my part." The client understood this to mean that he had no choice, so he gave up. The trial went well and the jury acquitted the defendant of all charges. Would Attorney Avery be subject to discipline under these circumstances? (A) Yes, because he spoke angrily with his client and made unnecessary references to the client's speech mannerisms, showing extreme insensitivity and disrespect; he also deprived the client of a good opportunity to explain his side of the story about all the previous cases in which he received convictions (B) No, because clients have a right to dictate the overall objectives of the representation, but the lawyer has a right to decide the means of achieving that objective, and leaving out the client's testimony shortened the trial time and thereby reduced the legal fees the client would owe to the lawyer (C) Yes, in a criminal case, the lawyer shall abide by the client's decision, after consultation with the lawyer, as to whether the client will testify (D) No, because the defendant suffered no harm from the lawyer's decision, as the jury gave a complete acquittal, and the lawyer was clearly correct in his reasoning about why it would backfire if the defendant testified at trial

(C) Yes, in a criminal case, the lawyer shall abide by the client's decision, after consultation with the lawyer, as to whether the client will testify The correct answer is C. Model Rule 1.2(a) requires lawyers to honor their client's decisions about important constitutional rights in criminal trials, such as the right to a jury trial and right to testify. In this case, the lawyer refused to let the client testify and incorrectly stated that it would be malpractice to allow the defendant to testify at trial. Answer C correctly reflects the provisions in Rule 1.2(a) and its associated Comments—it was appropriate for the lawyer to explain the downside of the client testifying, but he should still have let the client make the decision. Answer A, in contrast, incorrectly suggests that the attorney's violation of the rules was in his angry talk and insensitivity. While lawyers are supposed to treat their clients with respect, normally a lawyer would not be subject to discipline over his demeanor when arguing a point with a recalcitrant client and it was appropriate for the lawyer to explain that the client's mannerisms would alienate many jurors. Answer A also incorrectly suggests that it would have been good to have the client explain his prior convictions—the fact that prior convictions would normally be inadmissible suggests that discussing them at trial would harm the client. Answer B is incorrect, though it correctly states that clients control the objectives of the representation, because there is an absolute rule that clients have the final say about waiving their constitutional rights related to trials. Finally, Answer D wrongly suggests that the outcome determines whether the lawyer would be subject to discipline. The fact that the outcome seemingly validated the lawyer's opinion does not change the fact that he violated the rules by overriding the client's wishes regarding testifying at trial.

Attorney Allison represented a small business owner, Claire, for several years. In her professional opinion, Attorney Allison believed that Claire could validly claim a deduction on state income-tax returns for Claire's internet service charges as a business expense. Nevertheless, the Allison remembered that she had read some published reports and data suggesting that claiming such deductions increased the likelihood of a tax audit. Tax audits are often costly and time-consuming, even for clients with nothing to hide, so Claire expressed dismay upon hearing this. The next day, Claire called Attorney Allison and inquired whether there was a "safe" level of charitable deductions that taxpayers like her could claim without raising any suspicion or scrutiny. Attorney Allison had known Claire since they attended college together, and she knew that Claire never gave more than a dollar at a time to any charities. Under these circumstances, which of the following best describes Attorney Allison's proper course of conduct? (A) Attorney Allison may properly warn Claire about the increased risk of an audit for claiming the business expense deduction, and she may advise Claire of any known "safe" amount for claiming charitable deductions, assuming Allison does not propose that Claire intentionally falsify information about charitable donations. (B) It would be improper for Attorney Allison to tell Claire about the increased risk of an audit for claiming the business expense deduction or to tell Claire about any "safe" amount for claiming charitable deductions, because taxpayers should report only facts on a tax return. (C) Attorney Allison should not tell Claire about the increased risk of an audit for claiming the business expense deduction, because this goes against the client's interest in lowering her tax bill, but she may advise Claire of any known "safe" amount for claiming charitable deductions, as this would help avoid unnecessary taxes. (D) Attorney Allison may properly warn Claire about the increased risk of an audit for claiming the business expense deduction, but she should not advise Claire of any known "safe" amount for claiming charitable deductions, given the reasonable likelihood that Claire plans to falsify information about charitable donations.

(D) Attorney Allison may properly warn Claire about the increased risk of an audit for claiming the business expense deduction, but she should not advise Claire of any known "safe" amount for claiming charitable deductions, given the reasonable likelihood that Claire plans to falsify information about charitable donations. This question describes a scenario similar to that described in one of the Illustrations in the Restatement (Third) of the Law Governing Lawyers § 94, which says that it was appropriate for the lawyer to advise the client about enforcement policy on the first issue (here, internet service as a business expense, though the Restatement uses automobile expenses for its example), but advice about the second issue is not permissible, as a reasonable lawyer in this situation would assume that the client planned to falsely claim charitable donations in order to obtain the tax deductions. The Restatement echoes ABA Model Rule 1.2(d) on this point, which provides, "A lawyer shall not counsel a client to engage, or assist a client, in conduct that the lawyer knows is criminal or fraudulent, but a lawyer may discuss the legal consequences of any proposed course of conduct with a client and may counsel . . . [about the] application of the law." The warning about a legitimate (legal) deduction potentially triggering an audit falls under ABA Model Rule 2.1, which permits lawyers to give advice beyond merely reporting what the relevant laws say. Answer A is incorrect because it is not enough for a lawyer to refrain from proposing the illegal activity itself—the lawyer should refrain from advising the client when the lawyer knows that the client intends to use the lawyer's advice to violate the law but escape detection. Answer B wrongly restricts the lawyer's options too much—Model Rule 2.1, as well as the Restatement, contemplate that a lawyer could warn a client about unpleasant consequences of what would otherwise be a legal course of action for the client. Answer C does not reflect the rules about attorney advice—the attorney should provide the client with a clear picture of the legal (and sometimes nonlegal) consequences of certain actions, even if that means the client pays more (or less) taxes as a result.

An attorney worked for Big Firm for several years, during which he represented Conglomerate Corporation on several matters. At the beginning of representation in each matter, the attorney obtained written informed waiver of future conflicts of interest from Conglomerate, specifically including the possibility that the attorney might later represent the government in unrelated matters adverse to the company. The attorney eventually left Big Firm and went to work for Federal Regulatory Agency, in its enforcement and litigation division. On behalf of Federal Regulatory Agency, the attorney brought an enforcement action against Conglomerate for some very recent regulatory violations that were mostly unrelated to any previous work the attorney had performed for Conglomerate at Big Law. Would a court likely find that this attorney should be disqualified from representation in this matter? (A) Yes, because the attorney's prior experience representing Conglomerate gives him an unfair advantage in the current litigation since he is familiar with Conglomerate's litigation strategies and corporate hierarchy (B) Yes, because as a government lawyer, the attorney will presumably gain confidential government information about Conglomerate that he could use to Conglomerate's disadvantage (C) No, assuming Conglomerate ratifies its earlier waiver of future conflicts of interest (D) No, assuming the appropriate government agency gives its informed consent, confirmed in writing

(D) No, assuming the appropriate government agency gives its informed consent, confirmed in writing The correct answer is D. Model Rule 1.11(d)(2)(i) states that a lawyer shall not "participate in a matter in which the lawyer participated personally and substantially while in private practice or nongovernmental employment, unless the appropriate government agency gives its informed consent, confirmed in writing." The attorney in this problem would otherwise be disqualified from the representation if the relevant government agency did not provide written consent, which it did. Thus, Answer D is correct. Answer A is incorrect because it ignores the fact that the government agency consented to the representation, as required under Rule 1.11. Even though the attorney would normally face disqualification for the reasons described in Answer A, the fact that the government consents to the representation resolves the problem. Answer B is irrelevant to the type of conflict described in this question, though the information learned as a government lawyer would be relevant if the attorney returned to the private sector. The information the lawyer learns subsequent to entering government service does not create a conflict for the client previously represented. Answer C is incorrect because the resolution of the conflict of interest does not depend on the private sector client, but rather the consent of the government entity involved. Furthermore, it is not clear that the consent to something as speculative as future changes in the lawyer's career would be effective, even in cases where Conglomerate's consent mattered.

An attorney represented a small business client in a few transactional matters. None of the attorney's work for the client involved information about the client's finances or assets, so the attorney knew very little about the client's overall income, assets, or insurance coverage. On a wholly unrelated matter, Vick, a tort victim, approached the attorney seeking representation for a negligence lawsuit against the small business client over damage to Vick's expensive car. During the initial interview, Vick gave very few details about the accident or the scope of damages, except to identify the attorney's small business client as the intended defendant and to specify that the incident involved a scraped fender in a parking garage. The attorney believed there was no significant risk that the representation of the small business client would materially limit the attorney's responsibilities to Vick, and vice versa. Because the attorney believed there was no conflict, he did not seek consent from either party, although he mentioned to Vick that he had drafted some documents for the defendant's business, and the small business client would obviously learn about the representation of Vick when the attorney filed the lawsuit. May this attorney proceed with representing Vick in the negligence lawsuit? (A) Yes, because the attorney did not receive any relevant confidential information from either party that he could use against the other in the anticipated litigation (B) Yes, because there is no conflict when a lawyer acts as an advocate in one matter against a person the lawyer represents in another matter, as long as the matters are wholly unrelated (C) No, because the attorney already violated the rules of confidentiality by telling Vick that he had drafted business documents for the client (D) No, because absent consent, a lawyer may not serve as counsel in one matter against a person the lawyer represents in another matter, even when the matters are wholly unrelated

(D) No, because absent consent, a lawyer may not serve as counsel in one matter against a person the lawyer represents in another matter, even when the matters are wholly unrelated The correct answer is D. Conflicts of interest have become the leading cause of legal malpractice lawsuits and can also result in lawyer disqualification at trial (court-ordered removal from the proceedings) or discipline by the appropriate professional authority. Conflicts are also the most heavily tested subject on the Multistate Professional Responsibility Exam. This question sets forth one of the most basic scenarios for a conflict of interest between concurrent clients (ABA Model Rule 1.7)—a lawyer represents one client whose objectives in the matter are directly adverse to those of another client, whom the lawyer already represents on an unrelated matter. Answer D provides the best answer—the attorney cannot sue one of his own clients without first obtaining informed consent in writing from both clients involved. Answer A incorrectly suggests that the question turns solely on whether the lawyer already has confidential information from the other client that would be relevant (that is, prejudicial); while that certainly is an important factor in evaluating potential conflicts, there are also important concerns about the duty of loyalty, client trust, and the potential for the lawyer to have to withdraw abruptly at some point if the conflict problems escalate. Answer B wrongly states that conflicts of interest are not present when the clients' matters are unrelated—the official Comment to Rule emphasizes this point repeatedly. Answer C correctly states that the attorney cannot represent both clients, but the reason it offers is incorrect. Lawyers often must disclose some minimal information about the representation of another client, as was done here, in order to screen for conflicts of interest or to obtain informed consent from both parties when that is an option for proceeding. Note that in this question, it is possible that the attorney could represent Vick if both parties consented to the conflict of interest (in writing), but that option was not one of the answers provided, so Answer D is the best answer.

Attorney Smith, a solo practitioner who recently passed the bar exam, accepted Client's case for a flat fee of $3,000. Attorney Smith's contract included a statement in underlined and bold print that stated the entire fee is non-refundable regardless of the outcome of the case or whether Client continued to retain Smith through the finalization of the case. After several weeks with no movement on the case by Attorney Smith, Client fired Smith and hired other counsel to represent him on this case. Client sent a request in writing for reimbursement of the retainer. Attorney Smith responded to Client by stating the funds are non-refundable and refused to refund Client. Were Attorney Smith's actions proper? (A) Yes, because an attorney may refuse to refund an advanced payment of fees if the contract contained such language and the language was clear and obvious in the contract (B) Yes, because an attorney is not required to refund advanced payments if he or she is fired from a case (C) No, because Client requested a refund in writing and an attorney must refund any unused portion of an advanced fee if the client requests such reimbursement in writing (D) No, because an attorney shall refund unused portions of an advanced payment of fees and provide the client with a detailed listing of fees deducted from the advanced payment, regardless of how long the attorney represents the client

(D) No, because an attorney shall refund unused portions of an advanced payment of fees and provide the client with a detailed listing of fees deducted from the advanced payment, regardless of how long the attorney represents the client The correct answer is D. Clients sometimes fire their lawyers in the middle of the representation and switch to other counsel. In many of these cases, the attorney is holding unused client funds received in an advance payment at the outset of representation. Model Rule 1.5 requires that attorneys return all the remaining unused funds to the client (but not those that cover hours of work the attorney already completed), even if the client was unjustified or unreasonable in terminating the representation. Answer D captures the rule in these cases—attorneys must return any remaining funds from an advanced payment of fees and provide the client with a detailed listing of fees deducted from the advanced payment. This is true regardless of how long the attorney represents the client. Answer A incorrectly states that lawyers can retain advance fees if they contract around the rules. The Rules do not contain any such exception. Although lawyers may stipulate that a portion of the fee is non-refundable, the idea is that the non-refundable portion is for the initial time attorneys spend screening for conflicts of interest, possibly turning away other potential clients to allow time for this client's case, creating client files on the firm's network, and so on—these up-front investments in a new client matter make the non-refundable portion reasonable. In contrast, if the lawyer asked the client for a substantial sum that supposedly would be enough to cover the fees for most or all of the representation, it is unreasonable to keep those fees if the lawyer did not do the anticipated work—even if the client caused the work to stop prematurely by firing the lawyer. Answer B is incorrect because it states the opposite of the rule—lawyers indeed must return unused portions of the fees collected in advance. Answer C is incorrect because it misstates the rule, as if clients must request the refund in writing. The lawyer must refund unused fees even if the client has not requested it.

Attorney Francois is representing two French restaurants that are located across the street from each other. They are the only French restaurants in the area, so they compete for the same customers, and their menus, decor, and prices are very similar. Attorney Francois currently represents both restaurants, one in a dispute with its landlord, and the other in a wrongful discharge lawsuit by a former employee. The restaurants have sued each other in the past, using other lawyers. Attorney Francois had not sought consent from each client to represent its competitor, and when each client learned that Attorney Francois was representing the other, they both expressed concern. The second client eventually lost its wrongful termination lawsuit, and then sued Attorney Francois for legal malpractice, claiming that he had a conflict of interest in the representation. Under the ABA Model Rules, did Attorney Francois have a conflict of interest that would subject him to disciplinary action? (A) Yes, because simultaneous representation of competing economic enterprises, even in unrelated matters, ordinarily constitutes a conflict of interest and requires consent of the respective clients, which Attorney Francois did not obtain (B) Yes, because disciplinary authorities will use a subjective test for determining a conflict of interest, that is, evaluating whether the client would feel betrayed or perceive a conflict in the situation, and could therefore sanction Attorney Francois, but the conflict here does not reach the level of malpractice (C) No, because the fact that one client initiated a malpractice tort claim precludes disciplinary authorities from pursuing a disciplinary action against the attorney (D) No, because representation of clients whose interests are only economically adverse, such as representation of competing economic enterprises in unrelated litigation, does not ordinarily constitute a conflict of interest

(D) No, because representation of clients whose interests are only economically adverse, such as representation of competing economic enterprises in unrelated litigation, does not ordinarily constitute a conflict of interest The correct answer is D. The fact that two clients dislike each other, or that two clients own competing businesses, does not create a conflict of interest for the lawyer. Comment 6 to Model Rule 1.7 addresses this very point: "On the other hand, simultaneous representation in unrelated matters of clients whose interests are only economically adverse, such as representation of competing economic enterprises in unrelated litigation, does not ordinarily constitute a conflict of interest and thus may not require consent of the respective clients." Of course, if one client pressures a lawyer to provide less zealous representation to the other, this could create a material limitation on the attorney's ability to represent both, which would then trigger the conflicts prohibition in Rule 1.7—no such facts exist here. Some clients may choose to use a different lawyer than their main business rival, but without a clear conflict of interest, the lawyer does not have to notify either client that he represents the other. Answer D is therefore the correct answer to this question, as it reflects the guidance set forth in Comment 6 to Model Rule 1.7. Answer A, in contrast, states the opposite—it wrongly asserts that representation of business rival constitutes a conflict of interest without consent. Answer B is wrong for two reasons—state disciplinary authorities do not use a subjective test for conflicts of interest (the client's perceptions), but rather follow objective guidelines, and the standard for proving a conflict of interest in a malpractice lawsuit would be the same. Answer C incorrectly suggests that malpractice lawsuits preclude disciplinary actions for the same incident or malfeasance. On the contrary, a lawyer could face both a disciplinary action by the appropriate professional authority and a malpractice lawsuit from a client over the same ethical violation or incident.

A client hired a lawyer to defend him in a criminal matter involving assault charges. During the lawyer's interviews and investigation for this case, he learned that the client had also been committing identity theft and credit card fraud, obtaining credit cards in the names of other individuals and running up charges on the cards without paying the bills so that the individuals whose names are on the cards would have to pay the debts. The lawyer urged the client to stop this practice, but the client just laughed at him. The lawyer continued his representation of the client and won an acquittal on the assault charges. The representation is now over. May the lawyer warn some of the individuals in whose names the client has obtained credit cards, according to the Model Rules of Professional Conduct? (A) Yes, because the lawyer's representation of the client has ended, and the information he wants to disclose is unrelated to the matter for which he represented the client (B) Yes, because a lawyer may reveal information relating to the representation of a client to the extent the lawyer reasonably believes necessary to prevent the client from committing a crime or fraud that is reasonably certain to result in substantial injury to the financial interests or property of another (C) No, because the information he wants to disclose is unrelated to the matter for which he represented the client, and it is not certain that substantial injury to the financial interests of others will occur (D) No, because the exception that permits disclosure to prevent a client from injuring the financial interests of another applies only when the client has used (or is using) the lawyer's services in furtherance of this crime or fraud

(D) No, because the exception that permits disclosure to prevent a client from injuring the financial interests of another applies only when the client has used (or is using) the lawyer's services in furtherance of this crime or fraud The correct answer is D. Rule 1.6(b)(2) provides one of the narrow exceptions to the general rule of client confidentiality: "... to prevent the client from committing a crime or fraud that is reasonably certain to result in substantial injury to the financial interests or property of another and in furtherance of which the client has used or is using the lawyer's services." Generally, this allows lawyers to be "whistleblowers" in certain situations, such as corporate securities fraud; the exception thus attempts to eliminate tension between modern whistleblower statutes and the general duty of confidentiality. The last phrase of this exception, however, is very important—it applies only to situations in which the client is using, or has used, the lawyer's services to perpetrate the crime or fraud involved. In this question, the lawyer's representation entails defending the client against criminal assault charges—it does not appear that the client used the lawyer's services at all in his credit card fraud. Thus, the exception does not apply, making Answer D correct—the lawyer should not warn the potential victims of the client's credit card fraud, at least according to the Model Rules, because it would be an impermissible disclosure of confidential client information. Answer A is incorrect because it wrongly suggests that the confidentiality requirement ends with the conclusion of the representation. Lawyers have an ongoing duty to protect client confidentiality even after the representation ends. Answer B invokes the right exception—the wording comes directly from 1.6(b)(2)—but leaves out the last phrase of the exceptions, which is a crucial limitation on its applicability. Answer C is tricky because the credit card fraud itself is not part of the matter for which the client hired the lawyer—confidentiality applies only to information "related to the representation"—but given that the lawyer discovered the information as part of his factual investigation for the assault charges, the information is "related to the representation" for purposes of the confidentiality rule.

A corporate executive hired an attorney to represent him in a criminal matter. The client faced charges over corporate fraud that he perpetrated two years before when he was the chief financial officer of a large, publicly traded corporation. The attorney learns during his interviews with the client that the fraud will have some far-reaching consequences for investors and another large corporation in the area, consequences that the prosecution and regulatory authorities have overlooked so far. The attorney realizes that if he discloses this information now, he could prevent substantial injury to the financial interests or property of innocent people and that harm is reasonably certain to result otherwise. According to the Model Rules of Professional Conduct, may the attorney disclose the information to prevent this substantial injury to the financial interests of others? (A) Yes, because a lawyer may reveal information relating to the representation of a client to the extent the lawyer reasonably believes necessary to prevent substantial injury to the financial interests or property of innocent people (B) Yes, because a lawyer may reveal information relating to the representation of a client to the extent the lawyer reasonably believes necessary when the client has committed a crime or fraud that is reasonably certain to cause substantial financial injury to others (C) No, because a lawyer may never reveal information relating to the representation of a client, even if the lawyer reasonably believes necessary, when the client has committed a crime or fraud that is reasonably certain to cause substantial financial injury to others (D) No, because the exception that permits disclosure to prevent substantial financial harm to others does not apply unless the client used the lawyer's services in furtherance of the crime or fraud

(D) No, because the exception that permits disclosure to prevent substantial financial harm to others does not apply unless the client used the lawyer's services in furtherance of the crime or fraud The correct answer is D. Comment 8 to Rule 1.6 explains the exception to the confidentiality rule in cases where disclosure is necessary to prevent financial harm to others. The exception does not apply to lawyers who are hired to defend clients who have already committed the crime or fraud before retaining the lawyer, even if the consequences for others are still unfolding. Answer D is therefore correct because it best captures the rule about when the exception might not apply, even though financial injury is still going to occur. Answer A, on the other hand, merely states the "financial harm" exception and ignores this important limitation on when the exception applies. Answer B makes a similar mistake—stating part of the financial harm exception without the caveat that it does not apply to lawyers hired to defend the client against charges relating to the crime or fraud in question. Answer C goes to the opposite extreme, incorrectly suggesting that there is no exception, when in fact there is—even though the exception does not apply here.

Clint is a convicted felon serving a 30-year sentence in prison. Clint discharged the lawyer who lost his criminal trial and recently hired Attorney Bates to handle his appeal in federal circuit court. Attorney Bates has filed a preliminary notice of appeal, but briefs in the appeal are not due for several months and oral argument will not occur until two or three months thereafter. Yesterday, Attorney Bates received court appointments to handle last-minute appeals in three high-profile death penalty cases in which the executions are on the schedule for the next few weeks. Attorney Bates also took on a complex class action suit by prisoners against the state Department of Corrections, which if successful would pay Attorney Bates several million dollars in statutory legal fees. Given the urgency of the death penalty cases and the potential fees from the class action suit, Attorney Bates decides to transfer Clint's appeal of his life sentence to another competent lawyer, who is glad to take on the case. Clint refused to grant Attorney Bates permission to withdraw as counsel, though. Attorney Bates then mailed a letter to Clint explaining that he was withdrawing from the case, included all documents and papers relating to the representation, and filed a motion to withdraw in the appellate court. Did Bates violate the ethical rules by attempting to withdraw from the case over his client's objection? (A) Yes, because a lawyer must comply with the rules requiring permission of a client when terminating a representation (B) Yes, because Attorney Bates agreed to represent Clint first and could have declined the new cases if he were fulfilling his duty of loyalty to the client (C) No, because if the client is in fact guilty of the crime, he or she would be using the lawyer's services to perpetrate a fraud for the lawyer to reverse the client's conviction (D) No, because the withdrawal of representation in this case presents no material adverse effect on the interests of the client

(D) No, because the withdrawal of representation in this case presents no material adverse effect on the interests of the client The correct answer is D. Rule 1.16(b)(1) permits lawyers to withdraw from representation as long as the withdrawal will not present the client with undue adverse consequences. Attorney Bates seems to have good cause for the withdrawal due to the urgency of the court appointments in death penalty cases. Attorneys have a duty to manage their own caseload, which means sometimes lawyers will need to refer clients to other attorneys to handle their case. On the other hand, the condition in Rule 1.16 is intended to prevent lawyers from dropping clients capriciously, especially in situations where the client would likely suffer serious adverse consequences, such as missing a filing deadline or having to find another lawyer in the middle of a trial. Answer D is the best answer because Attorney Bates has satisfied the condition in Rule 1.16 that the client will suffer no material adverse effect from his withdrawal. Answer A is incorrect because Attorney Bates did not need the client's permission to withdraw from the representation—the client's permission is not, in fact, one of the criteria (or relevant factors) in the rule provisions about withdrawal. Similarly, Answer B is incorrect because there is no "first in time" rule regarding attorney representation—the fact that an attorney agreed to represent a client does not mean that the attorney cannot end that representation at some appropriate time (i.e., between a trial verdict and the appeal) in order to help other new clients. Finally, Answer C is incorrect—providing a legal defense for a client who is in fact guilty of the crime does not make the lawyer an accessory to the crime itself or mean that the lawyer is helping the client perpetrate a crime or fraud; the lawyer is merely ensuring that the client receive due process and that the rule of law is honored in the proceedings.

Litigation was underway between two parties, and the plaintiff sought to recover a significant sum from the personal assets of the defendant. The judge issued an order that the parties could not transfer any assets out of the jurisdiction. Two weeks later, the defendant's attorney learned from her client's spouse that the client had transferred hundreds of thousands of dollars to secret offshore bank accounts in the Cayman Islands. Although neither the attorney nor the client had made any affirmative representations to the court about following the court's order, it is clear to the attorney that the court and the opposing party are under the impression that both parties are complying with the court's order and are relying upon that fact in the ongoing proceedings. The client did not use the attorney's services in any way to make the transfers, and the attorney did not recommend it or know about it until after it occurred. Would it be proper for the attorney to do nothing and say nothing about the matter at this time, in order to protect the client's confidential information? (A) Yes, because the client has not made any false statements to the court (B) Yes, because the attorney has not made any material misrepresentations to the court (C) No, because a lawyer always has a duty to inform the court if a client is engaged in illegal or fraudulent conduct, even if it is unrelated to the attorney's representation (D) No, because this is a circumstance where failure to make a disclosure is the equivalent of an affirmative misrepresentation

(D) No, because this is a circumstance where failure to make a disclosure is the equivalent of an affirmative misrepresentation The correct answer is D. ABA Formal Ethics Opinion, Op. 98-412, which added a clarification to Model Rule 3.3, is relevant to this question. In some instances, silence can create or reinforce a false impression for the court, and the duty of candor requires the lawyer to make a proactive disclosure to clarify the matter, even though no false evidence has been submitted. The facts in this question echo the example discussed in Op. 98-412, in which the ABA Ethics Committee stated that failure to make a disclosure about the client's surreptitious violation of the court's order would constitute a misrepresentation, and therefore would violate Rule 3.3 (Comment 3 to Rule 3.3 had made the same point, albeit in passing, without offering examples). Answer D accurately reflects the ABA's conclusion for these situations. Answer A is incorrect, although this would be true in many other situations—normally, the lawyer has a duty to correct false statements by the client to the court but not to correct factual mistakes of the other side that are advantageous to the lawyer's client. In this case, however, the lawyer has an affirmative duty to reveal what the client has done, because otherwise there is a legal presumption that the parties are complying with the court's order. Similarly, Answer B wrongly suggests that the rule applies only to statements the lawyer makes in the proceedings, but the ABA has clarified that there are situations in which silence could constitute misrepresentation, and the lawyer has a duty to disclose the information. Answer C is incorrect because it is overly broad, as the duty of candor does not usually apply to matters unrelated to the representation of the client, and there are exceptions delineated in the Comment to Rule 3.3.

Attorney Abrams had a disagreement with a judge one day during a trial, and soon it turned into an intense argument in the courtroom. In the heat of the moment, the judge lost his temper in front of the jury and called Attorney Abrams "an embarrassment to the profession and a menace to his own clients." Indignant, Attorney Abrams made an obscene gesture and shouted that the judge was corrupt and too old to remain on the bench. Later, when each of them regained his composure, they apologized to each other and to the jury. Nevertheless, the lawyer serving as opposing counsel reported Attorney Abrams to the state bar disciplinary authority, but she did not report the judge, for fear of retaliation in future cases in that court. Attorney Abrams believes he will not be subject to discipline for his actions. Is he correct? (A) Yes, opposing counsel refrained from reporting the judge, which unfairly places all the fault on Attorney Abrams. (B) Yes, Attorney Abrams apologized to the judge as soon as he regained his composure, and a lawyer should not have to passively accept abuse from a judge in front of a jury. (C) No, because Attorney Abrams escalated the verbal argument by making an obscene gesture. (D) No, even though it is appropriate for a lawyer to stand firm against abuse by a judge, a lawyer must not reciprocate or escalate the matter.

(D) No, even though it is appropriate for a lawyer to stand firm against abuse by a judge, a lawyer must not reciprocate or escalate the matter. Answer D is correct. ABA Model Rule 3.5(d) says that lawyers must not "engage in conduct intended to disrupt a tribunal." Comment 4 to Model Rule 3.5 explains: The advocate's function is to present evidence and argument so that the cause may be decided according to law. Refraining from abusive or obstreperous conduct is a corollary of the advocate's right to speak on behalf of litigants. A lawyer may stand firm against abuse by a judge but should avoid reciprocation; the judge's default is no justification for similar dereliction by an advocate. An advocate can present the cause, protect the record for subsequent review and preserve professional integrity by patient firmness no less effectively than by belligerence or theatrics. Answer A is incorrect because the failure to report the judge does not mean that Attorney Abram's conduct should go unreported. There is no requirement that reporting a violation must include every person involved in the incident; besides, if the state disciplinary authorities inquire into the incident with Attorney Abrams, they will necessarily discover the judge's contribution to the acrimony. Answer B is wrong because subsequent apologies do not undo the violation that occurred, although a disciplinary panel may find this to be a mitigating factor when deciding on an appropriate punishment. Finally, C is incorrect because it suggests that escalation is a necessary element of the rules, whereas the rules indicate that merely reciprocating a judge's attack constitutes is a violation.

A client hired Attorney Andrews to handle several real estate transactions. Once the representation was underway, the client explained that the transactions are all part of a money laundering scheme and that the money ultimately is being used to fund terrorist activities. May Attorney Andrews continue with the representation, if each individual transaction appears to be technically legal? (A) Yes, because Attorney Andrews was not aware of the criminal purpose of the transactions when he consented to the representation (B) Yes, but Attorney Andrews may withdraw if he finds the course of action repugnant (C) No, Attorney Andrews must withdraw because the transactions involve a conflict of interest (D) No, if Attorney Andrews's services are being used to assist the client in conduct that the lawyer knows is criminal or fraudulent

(D) No, if Attorney Andrews's services are being used to assist the client in conduct that the lawyer knows is criminal or fraudulent ABA Model Rule 1.2(d) prohibits a lawyer from assisting a client in conduct that the lawyer knows is criminal or fraudulent. If the lawyer learns that the lawyer's services are assisting the client in such conduct, ABA Model Rule 1.16 requires the lawyer to withdraw from the representation because continued representation would violate Rule 1.2(d). Lawyers cannot excuse their participation in crimes or fraud by saying that they were merely doing what the client wanted. For this reason, Answer D is the best answer. Even though the individual transactions appear to be legal, the transactions are (by the client's own admission) part of an overall criminal enterprise. If the lawyer does not withdraw immediately from the representation, he could face both disciplinary action for violating the ethical rules of his profession and criminal charges as an accessory. Answer A wrongly suggests that a lawyer can continue with the representation if the criminal purpose was not apparent to the lawyer at the outset of the representation. A lawyer has a duty to decline representation if it is clear that the representation will facilitate illegal activities by the client and a duty to withdraw from the representation if this becomes clear sometime later. Answer B is incorrect, though it correctly states the rule for "permissive" withdrawals when the client's actions are repugnant to the lawyer. When the representation involves illegal activity, as here, it triggers the mandatory withdrawal provisions of Rule 1.16, making the permissive withdrawal provisions inapplicable. Finally, Answer C wrongly casts this question as a conflict of interest problem, but the issue is much more serious than that. A lawyer is not allowed to perform illegal actions on behalf of a client or to aid the client in committing crimes or fraud.

While serving as in-house counsel for a corporation, an attorney discovered that a regional manager had taken several actions that potentially violated state and federal laws. The manager had a reputation for being arrogant and unreasonable, though he was exceptional in his area of expertise and was an asset to the company, despite his unpleasant demeanor. The attorney had clashed with him a few times in the past, and now they were barely on speaking terms. The attorney summoned the nerve to confront the manager about the wrongdoing. The regional manager's initial response was to be dismissive, saying that he was unaware of any laws or regulations that he might have violated. The attorney walked away from the conversation discouraged and planned to take the matter up with the corporate officers, and perhaps the Board of Directors. Before doing so, he reconsidered and returned to the manager and patiently explained to him the relevant laws and regulations that the manager had violated. The manager begrudgingly accepted the attorney's advice and took all necessary measures to rectify the wrongdoing and prevent any long-term repercussions. The manager also insulted the attorney, called him incompetent for not bringing up the matter earlier, and suggested that the attorney's incompetence was due to the attorney's ethnic background. Could the attorney be subject to discipline for not referring the matter of the illegal actions to a higher authority in the corporation? (A) Yes, because referral to a higher authority in the corporation is part of the lawyer's professional duty under the Model Rules (B) Yes, because the manager continued to insult him and behave like a bigot even after the attorney proved that the manager's actions violated the law (C) No, because a lawyer for a corporation represents not only the corporation itself, but all the managers within the corporation, so the lawyer had a direct client-attorney relationship with the manager (D) No, if the circumstances involve a manager's misunderstanding of law and subsequent acceptance of the lawyer's advice, the lawyer may reasonably conclude that the best interest of the organization does not require that the matter be referred to higher authority

(D) No, if the circumstances involve a manager's misunderstanding of law and subsequent acceptance of the lawyer's advice, the lawyer may reasonably conclude that the best interest of the organization does not require that the matter be referred to higher authority The correct answer is D. Comment 4 to Rule 1.13 observes that when a lawyer discovers illegal actions by someone in the corporation (the corporate client), ordinarily this will require referral to a higher authority. Even so, in some circumstances the lawyer can ask the wrongdoer to reconsider the matter, and if this approach proves successful, then the lawyer may not need to escalate the matter to protect the best interests of the corporate client. In other words, a lawyer does not have an ethical obligation to escalate a matter if the person responsible for the incident is amenable (perhaps with some persuading) to rectifying the problem and resolving the issue. Answer D is therefore the right answer as it appears that the manager did not understand the law at first and eventually accepted the attorney's position and took steps to fix the problem. The attorney does not have to proceed to a higher authority in the corporation in that case. Answer A is incorrect because it ignores the exception set forth in Comment 4 to the lawyer's usual duty of escalating the matter. Even though the attorney would otherwise have a duty to take the matter to a higher authority (for a recalcitrant wrongdoer), in this case that next step is unnecessary because the wrongdoer agreed to rectify the illegal actions and resolve the issue. Answer B wrongly suggests that the manager's inappropriate insults would make it necessary for the attorney to bring the original wrongdoing to the attention of higher authorities. While the lawyer is free to discuss the regional manager's inappropriate insults to his superiors, escalating the original illegal actions is not necessary for protecting the organization's best interests, given that the manager plans to fix the problem. Answer C is incorrect because it states that the managers are the clients of the attorney along with the organization as a whole, and this is usually not the case; generally, when the organization is the client, its constituents (such as managers or employees) are not clients and do not have an attorney-client relationship with the lawyer.

An attorney specializes in intellectual property law, representing both inventors and venture capitalists in tech startup businesses. Even though the attorney represents only one or the other side in each transaction, she may represent an inventor in one contract with a venture capitalist and represent that venture capitalist in drafting agreements with other inventors. The attorney has drafted a standardized "waiver of future conflicts" form that she asks all clients to sign along with their retainer agreement at the beginning of representation. The waiver of conflicts form explicitly consents to representation despite any and all conflicts of interest that might arise regarding the attorney's past, present, or future clients. When an actual conflict of interest or adverse relationship exists between clients at the outset of representation, she carefully explains the situation to new clients and encourages them to seek advice from other counsel about signing the waiver. When no present conflicts are apparent, but only hypothetical potential conflicts are at issue, the attorney merely says that the form is for hypothetical, potential conflicts of interest that probably will not arise in the current transaction. Is this attorney's standardized "waiver of future conflicts," when signed by new clients, likely to be effective in this situation? (A) Yes, because whenever the client agrees to consent to a particular type of conflict with which the client is already familiar, the consent ordinarily will be effective with regard to that type of conflict (B) Yes, because the attorney explains obvious, existing conflicts to new clients and then uses an all-inclusive, open-ended waiver form for unforeseen conflicts of interest (C) No, because it violates the Rules of Professional Conduct for a lawyer to ask a client to waive future claims such as a conflict of interest, unless the client has representation by outside counsel in deciding whether to sign the waiver (D) No, if the consent is general and open-ended, then the consent ordinarily will be ineffective, because it is not reasonably likely that the client will have understood the material risks involved

(D) No, if the consent is general and open-ended, then the consent ordinarily will be ineffective, because it is not reasonably likely that the client will have understood the material risks involved The correct answer is D. Comment 22 to Model Rule 1.7 gives guidance about asking clients to consent in advance to future conflicts of interest. The bottom line is that waivers of future conflicts are more likely to be effective when they describe specific foreseeable conflicts so that the client could reasonably understand what is at stake. 'Effective' consent means that when the lawyer's actions later come under scrutiny, perhaps through a motion for disqualification or a legal malpractice lawsuit, it will validate the lawyer's decision to proceed with the representation even after the conflict arose. Comment 22 adds this cautionary note: "If the consent is general and open-ended, then the consent ordinarily will be ineffective, because it is not reasonably likely that the client will have understood the material risks involved." Answer D reflects the very wording of Comment 22, and the facts in this question fit—the waiver forms used by the lawyer are too vague and open-ended and are therefore ineffective. Answer A, in contrast, also uses verbiage from Comment 22, but it does not fit with the facts described in the question—the lawyer is not asking clients to consent to a specific type of conflict with which the client is already familiar but instead is asking for a waiver of any conflict that might arise, including novel or unforeseeable conflicts. Answer B is tricky but is incorrect—even though the attorney explained existing conflicts of interest in detail to her clients, the written consent she had each one sign was an open-ended waiver of future conflicts (not yet materialized) that she did not explain. Answer C incorrectly states that valid waivers of future conflicts always require representation by another lawyer. Even though Comment 22 mentions that a client simultaneously represented by another lawyer is more likely to result in the conflict waiver being effective, it is just one scenario in which a waiver of future conflicts would work. A lawyer could obtain a valid consent to a future conflict of interest from a client who is not represented by other counsel if the lawyer describes a specific conflict that is reasonably familiar to that client.

Mr. Barrows asked Big Bank for a loan using farm land and farm machinery, which Barrows claims to own, as a security interest. Big Bank had a lending policy that required borrowers to provide at the closing a lawyer's opinion letter (from the borrower's lawyer) that Big Bank's new mortgage lien on the security property will have priority to any other recorded liens. Barrows hired Attorney Abrams to provide the opinion letter. The letter that Attorney Abrams provided met Big Bank's requirements, but it included a disclaimer that the attorney has neither physically inspected the property nor investigated the state of the record title with respect to the mortgaged property, but that instead he had relied on the preliminary title report of a title-insurance company to the effect that there are no other liens on the property and that title to the property is vested in the client. After the closing, it turned out that a third party had previously acquired rights in the property by adverse possession. Worse, the adverse possessor had constructed some structures on the property that resulted in the filing of other contractor's and seller's liens on the property after the date of the preliminary title report. Attorney Abrams did not know any of this when he drafted the opinion. Which of the following is correct, based on these facts? (A) There was no attorney-client relationship in this case, so there is no conflict of interest here that would require informed consent. (B) The attorney had a conflict of interest in the representation described here, because he worked for the client at the behest of the lender. (C) Attorney-client privilege covers the contents of the report to the lender, if the lender kept the report confidential, so it cannot furnish the basis of a claim against the attorney. (D) The attorney did not violate a duty of care to the lender by relying solely on the preliminary title report and not conducting any other investigation, because the opinion letter expressly disclosed this limitation.

(D) The attorney did not violate a duty of care to the lender by relying solely on the preliminary title report and not conducting any other investigation, because the opinion letter expressly disclosed this limitation. Answer D is the best answer. The Restatement (Third) of the Law Governing Lawyers § 95 (An Evaluation Undertaken for a Third Person) covers this type of scenario in some of its illustrations, which explains: The factual basis of an evaluation is that which it states, expressly or by implication from the circumstances. In all events, unless stated or agreed otherwise, a lawyer's evaluation does not entail a guarantee by the lawyer that facts stated in it are accurate. In some circumstances, such as when the lawyer purports to be making a report of a factual investigation undertaken by the lawyer, a reasonable reader of the report would assume that the lawyer is reporting facts known by the lawyer to be accurate. A lawyer normally may rely on facts provided by corporate officers and other agents of a client that the lawyer reasonably believes to be appropriate sources for such facts without further investigation or specific disclosure, unless the recipient of the opinion objects or the version of the facts provided, or other circumstances indicate that further verification is required. Id. Note that without Attorney Abram's express disclaimer, he could have been liable to the lender, as the Restatement says in § 51: When a lawyer or that lawyer's client (with the lawyer's acquiescence) invites a nonclient to rely on the lawyer's opinion or other legal services, and the nonclient reasonably does so, the lawyer owes a duty to the nonclient to use care, unless the jurisdiction's general tort law excludes liability on the ground of remoteness. Accordingly, the nonclient has a claim against the lawyer if the lawyer's negligence with respect to the opinion or other legal services causes injury to the nonclient (see § 95). The lawyer's client typically benefits from the nonclient's reliance, for example, when providing the opinion was called for as a condition to closing under a loan agreement, and recognition of such a claim does not conflict with duties the lawyer properly owed to the client. Allowing the claim tends to benefit future clients in similar situations by giving nonclients reason to rely on similar invitations. . . . In some circumstances, reliance by unspecified persons may be expected, as when a lawyer for a borrower writes an opinion letter to the original lender in a bank credit transaction knowing that the letter will be used to solicit other lenders to become participants in syndication of the loan. Answer A is incorrect because the lawyer would, in fact, owe a duty of care to the lender (see above) in this case, but the disclaimer included in the report sufficiently put the lender on notice of the limitations of its content. Answer B is wrong because the Model Rules and the Restatement both contemplate that clients will often ask lawyers to prepare reports for third parties, as in this scenario, and there is no conflict of interest, although the lawyer's duty to his client is paramount. Answer C is incorrect because the client and lawyer waived privilege by submitting the report to the lender.

Attorney Thompson agreed to represent a new client named Collins. Collins needed a lawyer to represent him in a regulatory takings case involving changes in zoning and land use rules that interfered with Collins's intended use of his property. Attorney Thompson was a recent law school graduate and had her own small law firm. She had never handled a regulatory takings case before. Thompson explained her lack of experience to Collins. Thompson agreed to take Collins's case and to do the research necessary to get up to speed on the law in that area. As she began researching the area, Thompson found it terribly confusing. She eventually sought help from a law school friend who worked in the field. The other lawyer had an impressive familiarity with the law of regulatory takings and offered to join Attorney Thompson as co-counsel in the case in exchange for an even split in the fees. Thompson agreed to this arrangement privately with the other lawyer, even though Collins was not aware of it, and together the two lawyers achieved exactly the result that Collins sought in the case. At the end of the representation, Thompson's bill to the client was exactly what they had agreed in the fee agreement, and Collins paid the amount set forth in the fee agreement. Attorney Thompson then split the amount with the other lawyer who helped her. Is Thompson subject to discipline? (A) No, because it was proper for Attorney Thompson to bring in co-counsel with more expertise in the area of law (B) Yes, because Thompson did not know the area well enough to provide representation and was unable to teach herself the relevant law as she promised Collins she would do (C) No, because Thompson did in fact make a good-faith effort to learn the area of law as she had promised (D) Yes, because Thompson did not first obtain informed consent from Collins to include the other lawyer in the representation

(D) Yes, because Thompson did not first obtain informed consent from Collins to include the other lawyer in the representation The correct answer is D. Model Rule 1.1 states, "A lawyer shall provide competent representation to a client. Competent representation requires the legal knowledge, skill, thoroughness and preparation reasonably necessary for the representation." This does not mean that a lawyer cannot venture into a new area of law. Rather, the Comments to Rule 1.1 contemplates the very situation described here, in which a lawyer would essentially teach herself enough of the new field to provide competent representation. Comment 4 states that a lawyer "may accept representation where the requisite level of competence can be achieved by reasonable preparation." Further, the Comments recognize that lawyers may seek the input of other attorneys with more experience or expertise in the unfamiliar field of law. The problem with the attorney in this question is that she agreed to bring another lawyer into the representation (even to the point of splitting the fees) without telling the client. Answer D is correct because it reflects the requirement set forth in Comment 6 to Rule 1.1: "Before a lawyer retains or contracts with other lawyers outside the lawyer's own firm to provide or assist in the provision of legal services to a client, the lawyer should ordinarily obtain informed consent from the client and must reasonably believe that the other lawyers' services will contribute to the competent and ethical representation of the client." Clients have a right to know which lawyer is actually doing their legal work. Moreover, not asking the client first could violate other rules as well, such as the duty of confidentiality and to avoid conflicts of interest. Some of the other answers require careful reading because they refer to portions of the relevant rule but ignore the issue of client consent. Answer A is incorrect because even though it was proper to bring in another lawyer with more expertise (that part of the answer is correct), it was improper to do so without first obtaining the client's consent. Some clients might object to the inclusion of the other lawyer, due to confidentiality concerns, issues related to costs, or conflicts of interest with that other lawyer or the other lawyer's clients. Answer Bmisstates the rules—lawyers can indeed take on a case involving an unfamiliar area of law if they either teach themselves the necessary knowledge to carry out the representation or seek help from a more knowledgeable lawyer. Answer C is incorrect. Even though the lawyer did make a good-faith effort to learn the area of law, the answer ignores the consent issue. If another lawyer participates in the representation, the client must first consent.

Client is a second-year law student at a state law school. Client's Professional Responsibility professor forbids the use of the Internet by students during class sections. The school's student handbook also strictly forbids use of the school's wireless computer network, which provides the only Internet access inside the building, during class sessions unless the professor permits it. Client visited a social networking site during a class session, and when the professor discovered it, he had the student arrested for violating the state's Computer Fraud and Abuse Act, which imposes civil and criminal penalties for unauthorized use of a government computer network. Client hired Attorney Jones to represent him. Attorney Jones is shocked that the police and prosecutor are involved in such a ridiculous case and is reasonably certain a judge would dismiss the charges before trial. The prosecutor called Attorney Jones and explained that the district attorney regards this as an important test case and wants to bring it to trial, but they will offer a plea bargain of only twenty years in prison if the student will plead guilty and accept responsibility. Attorney Jones blurted out a profanity and hung up on the prosecutor. He did not mention the offer to Client, out of fear that it would upset him, and instead drafted a motion to dismiss. The court granted the motion and dismissed the charges against Client. Is Attorney Jones subject to discipline? (A) No, because the prosecutor's offer was unreasonable and the case was frivolous so there was no duty to discuss such an offer with the client (B) No, because the dismissal of the charges in this case meant that the client was far better off than if he had considered the plea bargain offered by the prosecutor (C) Yes, because a lawyer should show respect and decorum toward opposing parties and lawyers, and using profanity or hanging up on someone clearly violates the Rules of Professional Conduct (D) Yes, because a defense lawyer who receives a proffered plea bargain in a criminal case must promptly inform the client of its substance, unless the client has previously told the lawyer to accept or to reject the offer

(D) Yes, because a defense lawyer who receives a proffered plea bargain in a criminal case must promptly inform the client of its substance, unless the client has previously told the lawyer to accept or to reject the offer The correct answer is D. A point emphasized in the ABA's Comments to the Model Rules is that lawyers must tell the client about any offers to settle (civil cases) or for a plea bargain. Of course, a client could pre-authorize a lawyer to accept or reject certain offers, but otherwise, the lawyer cannot reject offers without first asking the client—even offers that seem unreasonable. Comment 2 to Rule 1.4 describes this duty in absolute terms. Answer D correctly states the rule—the defense lawyer cannot reject an offer for a plea bargain from the prosecutor, regardless of how unreasonable the offer seems, without first communicating the offer to the client. Of course, the lawyer could urge the client to decline the offer, but the client must be included in this decision. Answer A is incorrect because even if an offer seems ridiculous to the lawyer, the client has a right to learn about it and decide whether to reject it. Answer B is also incorrect because the ethical rules do not contain an exception for outcomes that seem objectively better for the client—the client's right to know and decide is paramount. Finally, Answer C is incorrect, even though it correctly states that the lawyer also had a duty to be respectful toward opposing counsel, because this answer omits the important, bright-line rule that a lawyer must consult with a client before rejecting any plea-bargain offer, unless the client previously instructed the lawyer to reject such an offer.

After an uncontested divorce, a client hired an attorney to represent her in litigation over the custody of her children. The client was concerned about her older sister, in whom she had confided about her struggles with substance abuse and mental illness. She and her sister had stopped speaking to each other before the client got married and had children because of an intense argument over money. The client had been free from substance abuse since she married and was now managing her mental health issues very well. The attorney located the sister, explained that the client was fighting for custody of her children, and that he expected the ex-husband's lawyer would call her to testify about the client's former troubles at the hearing. The attorney pleaded with her to keep her sister's confidence and refuse to testify about these matters. The attorney reasonably believed that the interests of the sister would not be adversely affected by refraining from giving such information. The sister felt deeply moved by this entreaty and agreed to stay out of the litigation. Would the ethical rules permit the attorney to request that the sister withhold information from the court or the other party? (A) No, because the lawyer has a duty to think about the best interests of the children in this case, rather that his client's convenience or feelings (B) Yes, because in a divorce or custody proceeding, a lawyer may ask various potential witnesses to refrain from disclosing information in order to keep the proceedings from becoming unnecessarily acrimonious or protracted (C) No, because a lawyer may not request a person other than a client to refrain from voluntarily giving relevant information to another party (D) Yes, because a lawyer may request that a relative of the client refrain from voluntarily giving relevant information to another party

(D) Yes, because a lawyer may request that a relative of the client refrain from voluntarily giving relevant information to another party The correct answer is D. This question echoes the previous one to highlight an exception in the rules. Model Rule 3.4(f)(1) generally prohibits lawyers from asking potential witnesses to refrain from divulging information to the court or the other party. There are two exceptions: family members or employees/agents of the client. Even with these exceptional cases, there is an important qualification that the lawyer cannot ask the client's family member or employee to keep something quiet if doing so runs counter to that individual's own legal interests. Answer D is correct because the lawyer's actions here fall within the exception to the general prohibition in Rule 3.4(f). Answer A wrongly suggests that the lawyer has an ethical duty to consider the best interests of the children in this case—the lawyer's duty is to the client's interests and to uphold the candor and fairness provisions of Rules 3.1-3.9. Answer B is incorrect because there is no distinction made in the rules for certain areas of law, such as custody—the prohibition and the exceptions for discouraging potential witnesses from talking apply regardless of the type of case. Answer C states the general provision from Rule 3.4(f) but ignores the important exception for family members of the client.

An attorney represents a large corporate client. The company has an employee hotline for complaints of workplace harassment, so that the company's management can quickly investigate the complaint as a personnel matter and can have their attorney review the complaints for potential claims or lawsuits against the corporation. An employee used the hotline to allege that her supervisor was harassing her. The management referred the complaint to the attorney for review, and after some inquiries and preliminary investigation, the attorney concluded the complaint was not valid. It turned out the employee was chronically late for work and had received a written reprimand from the supervisor, without further action or discussion. The attorney drafted a memorandum to the management detailing her investigation—the conversations with the employee, the supervisor, and some co-workers in that department—and explained that there was no basis for a lawsuit by the employee. Nevertheless, the employee eventually quit and filed a lawsuit against the corporation, and her lawyer sought to compel discovery of the memorandum by the company's attorney. Is the memorandum protected by attorney-client privilege? (A) No, because it includes statements made by the opposing party, as it recounts the lawyer's conversations with the employee after she made the original hotline complaint (B) No, because privilege applies only to statements from the client to the lawyer, not to communications from the lawyer to the client or their duly authorized agents (C) Yes, because the lawyer put the information in written form, which triggers attorney-client privilege (D) Yes, because corporate management had consulted with the lawyer to obtain legal advice and assistance, and the confidential communication was the lawyer's response to the client's inquiry

(D) Yes, because corporate management had consulted with the lawyer to obtain legal advice and assistance, and the confidential communication was the lawyer's response to the client's inquiry The correct answer is D. Attorney-client privilege allows a client to refuse to disclose information in litigation, or prevent others (such as the client's lawyer) from disclosing the information, if the information was part of a confidential communication between the client and the attorney for the purpose of obtaining legal advice or legal services. In this question, Answer D is the right answer—the corporation's managers (they are the agents of the client) referred the hotline complaint to the attorney to review the matter for potential legal liability, and the attorney's memorandum provided the advice that the client sought. This is a classic case of privileged information—the legal advice an attorney communicates to the client privately at the client's request. Answer Ais incorrect because the communication itself—the memorandum—was a private communication between the lawyer and the client. Note that the opposing party's statements to the lawyer might be admissible on their own if disclosed by the complainant herself, but the lawyer's version of the conversations—and the analysis of their legal significance—are privileged. Answer B incorrectly suggests that communications by the lawyer are not privileged. In previous centuries, some common law courts took this view, but modern courts consider private communications from lawyers to clients—like the one here—to come under the attorney-client privilege. Finally, Answer C is wrong because it suggests that privilege only applies to written communications. Attorney-client privilege covers both oral and written communications between the client and the lawyer if they are private and for the purpose of obtaining legal advice.


Conjuntos de estudio relacionados

Federal Tax Considerations for Life Insurance and Annuities

View Set

Exam 2, 4: completing the accounting cycle

View Set

CIT A+ Final Exam Question - Part Four

View Set

Primerica Health License: State Laws, Rules and Regulations

View Set